Download as pdf or txt
Download as pdf or txt
You are on page 1of 122

EBD_7711

• Corporate Office : 45, 2nd Floor, Maharishi Dayanand Marg, Corner Market, Malviya Nagar,
New Delhi-110017
Tel. : 011-49842349 / 49842350

Chief Editor: Satya Prakash


Editor: Khurshid Ali

Typeset by Disha DTP Team

DISHA PUBLICATION
All RightS Reserved

© Copyright Publisher

No part of this publication may be reproduced in any form without prior permission of the publisher. The author and the
publisher do not take any legal responsibility for any errors or misrepresentations that might have crept in. We have
tried and made our best efforts to provide accurate up-to-date information in this book.

For further information about the books from DISHA,


Log on to www.dishapublication.com or email to info@dishapublication.com
Contents
Section I – Basic Numeracy & Data Interpretation
Module 1 – Alligation and Mixture (Quick Revision Notes/ pdf)........................................... 1-2

Module 2 – Time Sequence (Clock and Calendar) (Quick Revision Notes/ pdf).................... 3-4

Module 3 – Permutation and Combination (Quick Revision Notes/ pdf).............................. 5-6

Module 4 – Table, Bar, Line, Pie and Mixed Graph (Quick Revision Notes/ pdf).................. 7-9

Module 5 – Miscellaneous Graph (Quick Revision Notes/ pdf)............................................. 10-12

Module 6 – Mean, Median, Mode (Quick Revision Notes/ pdf).......................................... 13-15

Quiz 1 on Basic Numeracy & Data Interpretation (15 Qns. With Solutions/ pdf)............ 16-19

Section II – Comprehension, Decision Making & Interpersonal Skills


Module 7 – Interpersonal & Communication Skills (Quick Revision Notes/ pdf)............. 20-23

Module 8 – Administrative Course of Action (Quick Revision Notes/ pdf)..............................24

Module 9 – Decision Making (Quick Revision Notes/ pdf)........................................................25

Module 10 – General Comprehension (Quick Revision Notes/ pdf).................................... 26-27

Module 11– English Language Comprehension (Quick Revision Notes/ pdf)................... 28-29

Quiz 2 on Comprehension, Decision Making & Interpersonal Skills (15 Qns. With Solutions/
pdf)............................................................................................................................................. 30-32

Section III – Mental Ability, Logical Reasoning & Problem Solving


Module 12 – Blood Relation (Quick Revision Notes/ pdf)..........................................................33

Module 13- Direction &Distance (Quick Revision Notes/ pdf)........................................... 34-35

Module 14 – Cube & Dice (Quick Revision Notes/ pdf)......................................................... 36-38

Module 15 – Evaluating Inferences (Quick Revision Notes/ pdf)..............................................39

Module 16 – Critical Reasoning (Quick Revision Notes/ pdf)............................................ 40-44

Quiz 3 on Mental Ability, Logical Reasoning & Problem solving (15 Qns. With Solutions/
pdf)............................................................................................................................................. 45-47

Previous Years Questions (with Solutions) /pdf................................................................... 48-105

TEST 1 – GENERAL APTITUDE (GS PAPER II) (30 Qns. with Solutions/ pdf)..................... 106-112

TEST 2 – GENERAL APTITUDE (GS PAPER II (30 Qns. with Solutions/ pdf)....................... 113-117
EBD_7711
1

Aptitude Test
Basic Numeracy & Data Interpretation
Module–1 : Alligation & Mixture

ALLIGATION
Alligation is the simplified, faster technique to solve the problems
based on weighted average. This method plays a vital role in saving
the time in solving the problems related to weighted average situation.
We know that
The ratios in the bracket [ ] are equal i.e.
Sum total of all numbers of all groups
Weighted Average = Total number of numbers in all
n1 : n2 = A2 – Aw : Aw – A1.
groups together In the above graphical representation five variables A1, A2, Aw, n1
and n2 are involved.
Therefore weighted average Aw of two groups having n1 and n2
numbers with averages A1 and A2 respectively is Illustration – 2
n1 A1 + n2 A2 If the average weight of the students of a class is 15 kg, the average
Aw = weight of the students of another class is 30 kg and average weight
n1 + n2
of the students of both the classes is 25 kg, then find the ratio of
Þ (n1 + n2) Aw = n1 A1 + n2 A2
Þ n1 (Aw – A1) = n2(A2 – Aw) the number of students of the first class to the another class.
n1 A2 - Aw
Þ = Solution :
n2 Aw - A1
n A - Aw
Equation 1 = 2 is called Alligation Formula.
n2 Aw - A1
For convenient, we take A1 < A2. Hence A1 < Aw < A2.

SOLVING THE PROBLEMS USING


ALLIGATION FORMULA Þ n1 : n2 = 5 : 10 = 1 : 2

Illustration – 1 RECOGNITION OF DIFFERENT


10 kg of wheat costing ` 12 per kg and 15 kg of wheat
SITUATIONS WHERE ALLIGATION
costing ` 20 per kg are mixed. Find the average cost of CAN BE USED
the mixture per kg. There are many types of situations where alligation can be used,
n1 A2 - Aw 10 20 - Aw which must be recognised by the students. Here you are given
Solution : n = A - A Þ 15 = A - 12 some situations (or problems) which help you to recognise
2 w 1 w
different alligation situations and identify A1, A2, n1, n2 and Aw in
2 20 - Aw each alligation situation.
Þ = Þ 5 Aw = 84
3 Aw - 12 In each of the following problems
84
A1 = 20, A2 = 35, n1 = 20, n2 = 40 and answer as Aw = 30.
Þ Aw = = 16.8
5 Weighted Mean
Hence average cost of the mixture = ` 16.8 per kg.
An average weight of students of a class of 40 students is 35
kg and an average weight of students of a class of 20 students
GRAPHICAL REPRESENTATION OF
is 20 kg. Find the average weight of the students of both the
ALLIGATION– CROSS METHOD combined classes. (30 kg)

The alligation formula


n1 A2 - Aw
= is graphically represent-
MIXTURE
n2 Aw - A1
(i) 20 litres of one variety of soda water is mixed with 40 litres
ed by the following cross-diagram: of other variety of soda water. The price of first variety of
EBD_7711
2
soda water is ` 20 per litre and price of other variety of Profit and Loss
soda water is ` 35 per litre. Find the cost of the mixture (i) A car agency sold 20 cars at 20% profit and 40 cars at 35%
per litre. (` 30) profit. Find the gain percent on the sale of all these cars.
(ii) A 40 litres mixture of water and milk contains 35% of milk (30%)
and in another 20 litres of mixture of water and milk (ii) A trader earns a profit of 20% on 20% of his goods sold
contains 20% of milk. If a new mixture is formed by mixing
while he earns a profit of 35% on 40% of his goods sold. Find
the both mixtures, then find the percentage of milk in new
the percentage profit on the total goods sold. (30%)
mixture. (30%)
(iii) A shopkeeper sold the 40% hardware at the profit of 35%
Time and Speed and 20% software at a profit of 20%. Find the average
A car travels at 20 km/h for 20 minutes and at 35 km/h for 40 profit% on the whole goods sold, if he sells only these
minutes. Find the average speed of the car for the whole two kind of things. (30%)
journey. (30 km/hr)
required ratio = 1 : 2
3
Module – 2 : Time Sequence (Clock and Calendar)
æ 5ö
x - ç 65 + ÷
è 11 ø
CLOCKS Times loss per misnute =
x
The concept of clocks is the same as the concept of circular motion. Just
like in circular motion, in case of clocks, the hour hand and the minute CALENDARS
hand are continuously racing against each other around a circle.
The topic of calendar includes concepts of leap year and odd
The hour hand travels 5 minutes or 30º in 1 hour. Similarly, the days.
minute hand travels 60 minutes or 360º in 1 hour. Therefore, in
one hour, the minute hand moves 55 minutes or 330º more than LEAP YEAR
the hour hand. This can also be called the relative speed of minute
An end of the century year (the last year of the century, e.g.
hand with respect to the hour hand. 1500, 1800, 2000 etc.) is a leap year only if divisible by 400.
For all the other years, check the divisibility by 4, and if the
Minute and Hour hand of a clock
year is divisible by 4 it is said to be a leap year and will have
366 days. This would mean that the year 1500 and 1800 inspite
Movement Relative speed of of being divisible by 4 not a leap year.
Actual
minute hand with
Time
respect to hour hand CONCEPT OF ODD DAYS:
Minute Hour
Hand Hand When a given number of days converted into week(s) by dividing
60 60 minutes or 5 minutes 55 minutes or 330º 7, then the remaining days which are not covered in week(s) are
minutes 360º or 30º called odd days.
(i) An ordinary year has 365 days, that is 52 weeks and one
odd day. This means out of the 365 days in an ordinary year,
CONCEPT OF INCORRECT CLOCKS 364 days will get converted into 52 weeks and one day will
remain. This one day is referred to 1 odd day. This means
Two hands of every correct clock coincide after every
that when we proceed from ordinary year to the next year,
æ 5ö the name of the first day of the next year will be the name of
ç 65 + ÷ minutes
è 11 ø the day which will come one day after the first day of previous
An incorrect clock can either be a fast clock which gains time (ordinary) year.
or a slow clock which loses time. For example, if 2 October 2015 is a Friday, then 2 October
2016 is a Sunday, that is a shift of two days because 2016 is
FAST CLOCK a leap year.
(ii) A leap year has 366 days, that is 52 weeks and two odd
In fast clock, two hands of a clock coincide every x minutes,
days. This means that when we proceed from leap year to
æ 5ö the next year, the name of the first day of the next year will
where x is less than ç 65 + ÷ minutes.
è 11 ø be the name of the day which will come two days after the
5 first day of previous (leap) year.
65 + - x
Time gained per minute = 11 For example, if 26th January 2016 is a Saturday, then 26th
x January 2017 would be a Monday, that is a shift of two days
because 2016 is a leap year.
SLOW CLOCK
(iii) If name of the day of a particular date is given and we have
In slow clock, two hands of a clock coincide every x minutes,
to find the name of the day of an other particular date, then
æ 5ö we need to look at not only the years but also the date from
where x is greater than ç 65 + ÷ minutes
è 11 ø which we are moving to the other given date. If the 29th of
EBD_7711
4
February falls between the two dates, there will be a shift of Solution : This is a leap year. that will give 2 odd days, So,
two days, otherwise there will be a shift of one day. next 3 years will give one odd day each. Then leap year gives
(iv) In ordinary year, 2 odd days.
1 year = 365 days = 52 weeks + 1 day Therefore (2 + 3 + 2) odd days = 7 odd days will be there.
(i.e. one odd day) Hence, the day of week will be Tuesday.
In leap year,
1 year = 366 days = 52 weeks + 2 days
(i.e. two odd days)
Illustration
Today is 5th February. The day of the week is Tuesday. This
is a leap year. What will be the day of the week on this date
after 5 years?
5
Module – 3 : Permutation & Combination Each of the different selections or groups which can be made by
some or all of a number of given things without consideration of
the order of things in any selection or group is called a
FACTORIALS combination.
As in selection order in which things are selected is not considered;
If n is a natural number then the product of all natural numbers hence, selections of two letters AB and BA out of three letters A, B
upto n is called factorial n and it is denoted by n ! or and C are the same. Similarly selections of BC and CB are the
Thus, n ! = n (n – 1) (n – 2) ..... 3.2.1 same. Also selections of CA and AC are the same.
Note that 0! = 1 = 1! Hence selection of two letters out of the three letters A, B and C
n! = n (n – 1)! can be made as AB, BC and CA only.
= n (n – 1) (n – 2)! Number of combinations (or groups) of two letters out of three
= n (n – 1) (n – 2) (n – 3)!, etc. letters A, B and C = 3.
For example : 6! = 6 × 5 × 4 × 3 × 2 × 1 COUNTING FORMULA FOR
But 4! = 4 × 3 × 2 × 1
\ 6! = 6 × 5 × 4! or 6 × 5 × 4 × 3! PERMUTATIONS
Remember that
0 ! = 1, 1 ! = 1, 2 ! = 2, 3 ! = 6, 4 ! = 24, 5 ! = 120, 6 ! = 720, etc. WITHOUT REPETITION
MEANING OF PERMUTATION AND 1. Number of permutations of n different things, taking r at
a time is denoted by nPr or P(n, r), which is given by
COMBINATION
n n!
Each of the different arrangements which can be made by taking Pr = (0 £ r £ n),
some or all of a number of things is called a permutation. (n - r )!
Note that in an arrangement, the order in which the things where n is a natural number and r is a whole number.
arranged is considerable i.e., arrangement AB and BA of two 2. Number of arrangements of n different objects taken all
letters A and B are different because in AB, A is at the first place at a time is nPn = n !
and B is at the second place from left whereas in BA, B is at the
first place and A is at the second place. Note : n P1 = n and n Pn = n Pn –1
The all different arrangements of three letters A, B and C are Illustration – 1
ABC, ACB, BCA, BAC, CAB and CBA.
Here each of the different arrangements ABC, ACB, BCA, BAC, Find the number of ways in which four persons can sit on six
CAB and CBA is a permutation and number of different chairs.
arrangement i.e. 6 is the number of permutations.
ABC, ACB, BCA, BAC, CAB and CBA are different arrangements Solution : 6P4 = 6.5.4.3 = 360
of three letters A, B and C, because in each arrangement, order
in which the letters arranged, is considered. But if the order in WITH REPETITION
which the things are arranged is not considered; then ABC, ACB,
BCA, BAC, CAB and CBA are not different but the same. 1. Number of permutations of n things taken all at a time,
As in arrangements, order in which things are arranged is if out of n things p are alike of one kind, q are alike of
considered. Hence all arrangements of two letters out of the second kind, r are alike of a third kind and the rest
three letters A, B and C are AB, BA, BC, CB, CA and AB. n!
\ Number of permutations (or arrangements) of two letters out of n – (p + q + r) are all different is
three letters A, B and C = 6. p ! q! r !
2. Number of permutations of n different things taken r at
a time when each thing may be repeated any number of
times is nr.
Illustration – 2

Find the number of words that can be formed out of the letters
of the word COMMITTEE taken all at a time.
Solution : There are 9 letters in the given word in which
two T’s, two M’s and two E’s are identical. Hence the required
EBD_7711
6
9! 9! 9! where n is a natural number and r is a whole number.
number of words = = = = 45360 W
2! 2! 2! (2!)3 8 Some Important Results
(i) nC =1 (ii) nC0 = 1 (iii) nC = nCn –
NUMBER OF PERMUTATIONS n r r

UNDER CERTAIN CONDITIONS Illustration – 5

1. Number of permutations of n different things taken all How many different 4-letter words can be formed with the
together when r particular things are to be placed at some letters of the word ‘JAIPUR’ when A and I are always to
r given places = n – rPn – r = (n – r)! be included ?
2. Number of permutations of n different things, taken r at Solution : Since A and I are always to be included, so first we
a time, when a particular thing is to be always included
select 2 letters from the remaining 4, which can be done in 4C2
in each arrangement, is
r. n – 1Pr – 1. = 6 ways. Now these 4 letters can be arranged in 4! = 24 ways,
So the required number
Illustration – 3 = 6 × 24 = 144.
How many different words can be formed with the letters of the Illustration – 6
word ‘JAIPUR’ which start with ‘A’ and end with ‘I’?
How many combinations of 4 letters can be made of the letters
Solution : After putting A and I at their respective places (only of the word ‘JAIPUR’ ?
in one way) we shall arrange the remaining 4 different letters at
4 places in 4! ways. Hence the required number = 1 × 4! = 24. Solution : Here 4 things are to be selected out of 6 different
6.5.4.3
Illustration – 4 things. \ number of combinations = 6C4 = = 15
4.3.2.1
How many different 3 letter words can be formed with the
letters of word ‘JAIPUR’ when A and I are always to be 2. Selection of Objects with Repetition
excluded? The total number of selections of r things from n different
Solution : After leaving A and I, we are remained with 4 things when each thing may be repeated any number of times
different letters which are to be used for forming 3 letters is n + r – 1Cr
words. 3. Restricted Selection
Hence the required number
(i) Number of combinations of n different things taken r
= 4P3 = 4 × 3 × 2 = 24.
at a time when k particular things always occur is
n – kC
r – k.
CIRCULAR PERMUTATIONS
(ii) Number of combinations of n different things taken r at
Total number of ways in which n different things can be arranged a time when k particular things never occur is n – kCr.
in a circle = (n –1)!
4. Selection From Distinct Objects
For example: If six persons are to be seated on a circular table
for dinner, the number of ways of doing so is (6–1)! = 5! Number of ways of selecting at least one thing from n different
= 5×4×3×2×1 = 120 ways. things is
nC + nC + nC + .....+ nC = 2n – 1.
1 2 3 n
COUNTING FORMULA FOR This can also be stated as the total number of combination of
n different things is 2n – 1.
COMBINATION
Illustration – 7
1. Selection of Objects Without Repetition
Ramesh has 6 friends. In how many ways can he invite one or
The number of combinations or selections of n different more of them at a dinner ?
things taken r at a time is denoted by nCr or C (n, r) or
Solution : He can invite one, two, three, four, five or six friends
æ nö at the dinner. So total number of ways of his invitation
Cç ÷
è rø = 6C1 + 6C2 + 6C4 + 6C5 + 6C6 = 26 – 1 = 63
n!
where nC = ; (0 £ r £ n),
r r ! (n - r )!
7

Module - 4 : Table, Bar, Line , Pie & Mixed Graph


In Data Interpretation, data are represented in various type of tables and graphs, so that the desire information can be easily
obtained from it.
Data Interpretation questions are based on the information given in these tables and graphs.

Presentation of Data

Tables Graphs

Bar Graph Line Graph Pie Graph Combined


(x-y Graph) or Graph
Pie Chart

1. BG + LG
Simple Bar Subdivided Multiple 2. BG + PG
Graph Bar Graph Bar Graph 3. LG + PG
etc.

TABLES (i) Simple Bar Graph


It is used to represent only one dependent variable.
A table is one of the easiest way for summarising data. Example:
A statistical table is the logical listing of related quantitative The following bar-graph, showing the number of students
data in vertical columns and horizontal rows of numbers with
in five sections A to E of a class in a school.
sufficient explantory and qualifying words, phrases and state-
ments in the form of titles, heading and notes to make clear the
meaning of data.

GRAPHS
Graphs are a convenient way to represent information. The graphs
should be labelled properly to show what part of the graphs shows
what value.

1. BAR GRAPHS
(ii) Sub-divided Bar Graphs
Bar diagram consist of a number of equidistant rectangles.
These are used to represent the break down of a total into
One for each category of the data in which the magni-
its comrponent bars. A bar is divided into different seg-
tudes are represented by the length or height of rectangle, ments, each segment respresenting a given component of
whereas width of rectangles are immaterial. Thus, a bar the total is of different shades, colours, designs etc. are
is just one dimensional as only the length of the bar is to used to distinguish the various components. An index is
given to represent the various components. To compare,
be considered and not the width. All the bars drawn in a the order of various components in the different bars is
diagram are generally of uniform width which depends same.
on the number of bars to be constructed and the availabil- Example:
ity of the space. The graph given below shows the yearly details of money
Types of Bar Graphs are- invested in producing a certain product over the years
2011 to 2015. It also gives the profit (in ‘000 rupees).
EBD_7711
8
3. PIE GRAPH (PG) OR PIE CHART
250 Pie graph is a pictorial representation of numerical data by
200 non-intersecting adjacent sectors of circle. Area of each
Profit sector is proportional to the magnitude of the data repre-
150 Interest
Overheads sented by the sector.
100 Wages 360º
50 Raw-material (i) Central angle of 1% of total value = = 3.6°
100
0 (ii) The % of components parts can be converted to de
–50 grees by multiplying each of them by 3.6°.
2011 2012 2013 2014 2015
(iii) Degree of any component part
(iii) Multiple Bar Graph (MBG) component value
= ×360° .
When a combination of inter-related variables are to be Total value
represented graphically, multiple bar diagrams are used. Example:
These are extended form of simple bar diagrams. In M.B.G. In the chart given below; A, B, C, D and E represents the
many aspects of the data are presented simultaneously costs of paper, printing, binding, miscellaneous and the
with separated bars of various shades of colours. An in- royalty respectively in publishing a book.
dex is given to explain the shades & colours used.
Example:
The graph below gives the data of the number of employ-
ees working in a company the total expenditure of the E A
company, and the total salary paid to the employees by D
the company over the years.
B
500 C
400 No of employees
of the company
300
Total exp. of the TOOLS FOR INTERPRETATION OF DATA
200 company in lakh
The tools which are mostly used for repersentation of data
100 Total salary paid to
the employees by (or a group of data) are
0 the company in ` 0000 1. Ratio 2. Percentage
3

3. Average 4. Combined Average


-1

-1

-1

-1

-1
12

13

14

15

16
20

20

20

20

20

1. RATIO
2. LINE GRAPH (LG) Ratio is the comparison between two quantities of the same
Line graphs are more obvious, precise & accurate than kind in terms of their magnitudes (which are in the same
the diagrams. LG are used to show how a quantity changes unit). The ratio of two quantities is equivalent to a fraction
that one quantity is of the other.
very often. If the line goes up, the quantity is increasing. For example, let Swati has 5 note books and Priya has 7
If the line is horizontal, the quantity is not changing. note books. Then the ratio of the number of books that
Example: have with Swati to the number of books that have with
The line graph given below shows the revenue (in mil- 5
lions $) generated by five restaurants in its take away Priya is 5 is to 7. This ratio is expressed as 5:7 or , which
7
and Dine in facility. The graph gives the break-up of the is a quotient of 5 and 7.
revenue generated through Take away and Dine in facility l In the ratio of two quantities, the two quantities must

of five restaurants in Indian and Chinese foods. be of the same kind and in the same unit.
l The ratio is a pure number i.e. without any unit of measurement.
l The ratio would stay unaltered even if both the nu-
merator and the denominator are multiplied or divided
by the same number.
Comparision of Ratios
a c a c
l > ,if ad > bc l < ,if ad < bc
b d b d
6 3
For example: > because 6×5 > 7×3
7 5
4 7
and < because 4×8 < 5×7
5 8
9
2. PERCENTAGE Value after x% increase
Original Value =
The word percent means per hundred or for every hun- x
1+
dred. The symbol % is used for the term percent. Thus 100
20 percent is written as 20% and it means 20 out of 100. l To find the original value, if the value after its x% de-
x crease is known.
x% of y means ×y Value after x% decrease
100 Original value = x
20 1–
Hence 20% of 600 = ×600 =120 100
100 3. AVERAGE
35 An average of a group of numbers is a number that is the
35% of 880 = ×880=308 best representative of the group of numbers because it
100
tells a lot about the entire numbers of the group.
20
20% of 600 kg Sugar = ´ 600kgSugar = 120 kg Sugar Sumof allnumbers
100
Average of a group of numbers = Number of numbers
For interpretation of data involving the percentage follow the
following rules: Thus if A is the average of n numbers a1, a2, a3,....., an;
Rule - 1 then
l To find by how much percent x is more than y or over y.
Required Percentage a 1 + a 2 + a 3 + ..... + a n
A=
( Value of x ) – ( Value of y) n
= ×100% ( Here x > y )
( Value of y ) Þ a1 +a 2 +a 3 +....+ a n = A.n
l To find by how much percent x is less than y Hence, sum of numbers = (Number of numbers) × (Av-
Required Percentage erage)
( Value of y) – ( Value of x ) For example, average of 6 to 10 natural numbers
= ×100% (Here x < y)
( Value of y) 6+7+8+9+10 40
Illustration = = =8
5 5
Find by how much percent 60 is more than 50? 4. COMBINED AVERAGE
Sol. Required percentage If we have two or more groups of numbers whose
60 – 50 10 indivisual average are known, then average of all the num-
= ×100% = ×100% = 20% bers of all the groups is called Combined Average.
50 50 Thus, if there are k groups having number of numbers n 1,
Rule - 2 n2, n3,...., nk with averages A1, A2, A3,...., Ak respectively,
l To find the percentage change in any value in year x com- then Combined Average or Weighted Average or Average
pared to that value in year y (over year y). mean which is represented by Aw.
Required percentage change
n 1 A 1 + n 2 A 2 + n 3 A 3 +....+ n k A k
(Value in year x) – (Value in year y) =
= ×100% n 1 +n 2 +n 3 +....+ n k
(Value in year - y)
l If the required percentage change obtained is positive, then For example, if average of weight of 5 boys students is
there is percentage increase in the value in year x over the 40 kg and that of 10 girls students is 43 kg, then average
year y. weight of all the 15 students i.e.
If the required percentage change obtained is negative, 5×40+10×43 630
then there is percentage decrease in the value in year x over Combined Average = = = 42 Kg
5+10 15
the year y.
Rule - 3
l To find the original value, if the value after its x% increase
is known.
EBD_7711
10
Module – 5 : Miscellaneous Graphs In this graph the value of x is increases when value of
y increases.
(ii) Non-linear Graph is a graph that does not show
REPRESENTATION OF A GRAPH a fixed pattern of increase or decrease.
Y (Vertical Axis)
Y Y
3 P(2, 3)

1
0 X (Horizontal Axis)
–2 –1 0 1
–1
2 3
Q(– 2, – 1) X X
–2 (a) (b)
–3
Fig. : Various types of non-linear increasing graphs.
Fig. : x and y axes of a graph
Graph (a) shows an overall increase but there is a small
The point O represents the origin. Both the axes are perpendicular dip in the graph which indicate decrease. The graph but
to each other.
On the right hand side of the origin of the x-axis, the value of does not show a fixed pattern between X & Y.
numerals is positive while that on the left hand side are negative. Graph (b) shows an increase in both X & Y but without a
Similarly, on the y-axis, the values above the origin are positive fixed pattern of dependency.
while that below the origin are negative.
Point P(2, 3) on the graph is located at 2 units of the x-axis from 2. DECREASING GRAPH
the origin towards the right hand side and 3 units of the y-axis
upwards from the origin. The decreasing graph shows when one quantity increases
While point Q (–2, – 1) on the graph is located 2 units on x-axis then other quantity decreases and vice-versa. This can also
to the left of the origin and 1 unit on the y-axis below the origin.
be divided into two types - Linear & non-linear.
Let us now understand with the help of a few examples what type
of graphs may appear to test our knowledge. (i) Linear Decreasing Graph

1. INCREASING GRAPH Y
Y
The increasing graph shows as one quantity increased other
quantity also increases. This type of graph may be of 2
types – linear or non-linear.
(i) Linear Graph is a graph that shows a directly
proportional relation between 2 quantities and is
denoted by a straight line.
Y X
X

Fig. : Linear decreasing graph


In this graph, when x increases, y decreases.

X
Fig. : Linear increasing graph.
11
(ii) Non-linear Decreasing Graph = 45°C while at time, t = 35 min. temperature = 17°C, therefore,
the curve A shows a linear decreasing graph.
In case of curve B, there is no further decrease in the
temperature with increasing time, so curve B can be known
Y Y as either an increasing time graph or a constant graph.

3. CONSTANT GRAPH
The constant graph is a curve which shows constant value
of one quantity while the other quantity may increase or
decrease.

X X (i) Increasing Constant Graphs


(a) (b)
Y Y
Fig. : Various types of non-linear decreasing graphs.

Graphs (a) & (b) are 2 curves showing decreasing non-


linear relationship.
Illustration – 1
What happens when an Air conditioner is switched on in X X
(a) (b)
summers in a room having temperature of 45°C (assuming
all the other factors governing the working of an AC like
Fig. : Increasing constant graphs.
filters, refrigerants etc. are in a perfect condition)? Explain
with the graph. Graph (a) shows a constant value of y while an
Solution : increasing value of x.
Graph (b) shows a constant value of x while an
increasing value of y.
(ii) Decreasing Constant Graphs
45
40 Y Y
35
A
30
Temperature (°C)

25
20
15
10 B X X
5 17°C (a) (b)

0 Fig. : Decreasing constant graphs


5 10 15 20 25 30 35 40 45 50
Time (min) Graph (a) shows a constant value of y while a decrease
in the value of x.
Graph (b) shows a constant value of x while a decrease
The graph has 2 parts - A & B - both being linear curves. in the value of y.
In a summer afternoon when a room is at 45°C (temperature
shown by y-axis), it takes about 30-35 minutes (time shown 4. UNIFORM GRAPH
by the x-axis) to cool down the room the minimum A uniform graph is a curve shows uniform pattern of increase
temperature to which the temperature drops to in a regular or decrease or constant curve.
AC is about 16-17° C. A linear graph is an example of a uniform graph.
Thus the temperature drops from 45°C to 17°C a total time
span of about 35 minutes, i.e., at time, t = 0 min. temperature
EBD_7711
12
Illustration – 2 Solution :
The given graph shows a distance - time curve for a man where We can observe from the given graph that for every 5 hours
distance is given on the Y-axis while time is shown on the X-axis. of time passed, 10 km of distance is covered by the man.
Explain its uniformity. This pattern is maintained further on as well. The man
covers in the next 5 hours (10 – 5 = 5), 10 km (20 – 10)
again and similarly does till 40 km and 20 hrs.
40 Thus this type of journey will be called a uniform journey,
i.e., every point denotes 10 km covered in 5 hours.
30
Distance (km)

20

10

0
5 10 15 20
Time (hours)
13
Module – 6 : Mean, Median, Mode MEDIAN
Median of a set of numbers (or data or values or observation) is
the middle most number (or data or value or observation) in a
MEAN, MEDIAN, MODE set of numbers, when the numbers are arranged either in
Mean, median and mode are the three measures of central tendency ascending or in descending order of their magnitude.
of the data. Method to find the median
When the numbers (or data) is arranged in ascending or
MEAN descending order, then median is calculated as follows:
Mean is actually the average. An average of a group of numbers (i) When the number of data (n) is odd, then the median is the
(or data or values or observations) is a number that is the best th
æ n +1ö
representative of the group of numbers because it tells a lot about value of the ç
è 2 ÷ø
data.
the entire numbers of the group.
(ii) When the number of data (n) is even, then the median is
Sum of all numbers
Mean of a group of numbers = Number of numbers th
n th æn ö
the mean of and ç + 1÷ data.
Thus if A is the mean of n numbers a1, a2, a3, ..., an; then 2 è2 ø
a1 + a 2 + a 3 + ... + a n
A= é th th ù
n i.e. median = 1 ê n data + æç n +1ö÷ data ú
2 ëê 2 è2 ø úû
Þ a1 + a 2 + a 3 + ... + a n = A.n
Hence, Sum of numbers = (Number of numbers) × (Average) Illustration – 1
For example, average of 6 to 10 natural numbers
The monthly salaries (in `) of 10 employees of a factory are:
6 + 7 + 8 + 9 +10 40 12000, 8500, 9200, 7400, 11300, 12700, 7800, 11500, 10320,
= = =8
5 5 8100. Find the median salary.
Solution:
Some Important Results About Mean Arranging the observation in ascending order:
(i) If each observation is increased by ‘a’, then the mean is 7400, 7800, 8100, 8500, 9200, 10320, 11300, 11500, 12000,
also increased by ‘a’. If x is the mean of n observations x1, 12700
x2, ...., xn, then the mean of observations (x1 + a), (x2 + a), Total number of observations (n) = 10 (even)
(x3 + a), ...., (xn + a) is ( x + a).
1 éæ n ö ù
th th
æn ö
(ii) If each observation is decreased by ‘a’, then the mean is \ median = êç ÷ observation + ç + 1÷ observation ú
2 êëè 2 ø è2 ø úû
also decreased by ‘a’. If x is the mean of n observations
x1, x2, ...., xn, then mean of observations (x1 – a), (x2 – a),
1 éæ 10 ö ù
th th
...., (xn – a) is ( x – a). æ 10 ö
= êç ÷ observation + ç + 1÷ observation ú
(iii) If each observation is multiplied by a non-zero number ‘a’, 2 ëêè 2 ø è 2 ø
ûú
Then, mean is also multiplied by ‘a’.
1 é th
If x is mean of n observations x1, x2, ...., xn, then mean of = 5 observation + 6th observation ù
2ë û
ax1, ax2, ....., axn is a. x .
1 19520
(iv) If each observation is divided by a non-zero number ‘a’, Median = [ 9200 + 10320] = = 9760
then, mean is also divided by the non-zero number ‘a’. 2 2
Median Salary = ` 9760
If x is mean of n observations x1, x2, ...., xn, then the mean
x1 x2 x x
MODE
of , , ..... n , is .
a a a a The mode of a group of numbers (or data or observations) is that
number (or data or observation) which occurs most frequently
i.e. which comes maximum number of times.
EBD_7711
14
Illustration – 2 (i) Data skewed to the Left
(ii) Data skewed to the Right
Find the value of mode of the following data
50, 70, 50, 70, 80, 70, 70, 80, 70, 50
Solution: DATA SKEWED TO THE LEFT
To find mode, we prepare ungrouped (or discrete) frequency table. If in a distribution of data 5 is given once, 6 is given twice, 7 is
given thrice, 8 is given four times, 9 is given seven times and 10
Observation Frequency is given eight times, then its bar graph will be as shown below.
50 3
70 5
80 2 8
7
In the above table we see that observation 70 is repeating 6
maximum number of times i.e. frequency of 70 is maximum. 5
Hence the mode of the given set of observation is 70. 4
Relationship Between Mean, Mode and Median 3

Mode = 3 Median – 2 Mean 2


1
Illustration – 3
If the value of mode and mean is 60 and 66 respectively, 5 6 7 8 9 10
then find the value of median.
Solution:
Mean = 8.52
Mode = 3 Median – 2 Mean Median = 9
Mode = 10
1 1 The bar graph above is an illustration of a special kind of data
\ Median = ( mode + 2 mean ) = ( 60 + 2 ´ 66) = 64
3 3 distribution. The distribution has the property that in every case,
as the values increases, the frequencies increases as well. This
SYMMETRICALLY (OR NORMALLY) means that on the bar graph, the columns get taller as we look
DISTRIBUTED DATA from left to right. Such type of distribution of data is called
skewed to the left.
If we represent the given data through bar graph, and the left Here we see that the mode (10) is the greatest of the three measures
side of this bar graph is the mirror image of its right side. Then of central tendency, the mean (8.52) the least of the three measures
the data is called symmetrically (or Normally) distributed data. of central tendency, and the median (9) is in between. This illustrates
PLACEMENT TEST SCORES a typical property of data which is skewed to the left.
Hence for the distribution of data to be skewed to the left,
Mean<Median<Mode

DATA SKEWED TO THE RIGHT


If a distribution of data are repersented by the bar graph in which
as values increases, the frequencies decreases as well. that is if in
the bar graph, the columns get shorter as we look from left to
right, then the distribution of data is called skewed to the right.
2 3 4 5 6

mean = 4 mode = 4
median = 4
Mean, median and mode of a symmetrically distributed data are always
same. This is the typical property of symmetric distribution of data.
i.e. for a symmetrically distributed data,
Mean=Median=Mode

SKEWED DISTRIBUTION OF DATA


The distribution of data which is not symmetric is called skewed Hence for the distribution of data to be skewed to the right
distribution of data. Mean>Median>Mode
Skewed distribution of data are of two types: Note: there may be exceptions of this trend
15
SUMMARY
Typical Relationships Between Mean, Median and Mode
For Three Special Distributions
EBD_7711
16

Quiz 1 on Basic Numeracy & Data Interpretation


1. The digits of a three-digit number A are written in the reverse 1. Yield rate is zero at B and C
order to form another three-digit number B. If B > A and B– 2. There is no yield with no fertilizer input
A is perfectly divisible by 7, then which of the following is
3. The yield is minimum at D
necessarily true?
(a) 100 < A < 299 (b) 106 < A < 305 4. The yield is neither minimum nor maximum at C
(c) 112 < A < 311 (d) 118 < A < 317 Which of the above statements are correct?
2. An empty fuel tank of a car was filled with A type petrol. (a) 1, 2 and 4 (b) 3 and 4
When the tank was half-empty, it was filled with B type
(c) 2 and 3 (d) 1, 3 and 4
petrol. Again when the tank was half-empty, it was filled
with A type petrol. When the tank was half-empty again, it Directions (Qs. 7 to 8): Study the following chart to answer the
was filled with B type petrol. What is the percentage of A questions given below.
type petrol at present in the tank?
(a) 33.5% (b) 37.5% (c) 40% (d) 50% Village % population below poverty line
3. The ratio of selling price of 3 articles A, B and C is A 45
8 : 9 : 5 and the ratio of percentage profit is 8 : 7 : 14 B 52
respectively. If the profit percentage of A is 14.28% and the C 38
cost price of B is ` 400, what is the overall percentage gain ? D 58
(a) 14.28% (b) 17.87% E 46
(c) 16.66% (d) None of these F 49
4. A contractor employed 25 labourers on a job. He was paid G 51
` 275 for the work. After retaining 20% of this sum, he
distributed the remaining amount amongst the labourers. If
the number of male to female labourers was in the ratio 2 : 3 Proportion of population of seven villages in 2012
and their wages in the ratio 5 : 4, what wages did a female
labourer get ? A
G 13%
(a) ` 15 (b) ` 8 15%
B
(c) ` 14 (d) ` 10 16%
5. A cane contains a mixture of two liquids A and B in the ratio F
13%
7 : 5. When 9 litres of mixture are drawn off and the cane is C
filled with B, the ratio of A and B becomes 7 : 9. How many E 8%
litres of liquid A was contained by the cane initially ? 18% D
(a) 10 (b) 20 17%
(c) 21 (d) 25
6. The yield versus fertillizer input is shown in the graph.
7. In 2013, the population of villages A as well as B is increased
by 10% from the year 2012. If the population of village A in
Yield 2012 was 5000 and the percentage of population below
poverty line in 2013 remains same as in 2012, find
50 approximately the population of village B below poverty
B (Maximum Point)
line in 2013.
(a) 4000 (b) 4500
(c) 3000 (d) 3500
8. If in 2014 the population of village D is increased by 10%
C (Saddle Point) and the population of village G is reduced by 5% from 2012
20
and the population of village G in 2012 was 9000, what is the
8 total population of villages D and G in 2014?
(a) 19770 (b) 19200
0
1 5 10 15 (c) 18770 (d) 19870
Fertilizer input
9. ABC company is soliciting donations to fund natural clamity in
Kerala. More than half of his donations have been in the
Consider the following statements based on this graph: amount of ` 500. The others have been equally divided among
17
values of ` 250 and values of ` 750. Select the statement that a trader will accept the shirts which are good, but
correctly gives a relationship between measures of central ten- Kapur another trader will reject the shirts which have
dency for this distribution. major defects. One shirt is drawn at random from the
(a) The median is greater than the mean. carton, what is the probalility that it is acceptable to
(b) The mean is less than the mode. Kapur?
(c) The mean is equal to the median. (a) 0.76 (b) 0.98
(d) The mode is less than the median. (c) 0.87 (d) None of these
10. A circle with radius 2 is placed against a right angle. Another
13. If 13x + 1 < 2z and z + 3 = 5 y 2 , then
smaller circle is also placed as shown in the adjoining figure.
What is the radius of the smaller circle? (a) x is ncessarily less than y
(b) x is necessarily greater than y
(c) x is necessarily equal to y
(d) None of the above is necessarily true
14. Find the value of xa + yb, if ay = 19683, where y is a multiple of
a and bx = 1024, where b is a factor of x. a, b, x and y being
positive integers-
(a) 1081 (b) 829
(c) 181 (d) 1729
15. A pipe, working at full speed, can fill an empty istern in 2
hours. However, during the first hours it worked at one
twelfth of its capacity, during the second hour at one-nineth
(a) 3-2 2 (b) 4- 2 2 of its capacity, during the third hour at one-sixth of its usual
capacity, during the fourth hour at one-fourth of its usual
(c) 7 - 4 2 (d) 6 - 4 2
capacity and during the fifth hour it was only one-third as
11. Rajdhani express going from Bombay to Delhi stops at 5
efficient as it was supposed to be. A second pipe also
intermediate stations. 10 passengers enter the train during
displayed similar performance, but if it worked at full speed
the journey with ten different ticket of two classes .The
would have filled the empty cistern in 4 hours. Together with
number of different sets of tickets they may have is
a drain pipe that drained water out of the tank at a constant
(a) 15C10 (b) 20C10 rate half of the empty cistern could be filled in 5 hours. All
(c) 30C (d) None of these the three pipes to empty the filled cistern if no other pipe
10
was functioning during the time?
12. A carton consists of 100 shirts of which 84 are good,
(a) 48 hours (b) 36 hours
10 have minor defects and 6 have major defects. Arun,
(c) 30 hours (d) 24 hours
EBD_7711
18

Hints and Solutions


1. (b) Let the 3 digits of number A be x, y and z
(8 + 9 + 5) - (7 + 8 + 4)
Hence A = 100x + 10y + z Therefore % profit = ´ 100
On reversing the digits of number A, we get the number (7 + 8 + 4)
B i.e., z y x.
3
\ B = 100z + 10y + x = ´ 100 = 15.78%
As B> AÞ z > x ...(i) 19
B – A = 99z – 99x = 99(z – x) 2
As 99 is not divisible by 7 4. (b) Number of males = ´ 25 = 10
5
so (z – x) has to be divisible by 7. ...(ii)
Using (i) & (ii), the only possible values of z and x are 3
Number of females = ´ 25 = 15
(8, 1) and (9, 2) 5
So the minimum and maximum range of A are 108 and Amount distributed among males and females
299, which Î 106 < A < 305 = 275 × 80% = ` 220
2. (b) Let the capacity of the tank be 100 litres. Then, Let the wage paid to a male be ` 5x and that to a female be ` 4x.
Initially : A type petrol = 100 litres. Therefore,10 × 5x + 15 × 4x = 220
After first operation: Þ 50x + 60x = 220 Þ x = 2
Wage received by a feamale labourer = 2 × 4 = ` 8
æ 100 ö 5. (c) Suppose the cane initially contains 7x and 5x litres of
A type petrol = ç ÷ = 50 litres;
è 2 ø mixtures A and B respectively.
B type petrol = 50 litres. Quantity of A in mixture left
After second operation:
æ 7 ö æ 21 ö
æ 50 ö = ç 7x - ´ 9 ÷ litres = ç 7x - ÷ litres.
A type petrol = ç + 50 ÷ = 75 litres; è 12 ø è 4ø
è 2 ø Quantity of B in mixture left
B type petrol = (50/2) = 25 litres
After third operation: æ 5 ö æ 15 ö
= ç 5x - ´ 9 ÷ litres = ç 5x - ÷ litres.
è 12 ø è 4ø
æ 75 ö
A type petrol = ç ÷ = 37.5 liters;
è 2 ø æ 21 ö
ç 7x - ÷ 7 28x - 21 7
è 4 ø
æ 25 ö \ = Þ =
B type petrol = ç + 50 ÷ = 62.5 litres. æ 15 ö 9 20x + 21 9
è 2 ø ç 5x - ÷ + 9
è 4ø
\ Required percentage = 37.5%.
Þ 252 x – 189 = 140 x + 147
3. (d) A B C Þ 112 x = 336 Þ x = 3.
8 : 9 : So, the cane contained 21 litres of A.
1 1 1 1 1 5 1
6. (b)
7 8 8 9 4 5
7 8 4 16
7. (d) Population of village B in 2012 = 5000 ´ » 6150
1 13
Since 14.28% =
7 110
So, the ratio of profit percentage of Population of village B in 2013 = 6150 ´ = 6765
100
A B C Population below poverty line = 52% of 6765 » 3500
8 : 7 : 14 (Given)
¯ ¯ ¯ 17
8. (a) Population of village D in 2012 = 9, 000 ´ = 10, 200
15
1 1 1
Population of village D in 2014
7 8 4
Thus the ratio of CP of A : B : C 110
= 10, 200 ´ = 11, 220
7:8:4 100
19
Similarly number of different tickets from second
95 intermediate station = 4
Population of village G in 2014 = 9,000 ´ = 8,550
100 So total number of different tickets
Total population of village D and G in 2014= 11,220 + = 5 + 4 + 3 + 2 + 1 = 15
8,550 = 19,770 And same number of tickets for another class
Þ total number of different tickets = 30 and number of
9. (c) Although we don’t know exactly how many donations selection = 30C10
have been collected, we have enough information to 12. (d) Total no. of shirts = 100
draw a useful bar graph summarizing the distribution. No. of shirts which have major defect = 6
The bar graph will show three values. 250, 500 and 750. \ Good shirts = 94
Since half of the donations are in the amount of ` 500, Probability of a shirt without major defects
the column above the value 500 will be as tall as the
other two columns combined. Moreover, since there 94
= = 0.94
were exactly as many ` 250 donations as there were ` 100
750 donations, the column above the value 250 will be
13. (d) 13x + 1 < 2 z and z + 3 = 5 y 2
the same height as the column above the value 750.
The bar graph looks like this: Þ 13x + 1 < 2(5 y 2 - 3)

Þ 13x + 7 < 10 y 2 Þ 10 y 2 > 13x + 7


In the above equation, all the options a, b & c are
possible but not necessarily true.
Number
of 14. (a) ay = 19683
donation Þ 39 = 273 = (19683)1 = 19683
As y is a multiple of a, a = 3, y = 9
bx = 1024
Þ 210 = 45 = 322 = (1024)1 = 1024
As b is a factor of x, b = 2, x = 10
250 500 750 Now, xa + yb
Þ 103 + 92 = 1000 + 81 = 1081
Value of donation (`)
Recall that this an example of symmetrically (or normally) 15. (d) Protion of tank filled by first pipe in five hours.
distributed data, So the mean, median and mode are all equal. 1é1 1 1 1 1
= ê + + + + ùú
We see that choice (c) is correct. 2 ë 12 9 6 4 3 û
Portion of tank filled by second pipe in five hours
1é1 1 1 1 1
10. (d)
= + + + + ù
4 êë 12 9 6 4 3 úû
O 2 Let the drain pipe, empty the tank in x hours.
A
According to the question,
2 D 1 é 1 1 1 1 1ù 1 é 1 1 1 1 1ù 5
+ + + + + + + + + -
r
r 2 êë12 9 6 4 3 úû 4 êë12 9 6 4 3 úû x
C O' B 1
=
2
OABC is square with side = 2
34 17 10
2 2 + - =1
\ OB = 2 + 2 = 2 2 36 36 x
OB = 2 2 = OD + r + O'B = 2 + r + r 2 51 10
-1 =
36 x
Þ r ( 2 +1) = 2( 2 –1)
15 10
2 =
2( 2 - 1) 2( 2 - 1) 36 x
Þr= = = 2(2 + 1 - 2 2)
( 2 + 1) 2 -1 10 ´ 36
x= = 24 hours
15
=6- 4 2
Hence, The drain pipe will empty the cirstern in 24 hours.
11. (c) For a particular class total number of different tickets
from first intermediate station = 5
EBD_7711
20

Aptitude Test
Comprehension, Decision Making & Interpersonal Skills
Module – 7 : Interpersonal & Communication skills Bureaucrats: Officers of all the services need to interact with their
contemporary, junior and senior bureaucrats. It is indispensable
Interpersonal skills or people’s skills constitute the ability to to interact genially with them, avoiding any conflict.
interact and communicate effectively with another individual Corporates: Officers need to meet senior executives of the
or others within a group. People with strong interpersonal skills corporate sector who will themselves be sharp, ambitious,
are often more successful and happy in both their professional dynamic and charismatic; will sport a high acumen in their field;
and personal lives. Interpersonal skills form the foundation of and will possess a flair for imposing their ideas and having the
organizational functioning. The Administrative Services on last word. It is important to establish a rapport with them and
account of their direct dealing with people require officers who tactfully safeguard their interests.
have strong people’s skills.
Politicians: Civil servants work under ministers who are often
Basically, centred on effective verbal communication, i.e., difficult to deal with. Good interpersonal skills are imperative
efficacious speakin g, focussed listening, and genuine to maintain a cordial relation with these bosses.
questioning, interpersonal skills include non-verbal
Public: Given the numerous diversities prevalent in the country,
communication which pertains to correct perception of body
civil servants may have to deal with an ignorant, unlettered old
language, gestures, facial expressions, eye contact, and the tone villager or a disgruntled aggressive NRI. They need to be
and tenor of speech. Apart from this, they also involve emotional extremely patient and tolerant with everyone.
intelligence (ability to understand and manage own and others’ Communication is the force by which an individual transmits
emotions) and team work (ability to work genially with others stimuli to modify the behavior pattern of other individuals.
in a formal or informal group).To result in an agreeable outcome, –Howland
team work requires ‘Communication’ comes from Latin communicare which means
1. negotiation, persuasion and influencing skills, ‘to share’. Communication is a process by which information
is exchanged between different entities or groups through a
2. conflict resolution and mediation skills,
mutually-understood system of symbols, signs, & behaviour.
3. problem solving and decision making skills The channel of communication can be visual (sight), auditory
(hearing), olfactory (smell) or haptic (touch)/ tactile (like
IMPORTANCE OF INTERPERSONAL Braille).
SKILLS FOR A CIVIL SERVANT
Interpersonal skills are important to solve real life problems.
They show a way out in critical and deadlocked situations.
Effective communication enables one to reach a target audience.
Good communicators can persuade people to favour their
decision, win people over to their side and can build consensus
on issues. An ineffective communication, on the other hand,
may deal a severe blow to one’s efforts and ruin the chances of Communication works both ways—it involves putting across a
the desired outcome. Civil servants have to deal with people point and receiving information & feedback. The receiving,
from different strata of the society for the administration of however, is incomplete until the information has been accurately
public services. So, they must be equipped with efficient interpreted by the recipient.
interpersonal skills to get the expected results from all quarters CONFLICT MANAGEMENT
and to ascertain effective dispensation of their services.
The purpose of including Interpersonal Skills in the paper is to The different contexts, belief systems, values, needs, aspirations
test the candidate’s attitudes, behavioral patterns and ability to and ideas of individuals contribute to conflict which must be
interact with a wide range of disparate personality types. A civil resolved for high productivity. So, conflict management skill
servant, in the dispensation of his duties, is required to meet a is essential for high performance team work.
diverse section of people who can be broadly classified into The following techniques help in conflict resolution:
four types. 1. Don’t aim at winning or proving yourself right
21
2. Don’t look to fix blame; find the root cause of problem
PRINCIPLES OF EFFECTIVE
3. Acknowledge your emotions beforehand to identify the
tangible cause of conflict
INTERPERSONAL FOR CIVIL
4. Collaborate with the other party on how to handle conflict SERVANTS
5. Keep your conversation goal-oriented Communication skills are a prerequisite for a civil servant given
6. Recognise the other person’s emotions and validate it aloud the high responsibility of public welfare that they shoulder and
to assert that you’re a good listener. the challenges they face in dispensing their services to the
complex social fabric of Indian diversities.
7. Admit if you go wrong or could have done better
Certain principles can be laid out or effective Interpersonal
8. Restrain yourself from directly accusing the other party for Communication with all the four categories.
lapses; use terms like ’I guess’, It appears’, ‘I believe’, etc.
(A) As an officer and leader, he should teach others to learn
9. Focus on conduct rather than personality
from their mistakes and not make an issue of it. The civil
10. Allow channels for open conversation to address conflict services environment demands that the officers should
respect each others views on all topics.
STRESS MANAGEMENT
(B) Instead of bickering or finding faults, they should learn to put
The disparity between the demands of the situation and the themselves in the other person's shoes and deal with them
individual’s capabilities for meeting those, leads to stress. Stress with affection and understanding. Such positive emotions will
may be positive (eustress) when accompanied with high help not only others but also the officers themselves.
motivation and the sense of thrill in meeting challenges. Lack of
drive and a feeling of utter hopelessness create negative stress (C) While speaking to individuals, it is not only important to
(distress) and overstrain. express views clearly and succinctly but also to listen
Research suggests the following strategies to manage negative attentively to others. It would be better not to interrupt too
stress: frequently and wait till the opponent has had his say. To
1. Critically examine the problem from different angles to find a express ourselves clearly but briefly without going into a
suitable way forward harangue should be our motto.
2. Plan with prudence to minimize stressful situations (D) If there are differences of opinion, it is important to ensure that
3. Regulate your breathing to relax your muscles and there is no clash of egos. Just analyse the viewpoint only and
calm your mind not the person expressing it. Relationships are strengthened
4. Fortify yourself by reflecting on your high values before and bettered and not soured by such as approach.
facing a stressor
(E) As a civil servant, it would be expected that you would not
5. Employ your signature strength to find a novel way to counter divulge official secrets. This applies to interpersonal
the stressor relationships also. Respect the confidentiality of not only
6. Maintain a balance of your top strengths; overuse as well your seniors but also your colleagues and your juniors.
as underuse of particular strengths leads to depression and This will earn respect for you at your workplace.
stress
Special rules for officers in the corporate environs: Officers
7. Forgive, let go and move on instead of lingering with the
who will hold high post in the corporate sector have to
stressor
understand the different interpersonal communication styles
8. Take a pause and deliberate for a while on how best to tackle employed in the business environment.
a stressor rather than reacting on a reflex
Honesty is important in interpersonal relations. If you say COMMUNICATION
something and your body language conveys something else,
you give out conflicting messages which erode trust. It is COMMUNICATION FLOWS AND
important to value others’ time, space and comfort zone; wasting NETWORKS
their valuable time through your ineffective communication or
imposing yourself to the point of stifling them will only scuttle Communication flows in five directions.
the chances of the desired outcome. 1. Downward 2.Upward 3. Lateral
4. Diagonal 5. External
Communication channels accordingly form five kinds of networks.
Create the figures below because they have been copied and therefore
their labelling has to be changed. Text boxes will not change.
EBD_7711
22

A A
Superior

P X
Feedback Message

Q Y
B

R Z
Subordinate Circuit Network
Vertical Network Chain of Command Message from one to the
Two-way, between the From every superior to the other and feedback to the
superior and the subordinate subordinate, in a hierarchy chain former, in a circuit

Y
O
T

S U S P
X

W V R Q

Wheel of Network Star Channel


Each member of the group
From one superior to
communicates with another
all subordinates
TYPES OF INTERPERSONAL and that too if it is wielded by a respected and senior
authority. Otherwise, it only serves to alienate the workers.
COMMUNICATION IN THE OFFICE
(a) One Way Communication : In one-way communication, the (B) Structuring Style
manager or individual in charge of the workplace just gives Structuring style is used to impose schedules and discipline
a set of instructions to be followed by those who are working
workers. Rules are cited with the objective of achieving
under him.
goals. Here too, alienation is created leading to resistance
(b) Two way Communication : Two way communication is time and low productivity.
consuming but it is more accurate and very rewarding in
terms of higher productivity. In this case, both the parties, (C) Egalitarian Style
the worker and the person allotting work, spend time
discussing the work procedures and work out a mutually Egalitarian style is a two way communication process where
acceptable method. the other party is encouraged to come up with ideas and
opinions. It is a very effective method of communication as it
TYPES OF BUSINESS COMMUNICATION based on mutual understanding and respect.
There six different styles of communication used in business.
(D) Withdrawing Style
(A) Controlling Style Managers use the withdrawing style when they sit back and
Managers use controlling style to impose discipline and allow employees to do things their own ways. They exhibit a
get work done and to discourage employees from asking complete lack of interest and do not participate at all.
too many questions. This style is useful only during crises
23
(E) Dynamic Style TIPS FOR EFFECTIVE NON-VERBAL
In dynamic style, the manager is a motivating factor trying to COMMUNICATION
inspire his employees to deliver the desired results. This
method is only effective if the employees are highly 1. Modulate your speech and wear a positive expression for
knowledgeable. desired effect( paralanguage): The intonation, pitch and
speed of speaking communicate your attitude and interest.
(F) Relinquishing Style Hesitation sounds like ‘er..’ mar the effect.
If employees are themselves knowledgeable, skillful and 2. Regulate time of speaking and take desirable pauses
talented, relinquishing style is used. The manager is (chronemics): Racing with your words will evoke
receptive to suggestions of his employees and adds his own negative emotions in the listener.
ideas. The employees become team-mates of management. 3. Be mindful of your posture, movements and gestures
(kinesics): Leaning forward communicates receptiveness
BARRIERS/GAPS IN THE PROCESS OF and interest. Slouching hints at disinterestedness. A nod
BUSINESS COMMUNICATION or smile conveys encouragement. A frown, a straight look,
or rolling of your eyes communicates disapproval. Be
1. Complicated and obscure messages sensitive to how the other person will interpret your
2. Usage of jargon, slang, or unfamiliar terms motion. Inappropriate gestures send negative images.
3. Misinterpretation of words, body language 4. Adopt friendly attitude: Friendly, flexible, respectful and
open-minded attitude eases communication. Rigidity
4. Presumptions and prejudices destroys headway.
5. Personality conflicts 5. Establish eye contact: It shows you’re honest. Looking
6. Emotional baggage away hints at dishonesty in words.
7. Generation gap 6. Be focused: Avoid distractions. Lack of focus is noticeable
and puts off the other person.
8. Mismatch of psychological, social or cultural 7. Touch the arm and shake hands (haptics): A firm handshake
contexts inspires trust; a two-handed shake expresses heartfelt
9. Distractions like noise emotions. A limp or clumsy handshake instills doubt. A
high five establishes mutual approval. A touch on the arm
10. Lack of follow up and feedback
or hand conveys empathy and assurance.
Interaction between people does not happen in isolation; it is, 8. Allow comfortable space (proxemics): Be mindful of the
indeed, a conglomeration of various contexts or environment, other person’s comfort zone. Moving too close may hint
which are likely to give rise to conflict and stress in individuals at a desire for intimacy. Being too far may suggest
and affect the group outcome. distancing and lack of feeling.
1. Psychical context: the communicators’ personality and 9. Adorn suitable attire and embellishments: The style and
background that governs individual thought and behaviour colour of your attire convey your outlook to life. The smell
2. Relational context: the relation between the you wear conveys you attitude. Mild fragrance endears,
communicators strong can irritate and offend the other person. Make
3. Cultural context: the learned behaviours, norms and sensible choices to avoid conveying undesirable messages.
ethics that govern the interaction You may undertake periodic self-appraisal to assess and improve
4. Social context: the interrelation of social factors and your interpersonal skills, including the art of communication.
site of the interaction Record videos of your conversation with others and practice
5. Ethical contexts: the set of ethics may differ for different corrective steps to make your communication skills perfect.
parties
Management of conflict as well as stress is a prerequisite for TIPS TO BE A GOOD LISTENER
effective team work.
1. Avoid prejudice and presumption
TIPS FOR EFFECTIVE VERBAL 2. Keep emotions in check, avoid over reaction
COMMUNICATION 3. Wear a positive expression
4. Maintain good posture and lean forward
1. Avoid wrong usage; choose your words carefully 5. Be focussed
2. Avoid ambiguity; be clear 6. Establish eye contact
3. Avoid cryptic or pompous words; use simple language for 7. Nod in acknowledgement
easy comprehension 8. Gently touch the other person’s arm, or squeeze the hand,
4. Be coherent to reassure that you understand and Empathiner
5. Use courteous and respectful expressions 9. Make good queries to demonstrate your involvement
6. Use language that elicits a feedback 10. Recapitulate the other person’s words to assert that you
7. Allow others space and chance to voice their mind are listening closely
EBD_7711
24
Module – 8 : Administrative Course of Action Technical skill refers to specialized knowledge, proficiency,
analytical ability, and facility in the use of the tools and
techniques in a specific kind of activity. Development of technical
WHAT MAKES A GOOD skills is the purpose of the majority of our vocational and on-the-
job training programs.
ADMINISTRATOR
Conceptual skill is the ability to see the enterprise as a
An administrator is one who bears the responsibility for achieving unit;understand abstract relationships; develop concepts and
objectives and meeting targets by directing and monitoring the ideas; and solve problems creatively.It requires a deep
activities of their subordinates. The mark of a good administrator perception of how the various organs and functions of the
is not what they are, or their qualifications, characteristics, etc.; organization are interdependent and how changes in any one
it is, rather, what they do, i.e., their behavioural skills which are part affect all the others.It extends to understanding the
manifest in their course of action. relationship of the individual business to the industry and
the community; and to the political, social, and economic
SKILLS OF AN EFFECTIVE
forces of the nation as a whole. The administrator should
ADMINISTRATOR acknowledge these inter-relationships and take them into
Skill is not mere potential, whether inborn or acquired, rather an consideration to perceive the significant elements and
ability which can be developed and which is clearly manifested developments in any situation, so as to be able to act in the
in behavioural action. Psychologist Daniel Katz identified three interest of the organisation.
skills that are invariably present in every effective manager/
ADMINISTRATIVE COURSE OF
leader/administrator: human skills, technical skills, and
conceptual skills.These are all are interrelated and can be ACTION
developed to the desired levels. According to Katz, the higher It is imperative to observe the following to be able to arrive at the
one rises in an organisation, less of technical skills and more of best administrative course of action.
conceptual skills are required. However, human skills are
• Have a neutral and unbiased frame of mind to understand
indispensable at all levels.
the problem
Human skill is primarily concerned with working in tandem with • Get to the root of the problem, then take the logical step
other people. An effective administrator has the ability to work in
• Consider rationality as complete knowledge and
cooperation with the other members of the team he leads.He
anticipation of the consequences that will follow the
demonstrates this skill in the way he regards his superiors, equals,
course of action
and subordinates; in the way he acknowledges their perceptions;
and in the way he behaves subsequently. • Avoid taking extreme steps that may have undesired
outcomes
A person adept in human skills creates an atmosphere of approval
• Aim the course of action at solving, easing or minimising
and security by encouraging subordinates to participate in the
planning process and make them speak their mind without fear the problem
of censure or ridicule. He is sensitive to the beliefs, needs and • Claim 'solved' status against a problem only when the
motivations of others in his organization so that he can perceive course of action is an established or experienced fact, or
and anticipate the possible reactions and outcomes vis-a-vis the a logical step
courses of action he may undertake. This enables him to act in a
way which satisfies all parties.
25
Module – 9 : Decision Making attentive and smart listener listens to the question with utmost
attention, answers it later on. This gives him/her enough time to
recall the facts and figures to reply comprehensively. On the
DECISION MAKING—A MENTAL contrary, a poor listener never replies to a question properly.
Thus, it is clear that listening is an ability that everyone must
PROCESS develop. Try the following steps to enhance your listening ability.
Decision Making is a mental process undertaken by an individual 1. Pay attention to the speaker as long as he/she speaks. Make
for selection of a course of action among many available options. him/her feel that you are concentrating on him/her only
Decision making will result in a final choice. Decisions are always and he/she is getting due attention. Try to show that you
taken in the context of a set of needs. understand what he/she is trying to convey.
In this type of questions, a given situation is followed by four 2. Try to make eye contact and observe the body language of
options of the decision to be made. the speaker. It will give you a chance to understand his/her
These questions are based on real-life situations that IAS officers posture as people often give out aggression or calmness
encounter routinely. They gauge your ability to deal with similar through their body language.
situations and arrive at the best decisions. Thus, these questions
3. Make the speaker feel that you are listening to him/ her.
are designed to judge your analytical skills and your ability to
resolve a problem. You can use your body language and small verbal
communication such as 'yeah', or nod your head positively.
The questions may deal with different situations like land
acquisition for a proposed economic zone; hurdles in setting up a 4. If you find anything difficult or miss any important point,
power generation unit, etc. You have to decide on the best possible ask the speaker to repeat that only after he/she completes
option in the given situation. There is no fixed rule for arriving at his/her point.
a decision. Your answer depends on your analytical ability and 5. Give feedback but only when a speaker—who may be a
also on your interpersonal skills. professor, a complainant, a student or anyone else—asks
The idea underlying this test is to evaluate your ability to take after he/she finishes his/her point. Every speaker solicits
decisions logically for exigencies, contingencies and emergencies some feedback after he/she finishes his/her lecture.
with appropriate management of people. Therefore, real life situations 6. Remember, your feedback should be balanced as it will
have been framed as questions, to test your skills in dealing with define your image in the eyes of the speaker. Therefore,
the hypothetical crisis presented to you in the form to a query. do not exaggerate, or understate your opinion. Try to
Take care while solving such questions. Never make assumptions summarize it precisely.
of your own for the examiner wants you to solve the given problem
7. Try to guess where the discourse is leading.
within the ambit of its original premises.
Remember, analytical ability is the best tool to deal with such If you are attentive, well focused, and you pay attention to
situations. You always use your personal abilities to analyse any minute details, you will gradually develop the ability of a
crisis. The examiner wants to know your solution to the problems good listener. It will help you in making decisions in your
that as a civil servant you may have to deal with on a daily basis. career, in your office and in your daily life.
Decision Making is difficult due to multiple alternatives available There are three aspects of listening.
and, risky consequence involved in a particular decision. There is 1. Receiving information
uncertainty, as well as complexity, and a need to evaluate issues 2. Solving problem
involved in any decision. A major challenge during decision making
is to settle interpersonal issues. 3. Sharing with others
When you receive information, you must pay utmost
STEP TECHNIQUE FOR DECISION attention so that you collect right information. Civil
MAKING servants often have to deal with tricky situations while
1. Identification of objectives. probing an important case. Suppose that you are an IPS
2. Classification of objectives. officer and receive the information that a station house
officer has indulged in corruption. As a district police
3. Analysis of alternative actions.
4. Evaluation of alternatives. chief, it is your responsibility to ensure action against
5. Decision evaluated with respect to consequences. the officer to solve the problem. But before taking any
6. Hindrances in implementing the decision. decision, you would like to investigate the truth behind
the complaint. You will share it with some of your juniors
EFFECTIVE LISTENING FOR DECISION and people in the area where the SHO is posted. So, your
MAKING listening ability will come to your great help in your
decision making.
Listening helps you not only in problem solving, but also in image
building in front of the interview board or the community. An
EBD_7711
26
Module – 10 : General Comprehension The following techniques will help you in answering the
questions based on the given passages.

HOW TO ATTEMPT READING IMPORTANT TECHNIQUES


COMPREHENSION
(A)USE YOUR PENCIL AS A POINTER:
Reading comprehension exercise judges your ability to
understand the passage; to analyse the problem in a proper To begin with, use your pencil as a pointer. Using the
perspective; as well as, to answer quickly, correctly and pencil to guide your eyes along a line of text helps you
systematically. Comprehension exercise measures your ability to focus on the details given in the passage. It holds your
to read and understand the written passage. attention to the precise words in the passage. In a long
The entire process of Reading Comprehension can be divided test, attention may weaken. Fatigue may blunt your
into 7 simple steps that are shown in the flow chart below: attention to details. But using your pencil as a pointer will
help you preserve your attention to details.

(B)READ FASTER ALONG YOUR PENCIL:


Another benefit of using the pencil as a pointer is that it
will probably speed up your reading. The steady flow of
the pencil across the page with each line of text draws the
eyes along at a steady pace. Do not go faster than you can
grasp the text, but do try to keep your reading going at
a steady pace set by the pencil.

(C) CIRCLE KEY WORDS AND PHRASES:


Remember, you are not reading just for a vague general
understanding of the passage. You have to read for detailed
understanding. Circle key words or phrases which will enable
you to zero in on precise points needed to answer a question.

(D) KEEP FORGING AHEAD:


Do not get bogged down if there is a word or sentence you
Though reading comprehension is what our brain practises all do not understand. You may get the main idea without
the time, we do not always perform very well when attempting a knowing the individual word or sentence. Sometimes, you can
reading comprehension question; why ? sense the meaning of the word from the context. Sometimes,
the word or sentence may not be the basis of any question.
Because what the brain does is at an ordinary simplistic level
If there is some idea you need to answer a question but do
and we are unaware of even that. But what is required of an aspiring
not understand, read it one more time. If you still do not
students is a conscious, skilful, determined effort to master the
understand it, move on. You can come back to this question
art of reading comprehension.
later if you have time left at the end of the test.
To answer the questions based on reading the passage, it is
important that they should be drawn from the information given
in the passage. Your common sense, knowledge and presumption (E) FORMATION OF AN IDEA FROM THE
should not be taken into account while answering the questions. QUESTIONS:
If you have some related information from your own experience Another good reading comprehension strategy is to read
and knowledge, you should not use it to answer the question. the questions before starting the passage. This does not
Even if you think that there is some mistake, you must still answer mean to read the answer choices as well. By reading the
the question on the basis of the information given in the passage. questions, you will have an idea of what information you
27
will need after reading the passage. This may alert you to For example, are they contr asting? Are they
certain details, ideas and specific areas in the paragraph complementary? Consider the points made by the author.
where the questions are drawn from. He conclusions drawn, and how and why those points are
made or conclusions drawn.
RIGHT APPROACH 4. Read each question carefully and be certain that you answer
1. Questions are to be answered on the basis of the exactly what is being asked.
information provided in the passage and you are not 5. Always read all the answer choices before selecting the
expected to rely on outside knowledge of a particular topic. best answer.
Your own views or opinions may sometimes conflict with 6. The best answer is the one that most accurately and
the views expressed or the information provided in the most completely answers the questions in accordance
passage. Be sure that you work within the context of the to the passage. Be careful not to pick an answer choice
passage. You should not expect to agree with everything simply because it is a true statement. Be careful not to
you encounter in reading passages. get misled by answer choices that are only partially true
2. You should analyse each passage carefully before or only partially satisfy the problem posed in the
answering the accompanying questions. As with any kind question.
of close and thoughtful reading, look for clues that will help 7. The methodology of eliminating wrong answers also
you understand less explicit aspects of the passage. Try works here. It simply means that if you are unable to judge
to separate main ideas from supporting ideas or evidences. the right choice or right answer, cross out the answers
3. Note transitions from one idea to the next, and examine the that are incorrect, so that the remaining one is the right
relationships among the different ideas or parts of the passage. choice.
EBD_7711
28

Module – 11 : English Language Comprehension Skills maintain a desire to know more, keep yourself
curious about every subject. Do not hesitate in
discussing your opinions, asking questions,
INTRODUCTION expressing your views with friends, teachers or
The success mantra in today’s highly competitive world with ever experts. This kind of interaction and
expanding boundaries of knowledge, is “know the right thing at the communication will greatly increase your interest
right time, make right use of it and express it in just the right words”. and knowledge and you will be drawn towards
Why do you think reading comprehension questions are asked reading more. Always keep in mind that developing
from the primary level in school examinations to a level as high these habits will help you reap long term benefits.
as competitive examinations for management or administrative
work? Very simply put, in the present era, which has a plethora of 1. Practise solving more and more Reading
information, facts, knowledge, it is important for any officer, or Comprehension exercises. Start with shorter
manager, to be able to extract relevant information from the given and simpler pieces on subjects of your
draft in minimum possible time and use it for the execution of the interest and gradually move to heavier and
assigned project in the best possible way. And this is what more difficult passages. This will strengthen
your confidence, as you build up your
language comprehension exercises give you a practice in.
momentum, and prepare you better to deal
Therefore, a student must consider this section not only as a
with longer passages.
preparatory exercise but also, as an exercise to build a skill that 2. Self-monitor your reading. Mark every time
he/she will use for the rest of his/her life. you get distracted or lose concentration. In this
manner, you will be aware of the number of
Make reading a habit to improve your expression
times you lost concentration in an hour; and
and vocabulary. Each day, list out the new words
you come across. Classify them under subjects, with every passing hour, you will make a
Science, Sports, Politics, Literature. Learn their stronger effort to be more focussed. You will
meanings and use them frequently. This will help see that with each passing hour, the number of
you understand the jargon of different fields. the marks will decrease. And, you will
overcome the tendency to get distracted.

MUST Dos FOR A GOOD READER READING SPEED


r Read about different fields, don’t restrict yourself to one. From a competitive point of view, ‘Time is money’. The
r Do intelligent reading, don’t waste time reading junk faster you read, the more time you will have for
information. comprehension and analysis. But you must keep in mind
r Read only from good newspapers and magazines certain points while trying to read fast. Students, in their
attempt to read fast, often increase their speed of reading
r Utilise maximum time, read while travelling, waiting, etc.
without trying to understand or retain the information.
r Try to indulge in a discussion everyday about what you read Remember,
that day.
Reading Comprehension = Reading + Comprehension.
r Improve your passive vocabulary as you read and use the
You must read at a fast speed but not at the cost of
learnt words to build your active vocabulary.
comprehension or understanding because in that case,
you’ll have to read the passage again and, the time you
For maximum utility of time, you can depend on saved by reading fast will be consumed in re-reading. So, it
News Channels. You can select certain talk shows will be a wasteful exercise.
aired on some good English news channels like
1. Your reading speed is the number of words
NDTV, CNN IBN or TIMES NOW and watch them
you can read as well as understand per
regularly. This will improve your general
minute. Remember, if you don’t understand
awareness, give you an analytical perspective, what you’re reading, it is of no use.
keep you updated with news from different 2. Track your reading speed. In this manner, you
sectors, and also improve your English. will be able to monitor your improvement.
An inquisitive mind is a gift for a student. Always
29
RETAINING CAPACITY Keeping these points in mind will improve your reading and
retaining efficiency greatly. What you have to target and
This is the most important part of Reading. If a student is achieve is EFFECTIVE READING. A good reader may or
able to read well, read with concentration and read fast, may not be just as good at comprehension and analysis but
but is not able to retain useful information of the data then an Effective Reader would definitely perform in
all is lost. A good reader need not come back to the passage comprehension and Analysis of data just as well. So, try
again and again to look for answers. The first reading of and be an EFFECTIVE READER.
the passage should, therefore, be done with much care and
attention so that most of the matter is retained. HOW TO APPROACH DIFFERENT
You can follow some simple steps to improve your retaining TYPES OF COMPOSITION
capability. Every time you read a passage, make a mental
Composition may be done in different styles as the
note of the following :
following diagram shows.
(a) title of the passage
(b) basic theme of the passage (1) Narrative.
(c) the positions that the passage takes or the points that Expository Analytical
the passage makes (2) Reflective
(d) conclusion of the passage.
Argumentative Philosophical
READING APTITUDE (3) Imaginative.
Reading Aptitude is different from reading skills. The points Let us now consider the different types of writing one may
and factors discussed so far, constitute reading skill and get to read and how one should deal with each.
pertain to the manner in which you read. Reading Aptitude 1. Narrative Passage
is the natural ability to read and understand well. It gives an
upper edge to a student in the RC section or even otherwise, A narrative passage usually tells a story which means a
if developed properly. sequence of events. Thus, a narrative passage gives an
orderly account of a series of related events or the
successive particulars of an event. A narrative passage
WHAT IS READING APTITUDE? could be of various kinds: (1) Biography (2) History
Reading aptitude includes the approach that you take while (3) Fiction (4) Execution of a process.
reading; the mindset with which you read and the 2. Reflective Passage
expectation you have from the piece. If you approach a
reading piece only for the purpose of collecting facts to add (a) Expository Passage
to your existing store of knowledge; or, only as a practice to An expository passage is the most rigid and restricted
improve your reading speed; or, merely as an examination form of composition. It is also the most common kind of
exercise; you would not receive the same results as you writing. Its purpose is to explain, its language is clear and
would if you read the passage with a different attitude. direct, mostly. Its appeal is to the intellect. What you would
mostly come across in an expository essay will be
HOW TO DEVELOP READING definition, comparisons and contradictions. Expository
APTITUDE passages can be either analytical or philosophical in
approach. While the former is investigative, the latter,
Consider every piece of written information as a prospective scholarly and sermonising.
useful draft.
Begin with the rule of WIIFM - what’s in it for me. Once you (b) Argumentative Passage
have used your wisdom to decide if the passage is worth An argumentative passage includes an argument and an
reading, approach the passage as a mystery novel. There are argument is possible only about a subject that invites
hidden clues you must look for. From the beginning, stay a argument on conflicting opinions. Such an essay admits
careful, clever reader. Remember, the first reading itself should difference of opinions and, therefore, the purpose of an
give you all you may need to have from a passage. argumentative essay is to persuade the readers to adopt a
If there are facts in the passage, quickly decide, as you read, certain idea, attitude or course of action and, if possible, to
which of those are important enough to be memorized; and resolve the conflict implicit in the subject.
readily memorize them. 3. Imaginative passage
If there is an argument in the passage, keep a trail of how the
Imaginative passages are those that talk about something
argument proceeds; involve your mind with the passage
which does not exist in reality. These passages are fictional
and form an opinion about the argument.
in nature; e.g., on conversation between two animals/trees,
If the passage is offering a philosophical perspective, got
demons etc., or fantasy; e.g. imaginative creatures and
down a short summaryof the philosophical theory in simple words.
situations.
If the essay describes a process or an event, then as you
read on, form a chain of events in your mind.
EBD_7711
30

Quiz – 2 on Comprehension, Decision Making &


Interpersonal Skills
Directions (Qs. 1-4) : Choose the correct response for each of (d) Advise him to directly apply to the allotting authority
the given situations. and inform him that you do not interfere in this matter.
1. If you hold an opinion that is radically different from that Directions (Qs. 5-6) : In each question below is given a
expressed by a lecturer, you would statement followed by two courses of action numbered I and II.
(a) argue right on the spot. You have to assume everything in the statement to be true, then
(b) tell the person about it during the lecture. decide which of the two given/suggested courses of action
(c) tell the person about it after the lecture. logically follows. Mark answer as :
(d) forget about it. (a) If only II follows
(b) If only I follows
2. You are working on a project with a deadline. You have
(c) If neither I nor II follows
delegated specific work to every team members. You are
(d) If both I and II follow
informed about the sudden demise of the father of a team 5. Statement :
member. You have been entrusted with the responsibility of Due to the heavy demand of milk during Diwali, there is a
breaking the news to him. What will you do? rumour that sweets hop owners are using adulterated milk
or synthetic milk to make sweets.
(a) Inform him at once and permit him to leave the important
Courses of action :
project. I. The district administration should direct the police to
(b) Inform him immediately but take his inputs about his raid all the sweet shops and check the purity of the milk
work, and assure him that you will manage and take care used to make sweets.
II. The district administration should issue a notice in the
of the rest. Then, you relieve him. newspaper starting that if any adulteration is found in
(c) Decide not to tell him as he will leave on receiving this any product made by the shops, the shops will be heavily
news which would be detrimental to your project. penalised along with cancellation of their licences.
6. Statement :
(d) Call off the project. There has been a shortage of water in the state for the last
3. You have been assigned a target but you have not been given four days as the neighbouring state has stopped the supply
appropriate resources. There is lack of finances, manpower of water due to some inter-state problems.
and government cooperation. What do you do in such a Courses of action :
situation? I. The Central Government should immediately intervene
to discuss the matter with these two states and find
(a) You will still try to achieve the target.
the solution.
(b) You will use the limited resources in an efficient manner. II. The CMs of both the states should take this matter
(c) You will request for more resources to achieve the target. seriously and discuss the matter in order to find a
(d) Use the limited resources and then demand resource at resolution to the issue of water crisis; and try to maintain
a later stage. a good relation with each other.
Directions (Qs. 7-9) : Read the given passages and answer the
4. There is a shortage of sugar in your district of which you are questions based on them.
the District Magistrate. The Government has ordered that
only a maximum amount of 30 kg sugar is to be released for PASSAGE-1
wedding celebrations. Your friend requests you to release
India's National Biodiversity Action Plan (NBAP) recognises the
at least 50 kg sugar for his son’s wedding. He expresses importance of biodiversity for inclusive development. The Green
annoyance when you tell him about the Government’s Agriculture project implemented by the Indian government and
restrictions on this matter. He feels that since you are the the Food and Agricultural Organisation (FAO) takes a novel
District Magistrate you can release any amount. You do not approach to support the NBAP and synergise biodiversity
want to spoil your friendship with him. In such conservation, agriculture production and development. It is being
implemented in five landscapes adjoining Protected Areas/
circumstances, how would you deal with the situation?
Biosphere Reserves. It envisages a transformation in Indian
(a) Release the extra amount of sugar which your friend has agriculture for global environmental benefits by addressing land
requested for. degradation, climate change mitigation, sustainable forest
(b) Refuse your friend the extra amount and strictly follow management, and biodiversity conservation.
the rules. 7. What is the most rational message that is implied from the
above passage?
(c) Show your friend the copy of the Government (a) India is facing stiff competition in biodiversity
instructions and then persuade him to accept the lower conservation at the global level so, it has taken an
amount as prescribed in the rules. initiative with various organisations to stay ahead.
(b) Forecasting a transformation in its agricultural sector
31
for global environmental benefits, India has started
addressing various issues related to it. PASSAGE – 4
(c) For the betterment of India's agricultural products, India With the inevitable growth of specialization I see the universities
needs to have an action plan that checks the flaws in facing two great dangers. First, it is very easy to get so involved
its agricultural developmen t an d biodiversity in the technical details of education that the object of education
conservation initiatives. is lost. And secondly, in an effort to condition a university to the
(d) Considering the significance of biodiversity in inclusive needs of its students and to the needs of the state it may lose its
growth, government of India in collaboration with FAO power to make or mould those students into responsible men,
is determined to make its biodiversity plan a success. capable of thinking for themselves and capable of expressing the
results of their thoughts to others.
PASSAGE-2 10. The author calls growth of specialisation ‘inevitable’. Which
one of the following statements is likely to be the most correct
The e-commerce industry been directly impacting the micro, small reason for this inevitability ?
& medium enterprises (MSME) in India by providing means of (a) Universities give grants only to do specialised work in
financing, technology and training and has a favourable cascading different disciplines
effect on other industries as well. The Indian e-commerce industry (b) The professors and researchers in universities are
has been on an upward growth trajectory and is expected to competent only for specialised work
surpass the US to become the second largest e-commerce market (c) Specialization helps economic growth of the nation.
in the world by 2034. Technology enabled innovations like digital (d) In an age of science and technology specialization
payments, hyper-local logistics, analytics driven customer becomes necessary
engagement and digital advertisements will likely support the 11. Which one of the following statements most correctly
growth in the sector. The growth in e-commerce sector will also suggests the central theme of the passage ?
boost employment, increase revenues from export, increase tax (a) The aim of education is specialization
collection by ex-chequers, and provide better products and services (b) The aim of education is to mould the youth to work for
to customers in the long-term. the state
(c) The aim of education is to make the youth capable of
8. What are the most crucial messages implied from the above independent thought and expression
passage? (d) The aim of education is to enable the youth to lead a
(I) E-commerce industry in India has jolted the MSME thus, comfortable living
its repercussions can't be neglected. 12. Which one of the following statements most correctly
(II) Indian e-commerce industry is highly supported by suggests the warning implied in the passage ?
technological advancements and innovations that have (a) University education should not be concerned with
technical details.
led to its growth in recent times. (b) Universities should not subordinate themselves to the
(III) The growth in the e-commerce industry looks forward interests of the state.
to inducing more jobs, increasing revenue and (c) Universities should be concerned only with the needs
becoming customer supportive in long run. of students.
(a) Both (I) and (III) (b) Both (I) and (II) (d) Universities should not go in for any specialization.
(c) Both (II) and (III) (d) All (I), (II) and (III) PASSAGE-5
PASSAGE-3 The unpleasant feeling passed and she glanced guardedly up at
him. He was walking unmarked in moonlight, innocent of her
The term "mental health" has become a euphemism for "mental reaction to him. She felt then – this thought had come to her
illness". This is in stark contrast to what it should be, for mental before – that there might be more to him than she had imagined.
health is the most valued asset we have, the most treasured aspect She felt ashamed she had never thanked him for the help he had
of our humanity. This is precisely why, when people are asked to given her father.
compare different health conditions, mental health problems are 13. She glanced at him when
ranked as the ones they fear the most. This is not surprising, for (a) He walked alone and unnoticed in moonlight
we rely on the diverse capabilities our mental health underpins to (b) She was sure that she was not being noticed
successfully learn and master the skills that make our lives (c) Her reactions did not have any effect on him
meaningful and worthwhile. So, one would expect that mental health (d) The unpleasant feeling passed
would be the most prioritised of all issues facing the world. Sadly, 14. Her unpleasant feeling passed when
that is quite far from reality. (a) He did not take any notice of her
9. With reference to the above passage, which of the following (b) The moonlight was beautiful
assumptions have been made? (c) She realized her sense of shame
(a) Mental health is everything so, it must be taken utmost (d) She looked carefully at him
care of. 15. She was ashamed because
(b) Mental illness ranks first when we talk about human (a) She was spying on him
sufferings. (b) There was more to him than she had imagined
(c) One should always strive to have a good mental health. (c) A recurring thought came back to her
(d) Mental sickness often results to poor human conduct (d) She had never thanked him for his help to her father
and behaviour.
EBD_7711
32

Hints and Solutions


1. (c) own. The Centre intervenes only when the two states
2. (b) Relieving the particular team member is necessary are not able to sort out the problems on their own.
given his tragic situation. The best course of action, 7. (d) This can clearly be inferred from the first two lines of
therefore, is to take report and inputs of his work and the passage.
manage with the rest of the available resources.
8. (c) These are clearly mentioned in the passage. The author
3. (b) This is the best approach as resources may not be is appraising the growth of e-commerce industry in India
given when they are demanded. and expressing his positive insights about the effect of
4. (d) This will make it very clear to him that doing favours the growth in coming times.
is not part of your job. You hold a responsible 9. (a) This is clearly evident from the second, third and second
position. The advise will show that you care for him last sentence of the passage.
and a direct approach to the allotting authority might 10. (d) In an age of science and technology, specialization
work for him. This gives a lasting impression that you becomes necessary.
follow the rules. 11. (c) The central theme of the passage is that the aim of
5. (a) II follows as issuing a warning letter will definitely help education is to make the youth capable of independent
in stopping the use of synthetic on adulterated milk (if thought and expression.
any). I doesn’t follow as it is not feasible for the police 12. (a) The major warning suggested in the passage is that
to raid each and every shop and check the purity of the University education should not be concerned with
milk used to make sweets. This step is taken later, only technical details rather it should embrace humanism.
on reports of violators despite the warning. 13. (d) She glanced at him when the unpleasant feeling passed.
6. (a) II follows as the meeting between the heads of the 14. (c) The unpleasant feeling passed when she realized her
sense of shame.
concerned states will definitely help in working out a
15. (d) She was ashamed because she had never thanked him
solution. I doesn’t follow because the Centre never for his help to her father. This thought had occurred to
intervenes straightaway in case of inter-state disputes; her before also, that's why she felt ashamed.
it waits for the two states to resolve the issue on their
Aptitude Test
Mental Ability, Logical Reasoning & Problem Solving
Module – 12 : Blood Relation top while children are put at the bottom.
‘–’ or minus sign is used for female
‘+’ or plus sign is used for male.
TYPES OF BLOOD RELATIONS
Illustration
There are mainly two types of blood relatives:
(i) Blood relation from paternal side Q is the brother of C and C is the sister of Q. R and D are brother
(ii) Blood relation from maternal side and sister. R is the son of A while A & C are wife and husband.
How is Q related with D.
BLOOD RELATION FROM PATERNAL Sol. Using the symbols, we can make a family tree and solve the
SIDE given problem. Let us see the family tree:
Family tree :
THIS TYPE OF BLOOD RELATION CAN BE FURTHER
– +
SUBDIVIDED INTO THREE TYPES: A+ C Q

(a) Past Generations of Father


Great grandfather, great grandmother, grandfather, grandmother etc.
(b) Parallel Generations of Father
Uncles (Brothers of father), Aunts (sisters of father) etc. R+ D–

(c) Future Generations of Father As per the question Q is the brother of C and C is the sister of Q.
Hence, relation between C & Q has been presented as
Sons, daughters, grandsons, granddaughters etc.
(C –
)
— Q + where ‘–’ sign above C makes it clear that C is a
BLOOD RELATION FROM MATERNAL female and ‘+’ sign above ‘Q’ makes it clear that Q is a male.
SIDE
æ+ -ö
THIS TYPE OF BLOOD RELATIONSCAN ALSO BE SUBDIVIDED Similarly, for R and D. The presentation ç R — D ÷ has been
è ø
INTO THREE TYPES:
made. Further according to the question,
(a) Past Generations of Mother A and C are having a husband and wife relationship and hence
Maternal great grandfather, maternal great grandmother,
æ+ ö
maternal grandfather, maternal grandmother etc. this has been presented as ç A « C - ÷ . As it is already given
è ø
(b) Parallel Generations of Mother that C is the sister of Q and A and C are wife and husband, this
Maternal uncles, maternal aunts etc. becomes clear that A is the male member of the family and this is
the reason A has ‘+’ as its gender sign. Lastly, the vertical line
(c) Future Generations of Mother
gives father and son relationship and has been presented as
Sons, daughters, grandsons, granddaughters etc.
æ A+ ö
ç| ÷
BLOOD RELATION BASED ON FAMILY çè R + ÷ø . Now from this family tree it becomes clear that C is the
TREE mother of R and D and as Q is the brother of C, then Q will
definitely be the maternal uncle of R & D. Hence, we can say that
Some symbols are used to draw family tree as below: Q is the maternal uncle of D and this is the required answer for our
‘«’ is used for husband & wife. question.
‘___’ is used for brother & sister
‘ | ’ is used for parents (father or mother). Parents are put on Note : In solving family tree based relations, make sure that
your diagram is in correct representation.
EBD_7711
34
Module – 13 : Direction & Distance CONCEPT OF TURN
Right turn = Clockwise turn
CONCEPT OF DIRECTION Left turn = Anticlockwise turn
In our day to day life, we make our concept of direction after Let us understand it through pictorial representation:
seeing the position of sun. In fact, this is a truth that sun rises in Right turn Left turn
the East and goes down in the West. Thus when we stand facing

Right turn

Left turn
Right turn
sunrise, then our front is called East facing while our back is called

Left turn
West facing. At this position our left hand is in the Northward and
the right hand is in the Southward. Let us see the following direction
map that will make your concept more clear. Right turn Left turn
(i) (ii)

DIRECTION MAP
North
Right turn Left turn
North-West North-East
(N-W) (N-E)

(iii) (iv)
West East
Illustration – 1
Panas walked 2 km West from his office and then turned South
South-West South-East covering 1 km. Finally, he walked 3 km towards East and again
(S-W) South (S-E) moves 1 km West. How far is Raman from his initial position.

q Shortcut Approach Sol. Panas starts from his office A, moves 2 km West upto B, then
To remember four main directions, always remember the 4 km to the South upto C, 3 km East upto D and finally 1 km West
word 'NEWS.' upto E, Thus his distance from the initial position AE = BC = 1 km.
B 2 km A
Note: On paper North is always on top while South is always in bottom.
1 km
CONCEPT OF DEGREE
1 km D
Let us see the following picture:
C 2 km E
360º 360º
0º 0º Remember
315º
Anti clockwise (ACW)

45º 45º 315º • If our face is towards North, then after left turn our face will be
Clockwise (CW)

towards West while after right turn, it will be towards East.


270º 90º 90º
• If our face is towards South, then after left turn our face will be
270º
towards East and after right turn it will be towards West.
• If our face is towards East, then after left turn our face will be
225º 135º 135º 225º
towards North and after right turn it will be towards South.
180º
• If our face is towards West, then after left turn our face will be
180º
towards South and after right turn it will be towards North.
• If our face is towards North-West, then after left turn our face
Remember will be towards South-West and after right turn it will be towards
North-East.
• North, East, South and West are called main directions. • If our face is towards South-West, then after left turn our face
• North-east, South-east, South-west and North-west are will be towards South-East and after right turn it will be towards
called subdirections. North-West.
• Angle between two consecutive main directions is always 90°. • If our face is towards South-East, then after left turn our face
• Angle between two consecutive subdirections is always 90°. will be towards North-East and after right turn it will be towards
South-West.
• Angle between a main direction and its adjacent subdirection
• If our face is towards North-East, then after left turn our face
is always 45°.
will be towards North-West and after right-turn it will be
towards South-East.
35
q Shortcut Approach = 16 + 9 = 25 = 5km.
Direction before Direction in which the person or
From figure, D is to the North-East of A.
taking the turn vehicle will be moving after taking the
turn
Right Left
SHADOW CASE
(i) North East West
(ii) South West East IN MORNING / SUNRISE TIME
(iii) East South North (a) If a person is facing towards Sun, the shadow will be towards
his back or in West.
(iv) West North South
(b) If a person is facing towards South, the shadow will be
(v) North-West North-East South-West
towards his right.
(vi) South-West North-West South-East (c) If a person is facing towards West, the shadow will be
(vii) South-East South-West North-East towards his front.
(viii) North-East South-East North-West (d) If a person is facing towards North, the shadow will be
towards his left.
CONCEPT OF SHORTEST/MINIMUM
DISTANCE IN EVENING / SUNSET TIME
(a) If a person is facing towards Sun, the shadow will be towards
The shortest distance between two points may be different from his back or in East.
the total distance covered in going from initial position to final (b) If a person is facing towards North, the shadow will be
position. towards his right.
To find the minimum distance between initial and last point, many (c) If a person is facing towards East, the shadow will be towards
times we have to use ‘Pythogoras Theorem’. A his front.
(d) If a person is facing towards South, the shadow will be
h2 = b2 + P2 towards his left.
where,
h P Note : At 12:00 noon there is no shadow because the rays of the
h = Hypotenuse
b = Base sun are vertically downward.
P = Perpendicular
Illustration – 3
Illustration – 2 B b C Early morning after sunrise, Rajesh was standing infront of his
house in such a way that his shadow is falling exactly behind him.
Rashmi walks 10 km towards North. She walks 6 km towards He starts walking straight and walks 5 m. He turns to his left
South then from here she moves 3 km towards East. How far and and walks 3 m and again turning to his left walks 2m. Now in
in which direction is she with reference to her starting point? which direction is he from his starting point?
Sol. It is clear, Rashmi moves 10 km from A towards North upto Sol. The shadow of Rajesh was falling exactly behind him. So,
B, then moves 6 km Southwards upto C, then turns towards East he was facing towards East. Diagram clearly shows that Rajesh
and walks 3 km upto D. was in North-East with reference to the starting point.
Then, AC = (AB – BC) = 10 – 6 = 4 km 2m
CD = 3km.
int

B
po

6 km
g

3m
tin

3 km
r
Sta

D
C

5m
10 km
q Shortcut Approach
• Draw four lines from a point (let O) repersenting four main
directions.
A • Think that you are at the point O.
• Read the statement line by line.
\ Rashmi’s distance from starting point A • Move yourself as per statement asked and prepare a diagram
as per line by line statement.
= AD = AC 2 +CD 2 = 4 2 + 32 • Show, check and verify the direction and distance moved
from starting point.
• Find the direction and distance of the final position.
EBD_7711
36
Module – 14 : Cube and Dice Top layer

CUBE
A cube is three dimensional object whose length, breadth and
height are equal and any two adjacent faces are inclined to each Bottom unpainted
other at 90°. It has 6 faces, 8 corners and 12 edges.
Layer II or middle layer :
G H The central cube has no face coloured, the four cubes at the
corner have two faces coloured and the remaining 4 cubes
C have only one face coloured.
D
Middle layer Top unpainted

E
F
Unpainted cube
A B
• Corners of the cube are A, B, C, D, E, F, G and H. Bottom unpainted
• Edges of the cube are AB, BE, EF, AF, AD, CD, BC,
EH, CH, GH, DG and FG. Layer III or bottom layer :
Faces of the cube are ABCD, EFGH, CDGH, BCHE, ABEF The central cube has only one face coloured, four cubes at
and ADFG. the corner have three faces coloured and the remaining 4
cubes have two faces coloured.
WHEN A CUBE IS PAINTED ON ALL OF Bottom layer
ITS FACES WITH ANY COLOUR AND
FURTHER DIVIDED INTO VARIOUS
SMALLER CUBES OF EQUAL SIZE, WE
GET FOLLOWING RESULTS :
Bottom painted
(i) Smaller cubes with no face painted will present inside faces
of the undivided cube. Also, number of divisions on the faces of cube,
(ii) Smaller cubes with one face painted will present on the faces Length of the edge of undivided cube
n = .
(except edges) of the undivided cube. Length of the edge of one smaller cube
(iii) Smaller cubes with two faces painted will present on the
edges (except corner) of undivided cube. q Shortcut Approach
(iv) Smaller cubes with three faces painted will present on the
• Total number of smaller cubes = (n)3
corners of the undivided cube.
• Number of smaller cubes with no face painted = (n – 2)3
• Number of smaller cubes with one face painted = (n – 2)3 × 6
• Number of smaller cubes with two faces painted = (n – 2) × 12
Cube with Cube with • Number of smaller cubes with three faces painted = 8
two sides three sides Illustration – 1
painted painted
A cube is painted blue on all faces is cut into 125 cubes of equal
Cube with size. Now, answer the following question :
one side How many cubes are not painted on any face?
painted Sol. Since, there are 125 smaller cubes of equal size, therefore,
n = number of divisions on the face of undivided cube = 5.
The above figure may be analysed by dividing it into three Number of cubes with no face painted = (n – 2)3
horizontal layers : = (5 – 2)3 = 27
Layer I or top layer : DICE
The central cube has only one face coloured, four cubes at
the corner have three faces coloured and the remaining 4 A dice is three-dimensional object with 6 surfaces. It may be
cubes have two faces coloured. in the form of a cube or a cuboid.
37
1 Can be opposite to 4, 5 or 6
2 Can be opposite to 4, 5 or 6
3 Can be opposite to 4, 5 or 6
4 Can be opposite to 1, 2 or 3
5 Can be opposite to 1, 2 or 3
6 Can be opposite to 1, 2 or 3
Case II: When one digit is common in both position and at the
TYPES OF DICE
same face as follows.

2 2
ORDINARY STANDARD 3 5 4 6
DICE DICE
(ii)
In this case, except the common digit, the digits on the other
1. ORDINARY DICE: faces are opposite to each other and the face opposite to
the common digit will have that digit which is invisible.
In this type of dice, the sum of numbers on opposite faces is Hence,
not 7 but the sum of numbers on two adjacent sides are 2 and 1 are opposite
seven.
3 and 4 are opposite
5 and 6 are opposite
3
Case III: When one digit is common in both positions but not at
5
4 same face as follows

2 4
1 4 6 3
Ordinary Dice
(ii)
4+3 = 7
In this case list the numbers on both the dices in either
2. STANDARD DICE: clockwise or anti-clockwise starting from common digit, on
comparing the numbers obtained from both dices will give
In such type of dice, the sum of numbers on opposite faces you the digit on opposite faces on two position.
is 7 or sum of numbers on adjacent faces is not 7.
In the above figures, number 4 is common in both positions,
now writing the digits in both position in clock-wise starting
Here,
3 1 1+4 =5 from 4, we get
5 4 4+5=9 4 1 2 [Position (i)]
4 5 1+5=6
4 3 6 [Position (ii)]
Thus 1 is opposite to 3
Ordinary Dice Opposite of 1 ........6 (since 1+6 =7) 2 is opposite to 6
Opposite of 5 ........2 (since 5+2 =7)
and 4 is opposite to 5
Opposite of 3 ........4 (since 3+4 =7)
Note that in this case digit opposite to common digit is the digit
which are not seen (invisible) in the two positions.
IMPORTANT RULES Case IV: When two digits are common in both the positions as
follows.
When Two Positions of a Single Dice are Given
Case-I: Digits are different in both position as follow. 1 4
4 2 2 6
1
3 (ii)
2
In this case,
(ii)
(i) There is a probability of coming digits on the faces opposite
In such case any digit in position (i) can be opposite to any of the to the faces having common digits are invisible.
three digits in position (ii) and vice-versa as given below: (ii) Uncommon digits in each dice are opposite to each other.
EBD_7711
38
Hence Illustration – 3
3 is opposite to 2 or 4
Two positions of a dice are shown below. If 1 is at the bottom,
5 is opposite to 2 or 4
which number will be on top?
2 is opposite to 3 or 5
4 is opposite to 3 or 5
1 is opposite to 6
Illustration – 2
A dice is thrown four times and its four different positions are
given below. Find the number on the face opposite the face (a) 4 (b) 3 (c) 8 (d) 5
showing 2.
Sol. (b) By looking, the dice position, we can say that 2, 4, 5
and 6 are adjacent faces of 3. Therefore, if 1 number is
2 1 3 6 at the bottom then 3 will be on the top.
3 4 2 3 5 4 2 4

(a) 4 (b) 5
(c) 6 (d) 3
Sol. (b) 1, 3, 4 and 6 are adjacent to 2. Therefore, number 5 is on
the face opposite to 2.
39
Module – 15 : Evaluating Inferences

In this section, a passage is given followed by some inferences based on it. We have to examine
each inference in the context of given passage and decide its degree of truth or falsity.
q Shortcut Approach

Take an inference

Is Is
yes it 100% it in the
No
true in context context of the
of passage? passage?

No
yes

Can it be
proved 100%
Data not true using
Is Chance that available some universally No
it contra Chance it is not 100% accepted
-dicts the to prove the assumptions
that it is not true in the truthness
passage? ? Negates the
100% false context of the May or
yes of the passage yes passage
may not
passage Data inadequate Definitely be true
Probably Probably (D.I.) Probably
False (P.F.) True (D.T.)
True (P.T.) Probably True (P.T.)
True (P.T.)
Note: Check the truthness of each inference one by one as above.

Mark answer
(a) if the inference is ‘definitely true’, i.e. it properly follows from the statement of facts given.
(b) if the inference is ‘probably true’ though not ‘definitely true’ in the light of the facts given.
(c) if the ‘data are inadequate’, i.e. from the facts given you cannot say whether the inference is likely to be true or false.
(d) if the inference is ‘probably false’/‘definitely false’ in the light of the facts given.
PASSAGE Solution :
There has been considerable improvement in the economy for the (d) It is not cleared that ‘Hire and Fire’ culture occured
last ten years. Indian economy has witnessed far-reaching changes, after reform in the economy. Hence, it is probably false.
e.g. emergence of India as an IT superpower; there has been wide Illustration – 3
increase in the global employment for the Indian managers and so India can compete with US as it has emerged as an IT superpower.
have the maximum salaries and so on. Solution :
However, there happened great upheaval in corporations, closure
(c) Situation of US economy, especially about IT, is not
of firms and reduction in the size of institutions. There also occurred given.
infiltration of the “Hire & Fire” culture into the corporations.
Employment opportunities have also declined. Illustration – 4
Illustration – 1 There are ample employment opportunities for the Indian
specialists world over following the closure of some Indian firms.
There are certain positive results of reform in economy but there Solution:
are also some unfavourable effects. (c) The given fact is neither mentioned in nor can be
Solution : derived from the contents of the given passage.
(a) The first para talks of “positive result” and the second Illustration – 5
of “unfavourable effect”. The living standard of Indian managers will really improve when
Illustration – 2 they seek jobs in foreign countries.
Solution:
Before reform in the economy in our country, there was no “Hire (b) Fetching maximum salary by Indian managers is likely
and Fire” culture in the world as a whole. to promote living standards.
Hence probably true.
EBD_7711
40
Module – 16 : Critical Reasoning Question: Which of the following can be correctly inferred
from the statements above?
Critical reasoning is an ability to reason clearly to evaluate (a) Areas with little vandalism can never benefit
and judge arguments. You are using this skill a lot during from visual deterrence.
your everyday life while reading newspapers or watching (b) Communities that do not increase their police
movies. patrols are at higher risk for crimes of opportunity
late at night.
TYPES OF CRITICAL REASONING (c) Federal matching grants for police patrols lower the
QUESTIONS rate of crimes of opportunity in some communities.
(d) Only federal matching grants are necessary to
Critical reasoning questions will ask you to: reduce crime in most neighborhoods.
1. Identify the inference / Must be true question
Solution : (c) (c) is a summary of the information provided; it is
2. Identify the assumption.
the logical end of a chain of reasoning started in the stimulus
3. Strengthen an argument.
argument. The sequence of events goes like this :
4. Weaken an argument.
5. Select the best conclusion / Main Point Increased funding ® Increased visual deterrence ®
6. Identify the paradox Lower crime
7. Evaluation/ Reasoning The last statement could be mapped as follows:
8. Identify a parallel argument/Structure. Federal grants ® Increased patrol funds
(c) makes the chain complete by correctly stating that
1. IDENTIFY THE INFERENCE/ MUST federal grants can lead to lower crime in some communities.
BE TRUE QUESTION Now the logical chain becomes:
Federal grants ® Increased funding ® Increased visual
These type of questions are extremely common. An Inference
deterrence ® Lower crime
means the same thing as “must be true”. Conclusions differ
from inferences. Conclusions are the result of premises and The other answer choices may not be correctly inferred
inferences are something that must be true. The following because they go beyond the scope of the argument. They
are the typical Inference (Must be true) based Questions: may be objectively, factually correct, or they may be
statements that you would tend to agree with. However,
• If the statements above are true, which of the following you are limited to the argument presented when choosing
must also be true? a correct answer.
• Which of the following is implied/must be true/implicit/
most reasonably drawn in the passage above?
• Which of the following inferences is best supported by 2. IDENTIFY THE ASSUMPTION
the statement given above? An assumption is an unstated premise that supports the
How to Tackle “Identify the inference / Must be author’s conclusion. It’s the connection between the stated
true questions”: premises and the conclusion. An assumption is something
• Read the stimulus and look for the argument. that the author’s conclusion depends upon. Assumption
• Note that Must Be True questions may not contain an questions are extremely common and have types that look
argument. They may just be a series of facts. like this:
Nevertheless, try to find the argument. Which of the following most accurately states a hidden
• Avoid choices which contain absolute statements - never, assumption that the author must make in order to advance
always, none, only etc. Although these words might the argument above?
appear in some correct choice, you should be very sure Which of the following is an assumption that, if true, would
about them. support the conclusion in the passage above?
• Some of the options can be eliminated as they go beyond
the scope of the passage. Note that an inference can be How to Approach “Identify the Assumption
based on only some of the information provided and not Questions”
the complete passage. • Look for gaps between the premises and the conclusion. Ask
yourself why the conclusion is true. Before you progress to
Illustration – 1 Stimulus Argument
the answer choices, try to get feel of what assumption is
Increases in funding for police patrols often lower the rate of crimes necessary to fill that gap between the premises.
of opportunity such as petty theft and vandalism by providing • Beware of extreme language in the answer choices of
visual deterrence in high-crime neighborhoods. Levels of funding assumption questions. Assumptions usually are not
for police patrols in some communities are increased when federal extreme. “Extreme” answer choices usually contain phrases
matching grants are made available. such as always, never, or totally.
41
Illustration – 2 Stimulus Argument Question: Which of the following statements, if true, would most
strengthen the scientists’ hypothesis?
Traditionally, decision making by doctors that is carefully,
deductively reasoned has been considered preferable to intuitive (a) The native fish species were still able to
decision making. However, a recent study found that senior reproduce in nearby streams where the annual
surgeons used intuition significantly more than did most residents temperature range remains approximately 40°.
or mid-level doctors. This confirms the alternative view that intuition (b) Before the dam was built, the river annually
is actually more effective than careful, methodical reasoning. overflowed its banks, creating temporary backwaters
Question: The conclusion above is based on which of the that were used as breeding areas for the local fish
following assumptions? population.
(a) Senior surgeons are more effective at decision (c) The lowest temperature ever recorded in the river
making than are mid-level doctors. prior to dam construction was 30°; whereas the
(b) Senior surgeons have the ability to use either lowest recorded river temperature after construction
intuitive reasoning or deductive, methodical was completed has been 40°.
reasoning in making decisions. (d) Non-native fish species, introduced after the dam
(c) The decisions that are made by mid-level and entry- was completed, have begun competing with the native
level doctors can be made as easily by using species for food.
methodical reasoning as by using intuitive Solution: (a) most strengthens the conclusion that the scientists
reasoning. reached. It does so by showing that there is a control
(d) Senior surgeons use intuitive reasoning in making group. In other words, a similar population, not subjected
the majority of their decisions. to the same change as the population near the dam, did
not experience the same type of result. Here the basic
Solution : (a) The correct answer is (a), which provides a
assumption about the conclusion that scientists reached
missing link in the author’s reasoning by making a
is that ‘because of the reduction of average temperature
connection from the evidence: that intuition is used more
range of the water, the reproduction of the native fish
by senior surgeons than other, less-experienced doctors,
species has reduced drastically’. Option (a) clearly
and the conclusion: that, therefore, intuition is more
effective. None of the other choices helps bridge this gap strengthens the assumption.
in the chain of reasoning. Although some of the other
statements may be true, they are not responsive to the
4. WEAKEN AN ARGUMENT
question. In fact, they mostly focus on irrelevant factors Assumptions connect premises to conclusions. An
such as appropriateness, ease of application, ability, etc. argument is weakened by weakening the assumptions. Here
are some examples of Weaken question types :
3. STRENGTHEN ARGUMENT
• Which of the following, if true, would weaken the
Assumptions connect premises to conclusions. An argument is conclusion drawn in the passage above?
strengthened by strengthening the assumptions. Here are some • The argument as it is presented in the passage above
examples of Strengthen question types :
would be most strengthened if which of the following were true?
• The conclusion would be more properly drawn if it were
made clear that... How to Approach “Weaken an Argument”
• Which of the following, if true, would most strengthen the Once you have identified the argument of the passage, i.e.
conclusion drawn in the passage above? the evidence(s) + conclusion, try putting in each option
with the argument. Check if the assumption(s) you have
How to Approach “Strengthen an Argument” drawn is (are) weakened if you accept the content of the
Once you have identified the argument of the passage, i.e. the option as true.
evidence(s) + conclusion, try putting in each option with the
argument. Check if the assumption(s) you have drawn is (are) Illustration – 4 Stimulus Argument
strengthened if you accept the content of the option as true. A drug that is very effective in treating some forms of cancer can,
at present, be obtained only from the bark of the Raynhu, a tree
Illustration – 3 Stimulus Argument that is quite rare in the wild. It takes the bark of approximately
Three years after the Bhakra Nangal Dam was built, none of the 5,000 trees to make one pound of the drug. It follows, then, that
six fish species native to the area was still repr oducing adequately continued production of the drug must inevitably lead to the
in the river below the dam. Because the dam reduced the average raynhu’s extinction.
temperature range of the water from approximately 40° to Question: Which of the following, if true, most seriously
approximately 10°, biologists have hypothesized that sharp weakens the above conclusion?
increases in water temperature must be involved in signaling the (a) The drug made from Raynhu bark is dispensed
affected species to begin their reproduction activities. to doctors from a central authority.
EBD_7711
42
(b) The drug made from the Raynhu bark is expensive • Knock out answers with extreme wording. Main Point
to produce. answers typically do not use only, always, never, best or
(c) The Raynhu generally grows in largely inaccessible any strong words that leave little room.
places. Illustration – 5 Stimulus Argument
(d) The Raynhu can be propagated from cuttings and
People should be held accountable for their own behaviour, and if
cultivated by farmers. holding people accountable for their own behaviour entails capital
Solution : (d) (d) provides an alternate source of the Raynhu punishment, then so be it. However, no person should be held
bark. Even though the tree is rare in the wild, the argument accountable for behaviour over which he or she had no control.
is silent on the availability of cultivated trees. The author Question : Which of the following is the most logical conclusion
of the argument must be assuming that there are no Raynhu of the argument above?
trees other than those in the wild, in order to make the leap (a) People should not be held accountable for the
from the stated evidence to the conclusion that the Raynhu behaviour of other people.
is headed for extinction. The option (d) weakens the (b) People have control over their own behaviour.
assumption - ‘there are limited Raynhu trees’ - by saying
(c) People cannot control the behaviour of other people.
that there are other ways as well for the propogation of
Raynhu. The other answer choices all contain information (d) People have control over behaviour that is subject to
that is irrelevant. Note that the correct choice does not capital punishment.
make the conclusion of the argument impossible. In fact, it Solution : (b) The correct response is (b). The argument
is possible that there may be domesticated Raynhu trees includes the following two premises:
and the species could still become extinct. Answer choice Premise 1: People are accountable for their own behaviour.
(d) is correct because it makes the conclusion about Premise 2: People are not accountable for behaviour they
extinction less likely to be true. cannot control.
Here’s the logical conclusion based on these two premises:
5. CONCLUSION/MAIN POINT Conclusion: People can control their own behaviour.
QUESTION (a) would require that people never have control over
the behaviour of other people. Yet the argument does
In Main Point / Conlcusion questions, you have to identify the not provide this premise.
conclusion of an argument. You are trying to find the author’s
(c) would require that people should not be held
point and should approach this question in a similar way to the
accountable for the behaviour of other people. Yet
reading comprehension main point questions. They come in the argument does not provide this premise.
several different formats:
(d) is not inferable.
• The main point of the passage is that...
• Which of the following statements about... is best 6. IDENTIFY THE PARADOX
supported by the statements above? These questions present you with a paradox, a seeming
• Which of the following best states the author’s conclusion contradiction or discrepancy in the argument, and ask you
in the passage above? to resolve it or explain how that contradiction could exist.
• Which of the following conclusions can be most properly In other words, there are two facts that are both true, and
drawn from the data above? yet they appear to be in direct conflict with one another.
The conclusion of arguments in Main Point questions is Here are some examples of the ways in which these
usually not directly stated. To find the conclusion, identify questions are worded:
the premises and then identify the conclusion drawn from Which of the following, if true, would help to resolve the
the premises. Main Point questions differ from the other apparent paradox presented above?
Critical Reasoning questions in that the argument in the Which of the following, if true, contributes most to an
stimulus is usually valid. (In most other Critical Reasoning explanation of the apparent discrepancy described above?
questions the reasoning is flawed.) Conclusion questions
require you to choose the answer that is a summary of the How to Approach “Identify the Paradox
Questions”
argument.
• Read the argument and find the apparent paradox,
How to Approach “Main Point Questions” discrepancy, or contradiction.
• Main Point answers must be within the scope of the • State the apparent paradox, discrepancy, or contradiction
passage. in your own words.
• Your opinions or information outside of the passage are • Use process of elimination. The best answer will explain
always outside of the scope. how both sides of the paradox, discrepancy, or
• Some of the options given can be out of the scope of the contradiction can be true. Eliminate answers that are out
passage. of scope.
43
Illustration – 6 Stimulus Argument This record demonstrates that the recently elected judge has not
discriminated against men in cases of child custody.
Town Y is populated almost exclusively by retired people and has almost
no families with small children. Yet Town Y is home to a thriving business Question: The argument above is flawed in that it ignores the
specializing in the rental of furniture for infants and small children. possibility that
Question: Which of the following, if true, best reconciles the (a) A large number of the recently elected judge’s cases
seeming discrepancy described above? involve child custody disputes.
(a) The business specializing in the rental of (b) The recently elected judge is prejudiced against men
children’s furniture buys its furniture from in divorce cases that do not involve child custody
distributors outside of Town Y. issues.
(b) The few children who do reside in Town Y all know (c) The majority of the child custody cases that have
each other and often stay over night at each other’s reached the recently elected judge’s court have been
houses. appealed from a lower court.
(c) Many residents of Town Y who move frequently prefer (d) The evidence shows that men should have won
to rent their furniture rather than buy it outright. custody in more than 40% of the recently elected
(d) Many residents of Town Y must provide for the needs judge’s cases involving divorcing fathers.
of visiting grandchildren several weeks a year. Solution : (d) The correct answer (d), points out a flaw in the
Solution : (d) The correct answer (d), explains why a town of argument. Specifically, it points out that the author of the
mostly retired residents might need to rent children’s argument was comparing the recently elected judge to
furniture. The other answer choices all contain irrelevant other judges, not to the evidence presented in the recently
information. This further illustrates the fact that, on all elected judge’s cases. In other words, the author of the
question types, if you eliminate the irrelevant choices, the argument made an unwarranted assumption that the
remaining choice will most likely be correct. recently elected judge did not rule against many men in
custody battles where the evidence clearly favored the
7. EVALUATION/REASONING men. As with strengthening and weakening questions, the
BASED QUESTIONS correct answer in flaw questions often involves
unwarranted assumptions.
Reasoning questions ask you to describe how the
argument was made, not necessarily what it says. These Illustration – 8 Stimulus Argument
questions are closely related to assumption, weakening,
Although dentures produced through a new computer-aided
and strengthening questions. The correct answer identifies
a question that must be answered or information that must design process will cost more than twice as much as ordinary
be gathered to determine how strong the stimulus argument dentures, they should still be cost effective. Not only will fitting
is. The information will be related to an assumption that time and X-ray expense be reduced, but the new dentures should
the author is making. Another type of question that you fit better, diminishing the need for frequent refitting visits to the
will encounter asks you to identify a flaw in the stimulus dentist’s office.
argument. The question tells you that there is a problem Question: Which of the following must be studied in order to
with the logic of the argument. You just have to choose evaluate the argument presented above?
the answer that describes the flaw. Here are some examples
of the ways in which these questions are worded: (a) The amount of time a patient spends in the fitting
process versus the amount of money spent on X-
• How does the author make his point?
rays
• A major flaw in the argument above is that it...
(b) The amount by which the cost of producing dentures
• A’s response has which of the following relationships to
has declined with the introduction of the new
B’s argument?
technique for producing them
How to Approach Reasoning Questions (c) The degree to which the use of the new dentures is
• Read the argument and find the conclusion. likely to reduce the need for refitting visits when
• State the reasoning in your own words. compared to the use of ordinary dentures
• Check whether the reasoning given in the various options (d) The amount by which the new dentures will drop in
fall in line with the reasoning described above. cost as the production procedures become
standardized and applicable on a larger scale
Illustration – 7 Stimulus Argument
Solution : (c) The correct answer (c), highlights an
Some observers have taken the position that the recently elected assumption in the stimulus argument. It shows that the
judge is biased against men in divorce cases that involve child author must be assuming that the reduction in refitting
custody. But the statistics reveal that in 40% of such cases, the with the new dentures compared to ordinary dentures is
recently elected judge awards custody to the fathers. Most other significant in order to conclude that that difference will
judges award custody to fathers in only 20%–30%of their cases.
EBD_7711
44
help offset an initial outlay that is twice as much. In other (b) It is true that it is illegal to sell diamonds that
words, if you answer the question posed by answer choice originate in certain countries. But there is a long
(c) with “not much,” the argument is weakened. If you tradition in Russia of stockpiling diamonds.
answer it with “a tremendous amount,” the argument is (c) It is true that it is illegal for an attorney to participate
in a transaction in which there is an apparent conflict
strengthened. The other answer choices are all irrelevant
of interest. But, if the facts are examined carefully,
because no matter what the answers are, there is no impact it will clearly be seen that there is no actual conflict
on the relationship between the evidence presented in the of interest in the defendant’s case.
stimulus argument and its conclusion. (d) It is true that it is against the law to steal cars. But
someone else certainly would have stolen that car if
8. IDENTIFY A PARALLEL the defendant had not done so first.
ARGUMENT/STRUCTURE. Solution: (d) The correct answer (d), has the same structure
as the stimulus argument. If you just replace “aid to
The last type of Critical Reasoning question is the parallel developing nuclear powers” with “car theft,” and “Russian
structure question. In this type of question, you must choose the companies” with the “defendant,” it is essentially the same
answer that has the same structure as the stimulus argument. In argument. Sometimes the parallel structure is easier to see
other words, you have to find the argument that is analogous to if you use symbols to represent the terms of the argument:
the given argument in that it includes the same relationship It is true that X is illegal. But, if Y doesn’t do it, others will.
between the evidence presented and the conclusion. Here are Granted, the stimulus argument is in the future tense and
some examples of the ways in which these questions are worded: the credited answer is in the past tense. However, it
• Which of the following is most like the argument above in certainly is most like the stimulus.
its logical structure?
GENERAL APPROACH FOR SOLVING
• Which of the following is a parallel argument to the above
given argument? CRITICAL REASONING QUESTIONS
Illustration – 9 Stimulus Argument (i) Read the question stem first. This will help you to decide
what you are going to be looking for in the paragraph when
It is true that it is against international law to provide aid to certain
you read it for the first time.
countries that are building nuclear programs. But, if Russian
companies do not provide aid, companies in other countries will. (ii) Next read the passage, identifying the claim, the supporting
reasons / evidence, the assumptions inherent in the argument
Question: Which of the following is most like the argument and the inferences that can be made from the argument.
above in its logical structure?
(iii) Pre-phrase your answer-knowing what you are going to be
(a) It is true that it is against United States policy to
looking for while reading the options is helpful in order to
negotiate with kidnappers. But if the United States
wants to prevent loss of life, it must negotiate in get the correct answer. Then check the options to see which
some cases. one best fits your option.
45

Quiz – 3 on Mental Ability, Logical


Reasoning & Problem solving
1. This questions is based on the information given below : • D's daughter sits second to right of F. F is the sister of G. F is
not an immediate neighbour of H's husband.
• Only one person sits between A and F. A is the father of G.
A B C D H's brother D sits to the immediate right of H's mother. Only
N one person sits between H's mother and E.
W E
• Only one person sits between H and G. G is the mother of C.
S G is not an immediate neighbour of E.
S R Q P 4. What is the position of A with respect to his mother-in-law ?
(a) Immediate left (b) Third to the right
A posse of four policemen A, B, C and D is (c) Third to the left (d) Second to the right
combing in a circular park which is divided 5. What is the position of A with respect to his grandchild ?
into sixteen plots as shown. (a) Immediate right (b) Third to the right
P, Q, R and S are the offenders whom they (c) Third to the left (d) Second to the left
have to catch after given moves. The figure Directions (Qs. 6-7): Study the following information and
shows their positions. answer the questions that follow:
Note that the by-lanes are North-south and Group captain Malhotra is choosing the last part of his crew for
East-west. the spaceship COSMOS, with which he plans to land on the
If A, B, C and D were to move clockwise four plots and P, Q, moon. He needs 4 more crew members of whom at least two
R and S were to move anti-clockwise six plots, then who two must be pilots, the others being engineers. The candidates for
would be the north-south ? Pilots are Dalbir, Eric and Farid. The candidates for Engineers
(a) P, A (b) Q, B
are Lal, Monty, Naveen and Paul.
(c) R, D (d) P, C
Eric will not be a crew with Lal, Dalbir and Paul will not crew
2. In a cricket team, three batsman Ricky, Sachin and Brian are
the top three run-scorers in any order. Each of them gives with Naveen.
two replies to any question, one of which is true and the 6. If Naveen is chosen, which of the following must be other
other is false, again, in any order. When asked about who members of the crew?
the top scorer was, following were the replies they gave : (a) Farid, Lal and Monty
Sachin : I got the top score. Ricky was second. (b) Dalbir, Eric and Monty
Brian : I got the top score. Sachin was second (c) Eric, Farid and Monty
Ricky : I got the top score. Sachin was third. (d) Eric. Farid and Paul
Which of the following is the correct order of batsmen who 7. If Paul is chosen, which candidates will NOT be chosen
got the top score, second best and third best score to be on the crew?
respectively? (a) Dalbir, Eric and Monty
(a) Brian, Ricky, Sachin (b) Brian, Sachin, Ricky (b) Dalbir, Eric and Farid
(c) Ricky, Sachin, Brian (d) Sachin, Brian, Ricky (c) Dalbir, Farid and Lal
3. Six identical cards are placed on a table. Each card has number (d) Eric, Farid and Lal
'1' marked on one side and number '2' marked on its other side. 8. Statement: Should all the deemed universities be
All the six cards are placed in such a manner that the number derecognised and attached to any of the central or state
'1' is on the upper side. In one try, exactly four (neither more universities in India ?
nor less) cards are turned upside down. In how many least Arguments :
number of tries can the cards be turned upside down such that (A) Yes, many of these deemed universities do not conform
all the six cards show number '2' on the upper side? to the required standards of a full – fledged university
(a) 3 (b) 5
and hence the level of education is compromised.
(c) 7 (d) This cannot be achieved
(B) No, these deemed universities have been able to
Directions (Qs. 4-5) : Study the following information carefully introduce innovative courses suitable to the
and answer the given questions : requirement of various industries as they are free from
strict Government controls.
A, B, C, D, E, F, G and H are sitting around a circle facing the centre (C) Yes, many such universities are basically money
but not necessarily in the same order. spinning activities and education takes a backseat in
• B sits second to left of H's husband. No female is an immediate these institutions
neighbour of B.
EBD_7711
46
(a) Only (A) and (B) are strong (a) Plan A (b) Plan B
(b) Only (B) and (C) are strong (c) Plan C (d) Data insufficient
(c) Only (A) and (C) are strong 11. If 'P $ Q', means 'P is father of Q'; 'P # Q' means 'P is mother
(d) All (A), (B) and (C) are strong of Q'; 'P * Q' means 'P is sister of Q', then how is D related to
9. Most large retail stores of all goods and brands hold discount N in N # A $ B * D?
sales in the month of November. The original idea of price (a) Nephew (b) Grandson
reduction campaigns in November became popular when it (c) Granddaughter (d) Cannot be determined
was realized that the sales of products would generally slow 12. Consider that :
down following the Diwali rush. The lack of demand could 1. A is taller than B.
be solved by the simple solution of reducing prices. There is 2. C is taller than A.
now an increasing tendency among major chains of stores
3. D is taller than C.
across the country to have their “November sales” begin
4. E is the tallest of all.
before Diwali. The idea behind this trend is to endeavour to
sell the maximum amount of stock at a profit, even if that may If they are made to sit in the above order of their height, who
not be at the maximum profit. will occupy the mid position ?
Which of the following conclusions cannot be logically (a) A (b) B
drawn from the above? (c) C (d) D
(a) The incidence of “early” November sales results in 13. Among five boxes numbered 1, 2, 3, 4 and 5 each has a
lower holdings of stocks with the corollary of lower different weight. Box no.1 is heavier than Box no. 5 but lighter
stock holding costs. than box no. 3. Box no. 4 is heavier than Box no. 3 but lighter
(b) Demand is a function of price; as you lower price, than Box no. 2. Which of the following represents the boxes
demand increases. which are the heaviest and the lightest respectively?
(c) Major stores seem to think it makes sense to have the (a) Box no. 4 and Box no. 1
November sales campaigns pre-Diwali. (b) Box no. 2 and Box no. 5
(d) The major department stores do not worry as much (c) Box no. 2 and Box no. 1
about profit maximization as they do about sales
maximization. (d) Box no. 4 and Box no. 5

DIRECTIONS (Qs. 14-15) : Read the following passage and


Directions (Qs. 10): Read the following information carefully
solve the questions based on it.
and answer the questions given below.
Five friends—Umesh, Vishnu, Xinhua, Yogesh and Zeta collected
M/s XYZ Invest, an Investment company offers three type pebbles on the sea shore. They collectged a total of 100 pebbles.
of investment plans which are as under; None of them collected less than 10 pebbles each. No two among
Plan A- them collected the same number of pebbles. Following information is
• Investment in export business. given regarding the number of pebbles with each one of them:
• Rate of return 30% simple interest per annum.
(i) Umesh collected the same number of pebbles as Vishnu and
• No return on investment is given in the first three years.
Xinhua put together.
Plan B-
• Investment in manufacturing concern. (ii) Xinhua collected 3 more pebbles than the cube of an integer.
• Rate of return 20% per annum simple interest. (iii) The number of pebbles collected by Umesh was the square
• 20% of investment is used in procurement of licences of an integer.
and other establishment charges on which no return is (iv) The number of pebbles collected by Vishnu was either the
given. At the end of the term only 80% of principal and square or the cube of an integer.
accrued interest there upon is returned back.
(v) The number of pebbles collected by Yogesh and Zeta are in
Plan C-
the ratio 4 : 3.
• Investment in stock market
14. What was the number of pebbles collected by Umesh?
• Rate of return is at 15% per annum (simple interest)
(a) 19 (b) 36
At the end of the specified period in all three investment, the
(c) 52 (d) 64
interest as well as the principal is returned back to the
15. What was the number of pebbles collected by Vishnu?
investor in one lump sum.
(a) 16 (b) 25
Ms. Sinha wants to invest ` 1,00,000/- in any one scheme for
(c) 46 (d) 64
five years. Which will be the most beneficial to him?
47

Hints and Solutions


1. (c) 8. (c) Only Argument (A) and (C) are strong because
P Q R S compromise with level of education cannot be done.
A 9. (c) Statement (a) is true as the goods will be sold earlier so
B
the cost of stocking will be reduced. (b) is clearly true
as is directly mentioned in the passage. (d) is correct as
D C it is clear from the last sentence of the passage that
2. (a) Only one of 1st replies given by all three can be true as even a small profit is good enough for these stores.
top score can be taken by only one. Similarly, only two Only (c) conclusion can not be drawn from the passage.
10. (c) (Plan C)
of 2nd replies given by all three can be true as maximum
Amount received back after 5 years in 3 plans are;
true replies are three. Now, consider the each of replies.
Plan A - 1,60,000/- : ` 60,000/- Interest + ` 1,00,000/- principal
Suppose Sachin’s 1st reply is true and Brian’s and
Plan B - 1,60,000/- : ` 80,000/- Interest + ` 80,000/- principal
Ricky’s 2nd reply is true. Then Brian’s and Ricky’s 1st Plan C - 1,75,000/- : ` 75,000/- Interest + ` 1,00,000/- principal
reply will be false. Then, 11. (d)
Sachin Brian Ricky N (–)
I Sachin
II Brian Sachin
III Ricky Sachin A (+)
Hence, it is not possible.
Now, suppose Brian’s 1st reply is true and Sachin’s
and Ricky’s 2nd reply is true. Then B (–) D
Data is not sufficient. So can't be determined.
Briain Sachin Ricky 12. (c) Sequence according to height.
I Briain E > D> C>A> B
II Ricky Ricky
III Sachin Sachin
Mid Position
Hence, it is clear that option (a) is correct. Therefore, ‘C’ occupys the mid position.
3. (a) 13. (b) The order of weight of boxes is as follows
3 Flip Box no. 2 > Box no. 4 > Box no. 3 > Box no. 1 > Box
no. 5
1 1 1 1 1 1 Sol. (14-15):
Flip U—Umesh V—Vishnu
1 2 2 2 2 1 1st tries X—Xinhua Y—Yogesh
Flip Z—Zeta
1 2 1 1 1 2 2nd tries We have
2 2 2 2 2 2 3rd tries U + V + X + Y + Z = 100
4. (d) A's wife is H. H's mother is B. Therefore, B is the mother- and each one of U, V, W, X £ 10
in-law of A, A is second to the right of B. U = V + X and 3Y = 4Z
5. (b) A is the father of F and G. G is the mother of C. Therefore, Now,
C is the grandchild of A. A is third to the right of C. X = 8 + 3 = 11
7. (b) Except C, all others are females. or X = 27 + 3 = 30 and
Sol. (6-7): U = 16, 25, 36, 49 and
Group Captain – Malhotra V = 16, 25, 27, 36
Observing the values of U, V and X, we can easily get that
Additional No. of crews = 4
X(11) + V(25) = U(36)
At least two are pilots.
So, X = 11, V = 25
While others are engineers.
and U = 36
Candidates for enginers: Lal, Monty, Naveen and Paul. Now, Y + Z = 100 – (U + V + X)
Candidates for pilots: Dalbir, Eric and Farid. = 100 – (11 + 36 + 25)
6. (c) As Naveen is chosen, Dalbir and paul cannot be chosen. = 28
Therefore, the pilots chosen are Eric and Farid. Eric cannot now put Y = 4/3 Z in the above equation to get Z = 12 and
be a crew with Lal. Hence, Monty will be chosen. then Y = 16.
7. (d) As it has been stated, Eric cannot be a crew with Lal. 14. (b) 15. (b)
EBD_7711
Previous Years' Questions
BASIC NUMERACY & DATA (a) C is the richest
(b) D is the poorest.
INTERPRETATION (c) C has more than what A and D have together.
1. For a charity show, the total tickets sold were 420. (d) B is richer than D
Half of these tickets were sold at the rate of ` 5 each, 9. In a town, 45% population read magazine A, 55% read
one-third at the rate of ` 3 each and the rest for ` 2 magazine B, 40% read magazine C, 30% read
each. What was the total amount received? [2014 - II] magazines A and B, 15% read magazines B and C, 25%
(a) ` 900 (b) ` 1,540 read magazines A and C; and 10% read all the three
(c) ` 1,610 (d) ` 2,000 magazines. What percentage do not read any
2. If Sohan, while selling two goats at the same price, magazine? [2015-II]
makes a profit of 10% on one goat and suffers a loss of (a) 10% (b) 15%
10% on the other [2014 - II] (c) 20% (d) 25%
(a) he makes no profit and no loss. 10. Two equal glasses of same type are respectively 1/3
(b) he makes a profit of 1%. and 1/4 full of milk. They are then filled up with water
(c) he suffers a loss of 1%. and the contents are mixed in a pot. What is the ratio
(d) he suffers a loss of 2%. of milk and water in the pot? [2015-II]
3. Five persons fire bullets at a target at an interval of 6, (a) 7 : 17 (b) 1 : 3
7, 8, 9 and 12 seconds respectively. The number of times (c) 9 : 21 (d) 11 : 23
they would fire the bullets together at the target in an 11. In a test, a candidate attempted only 8 questions and
hour is [2014 - II] secured 50% marks in each of the questions. If he
(a) 6 (b) 7 obtained a total of 40% in the test and all questions in
(c) 8 (d) 9 the test carried equal marks, how many questions were
4. A bell rings every 18 minutes. A second bell rings every there in the test? [2015-II]
24 minutes. A third bell rings every 32 minutes. Ifall (a) 8 (b) 10
the three bells ring at the same time at 8 o'clock in the (c) 15 (d) 16
morning, at what other time will they all ring 12. A father is nine times as old as his son and the mother
together? [2014 - II] is eight times as old as the son. The sum of the father's
(a) 12 : 40 hrs (b) 12 : 48 hrs and the mother's age is 51 years. What is the age of
(c) 12 : 56 hrs (d) 13 : 04 hrs the son ? [2015-II]
5. As per agreement with a bank, a businessman had to (a) 7 years (b) 5 years
refund a loan in some equal instalments without (d) 4 years (d) 3 years
interest. After paying 18 instalments he found that 13. The monthly incomes of Peter and Paul are in the ratio
60 percent of his loan was refunded. How many of 4 : 3. Their expenses are in the ratio of 3 : 2. If each
instalments were there in the agreement ? [2014 - II] saves ` 6,000 at the end of the month, their monthly
(a) 22 (b) 24 incomes respectively are (in `) [2015-II]
(c) 30 (d) 33 (a) 24,000 and 18,000 (b) 28,000 and 21,000
6. A person X has four notes of Rupee 1, 2, 5 and 10 (c) 32,000 and 24,000 (d) 34,000 and 26,000
denomination. The number of different sums of money
Directions (Qs. 14-15) : Read the following passage and answer
she can form from them is [2010]
the 2 (two) items that follow:
(a) 16 (b) 15
(c) 12 (d) 8 A, B, C, D, E and F are cousins. No two cousins are of the
7. There are 100 students in a particular class. 60% same age, but all have birthdays on the same day of the
students play cricket, 30% student play football and same month. The youngest is 17 years old and the oldest E
10% students play both the games. What is the is 22 years old. F is somewhere between B and D in age. A is
number of students who play neither cricket nor older than B. C is older than D. A is one year older than C.
football ? [2011 - II] [2015-II]
(a) 25 (b) 20 14. Which one of the following is possible?
(c) 18 (d) 15 (a) D is 20 years old (b) F is 18 years old
8. Each of A, B, C and D has ` 100. A pays ` 20 to B, who (c) F is 19 years old (d) F is 20 years old
pays ` 10 to C, who gets ` 30 from D. In this context, 15. What is the number of logically possible orders of all
which one of the following statements is not correct? six cousins in terms of increasing age?
(a) 1 (b) 2
[2015-II]
(c) 3 (d) 4
49
16. A cow costs more than 4 goats but less than 5 goats. If job. How long would Ram take to complete the entire
a goat costs between ` 600 and ` 800, which of the job alone? [2016-II]
following is a most valid conclusion? [2015-II] (a) 6 days (b) 8 days
(a) A cow costs more than ` 2,500. (c) 10 days (d) 11 days
(b) A cow costs less than ` 3,600. 24 If R and S are different integers both divisible by 5,
(c) A cow costs between ` 2,600 and ` 3,800. then which of the following is not necessarily true?
(d) A cow costs between ` 2,400 and ` 4,000. (a) R – S is divisible by 5 [2016-II]
17. Candidates in a competitive examination consisted (b) R + S is divisible by 10
of 60% men and 40% women. 70% men and 75% (c) R × S is divisible by 25
women cleared the qualifying test and entered the (d) R2 + S2 is divisible by 5
final test where 80% men and 70% women were 25. How many numbers are there between 100 and 300
successful. which either begin with or end with 2? [2016-II]
Which of the following statements is correct? [2015-II] (a) 110 (b) 111
(a) Success rate is higher for women. (c) 112 (d) None of the above
(b) Overall success rate is below 50%. 26. W can do 25% of a work-in 30 days, X can do 1/4 of the
(c) More men cleared the examination than women. work in 10 days, Y can do 40% of the work in 40 days
(d) Both (a) and (b) above are correct. and Z can do 1/3 of the work in 13 days. Who will
18. An agricultural field is in the form of a rectangle complete the work first? [2016-II]
having length X1 meters and breadth X2 meters (X1 (a) W (b) X
and X2 are variable). If X1 + X2 = 40 meters, then the (c) Y (d) Z
area of the agricultural field will not exceed which 27. The average monthly income of a person in a certain
one of the following values? [2016-II] family of 5 is ` 10,000. What will be the average monthly
(a) 400 sq m (b) 300 sq m income of a person in the same family if the income of
(c) 200 sq m (d) 80 sq m one person increased by ` 1,20,000 per year?
19. The sum of the ages of 5 members comprising a (a) ` 12,000 (b) ` 16,000 [2016-II]
family, 3 years ago, was 80 years. The average age of (c) ` 20,000 (d) ` 34,000
28. In a race, a competitor has to collect 6 apples which
the family today is the same as it was 3 years ago,
are kept in a straight line on a track and a bucket is
because of an addition of a baby during the
placed at the beginning of the track which is a
intervening period. How old is the baby ? [2016-II]
starting point. The condition is that the competitor
(a) 6 months (b) 1 year
can pick only one apple at a time, run back with it
(c) 2 years (d) 2 years and 6 months
and drop it in the bucket. If he has to drop all the
20. The total emoluments of two persons are the same,
apples in the bucket, how much total distance he has
but one gets allowances to the extent of 65% of his
to run if the bucket is 5 meters from the first apple and
basic pay and the other gets allowances to the extent
all other apples are placed 3 meters apart ?
of 80% of his basic pay. The ratio of the basic pay of (a) 40 m (b) 50 m [2016-II]
the former to the basic pay of the latter is: [2016-II] (c) 150 m (d) 75 m
(a) 16 : 13 (b) 5 : 4 29. A round archery target of diameter 1 m is marked
(c) 7 : 5 (d) 12 : 11 with four scoring regions from the centre outwards
21. A person is standing on the first step from the bottom as red, blue, yellow and white. The radius of the red
of a ladder. If he has to climb 4 more steps to reach band is 0.20 m. The width of all the remaining bands
exactly the middle step, how many steps does the is equal. If archers throw arrows towards the target,
ladder have? [2016-II] what is the probability, that the arrows fall in the red
(a) 8 (b) 9 region of the archery target? [2016-II]
(c) 10 (d) 11 (a) 0.40 (b) 0.20
22. AB is a vertical trunk of a huge tree with A being the (c) 0.16 (d) 0.04
point where the base of the trunk touches the ground. 30 A person allows 10% discount for cash payment from
Due to a cyclone, the trunk has been broken at C the marked price of a toy and still he makes a 10%
which is at a height of 12 meters, broken part is gain. What is the cost price of the toy which is
partially attached to the vertical portion of the trunk marked ` 770? [2016-II]
at C. If the end of the broken part B touches the (a) ` 610 (b) ` 620
ground at D which is at a distance of 5 meters from A, (c) ` 630 (d) ` 640
then the original height of the trunk is: [2016-II] 31. A class starts at 11:00 am and lasts till 2:27 pm. Four
(a) 20 m (b) 25 m periods of equal duration are held during this interval.
(c) 30 m (d) 35 m After every period, a rest of 5 minutes is given to the
23. Ram and Shyam work on a job together for four days students. The exact duration of each period is:
and complete 60% of it. Ram takes leave then and (a) 48 minutes (b) 50 minutes [2016-II]
Shyam works for eight more days to complete the (c) 51 minutes (d) 53 minutes
EBD_7711
50
32. 30g of sugar was mixed in 180 ml water in a vessel A, income of B and D. Whose income is the highest?
40 g of sugar Was mixed in 280 ml of water in vessel B (a) A (b) B [2017-II]
and 20 g of sugar was mixed in 100 ml of water in (c) C (d) D
vessel C. The solution in vessel B is : [2016-II] 40. There are three pillars, X, Y and Z of different heights.
(a) sweeter than that in C Three spiders A, B and C start to climb on these
(b) sweeter than that in A pillars simultaneously. In one chance. A climbs on
(c) as sweet as that in C X by 6cm but slips down 1 cm. B climbs on Y by 7
(d) less sweet than that in C cm but slips down 3 cm. C climbs on Z by 6.5 cm
33. In aid of charity, every student in a class contributes but slips down 2 cm. If each of the requires 40
as many rupees as the number of students in that chances to reach the top of the pillars, what is the
class. With the additional contribution of ` 2 by one height of the shortest pillar? [2017-II]
student only, the total collection is ` 443. Then how (a) 161 cm (b) 163 cm
many students are there in the class? [2016-II]
(c) 182 cm (d) 210 cm
(a) 12 (b) 21
(c) 43 (d) 45 41. 15 students failed in a class of 52. After removing
34. Anita’s mathematics test had 70 problems carrying the names of failed students, a merit order list has
equal marks i.e., 10 arithmetic, 30 algebra and 30 been prepared in which the position of Ramesh is
geometry. Although she answered 70% of the 22nd from the top. What is his position from the
arithmetic, 40% of the algebra and 60% of the bottom? [2017-II]
geometry problems correctly, she did not pass the (a) 18th (b) 17th
test because she got less than 60% marks. The (c) 16th (d) 15th
number of more questions she would have to answer 42. Gopal bought a cell phone and sold it to Ram at 10%
correctly to earn a 60% passing marks is: [2016-II] profit. Then Ram wanted to sell it back to Gopal at
(a) 1 (b) 5 10% loss. What will be Gopal's position if he agreed?
(c) 7 (d) 9 (a) Neither loss nor gain [2017-II]
35. In a class, there are 18 very tall boys. If these (b) Loss 1%
constitute three-fourths of the boys and the total (c) Gain 1%
number of boys is two-thirds of the total number of (d) Gain 0.5%
students in the class, what is the number of girls in
43. Suppose the average weight of 9 persons is 50 kg.
the class? [2016-II]
The average weight of the first 5 persons is 45 kg,
(a) 6 (b) 12
whereas the average weight of the last 5 persons is
(c) 18 (d) 21
55 kg, Then the weight of the 5th person will be
36. The monthly average salary paid to all the employees
of a company was ` 5000. The monthly average (a) 45 kg (b) 47.5 kg [2017-II]
salary paid to male and female employees was ` 5200 (c) 50 kg (d) 52.5 kg
and ` 4200 respectively. Then the percentage of males 44. A watch loses 2 minutes in every 24 hours. while
employed in the company is : [2016-II] another watch gains 2 minutes in every 24 hours.
(a) 75% (b) 80% At a particular instant, the two watches showed an
(c) 85% (d) 90% identical time. Which of the following statements is
37. Two numbers X and Y are respectively 20% and 28% correct if 24-hour clock is followed? [2017-II]
less than a third number Z. By what percentage is the (a) The two watches show the identical time again
number Y less than the number X ? [2016-II] on completion of 30 days.
(a) 12% (b) 10% (b) The two watches show the identical time again
(c) 9% (d) 8% on completion of 90 days.
38. If there is a policy that 1 / 3rd of a population of a (c) The two watches show the identical time again
community has migrated every year from one place on completion of 120 days.
to some other place, what is the leftover population
(d) None of the above statements is correct.
of that community after the sixth year, if there is no
further growth in the population during this period? 45. In a city, 12% of households earn less than ` 30,000
(a) 16/243rd part of the population [2017-II] per year, 6% households earn more than ` 2,00,000
per year, 22% households earn more than `1,00,000
(b) 32/243rd part of the population
per year and 990 house- holds earn between ` 30,000
(c) 32 / 729th part of the population
and ` 1,00,000 per year. How many households earn
(d) 64/ 729th part of the population
between ` 1,00,000 and ` 2,00,000 per year? [2017-II]
39. The sum of income of A and B is more than that of
(a) 250 (b) 240
C and D taken together. The sum of income of A and
C is the same as that of B and D taken together. (c) 230 (d) 225
Moreover, A earns half as much as the sum of the
51
46. A clock strikes once at 1o'clock, twice at 2o'clock and his age to become the cube of a number again ?
thrice at 3 o'clock, and so on. If it takes 12 seconds [2017-II]
to strike at 5 o'clock, what is the time taken by it to (a) 42 (b) 38
strike at 10 o'clock? [2017-II] (c) 25 (d) 16
(a) 20 seconds (b) 24 seconds 55. P works thrice as fast as Q, whereas P and Q together
(c) 28 seconds (d) 30 seconds can work four times as fast as R. If P, Q and R
47. There are thirteen 2-digit consecutive odd numbers. together work on a job, in what ratio should they
If 39 is the mean of the first five such numbers, then share the earnings ? [2017-II]
what is the mean of all the thirteen numbers? (a) 3 : 1 : 1 (b) 3 : 2 : 4
[2017-II] (c) 4 : 3 : 4 (d) 3 : 1 : 4
(a) 47 (b) 49 56. A 2-digit number is reversed. The larger of the two
(c) 51 (d) 45 numbers is divided by the smaller one. What is the
48. There is a milk sample with 50% water in it. If 1/3rd largest possible remainder [2017-II]
of this milk is added to equal amount of pure milk, (a) 9 (b) 27
then water in the new mixture will fall down to (c) 36 (d) 45
[2017-II] 57. The monthly incomes of X and Y are in the ratio of
(a) 25% (b) 30% 4 : 3 and their monthly expenses are in the ratio of
(c) 35% (d) 40% 3 : 2. However, each saves ` 6,000 per month. What
49. There are 4 horizontal and 4 vertical lines, parallel is their total monthly income? [2017-II]
and equidistant to one another on a board. What is (a) ` 28,000 (b) ` 42,000
the maximum number of rectangles and squares (c) ` 56,000 (d) ` 84,000
that can be formed? [2017-II] 58. Two walls and a ceiling of a room meet at right
(a) 16 (b) 24 angles at a point P. A fly is in the air 1 m from one
(c) 36 (d) 42 wall, 8 m from the other wall and 9 m from the
50. In a test, Randhir obtained more marks than the total point P. How many meters is the fly from the ceiling ?
marks obtained by Kunal and Debu. The total marks [2017-II]
obtained by Kunal and Shankar are more than those (a) 4 (b) 6
of Randhir. Sonal obtained more marks than Shankar, (c) 12 (d) 15
Neha obtained more marks than Randhir. Who 59. There are certain 2-digit numbers. The difference
amongst them obtained highest marks? [2017-II] between the number and the one obtained on
(a) Randhir (b) Neha reversing it is always 27. How many such maximum
(c) Sonal (d) Data are inadequate 2-digit numbers are there? [2017-II]
51. Certain 3-digit numbers have the following (a) 3 (b) 4
characteristics : [2017-II] (c) 5 (d) None of the above
1. All the three digits are different.
60. What is the total number of digits printed, if a book
2. The number is divisible by 7.
containing 150 pages is to be numebred from 1 to 150
3. The number on reversing the digits is also
[2017-II]
divisible by 7.
(a) 262 (b) 342
How many such 3-digit numbers are there?
(c) 360 (d) 450
(a) 2 (b) 4
61. A gardener increased the area of his rectangular
(c) 6 (d) 3
garden by increasing its length by 40% and decreasing
52. How many numbers are there between 99 and 1000
its width by 20%. The area of the new garden [2014 - II]
such that the digit 8 occupies the units place?
[2017-II] (a) has increased by 20%.
(a) 64 (b) 80 (b) has increased by 12%.
(c) 90 (d) 104 (c) has increased by 8%.
53. If for a sample data [2017-II] (d) is exactly the same as the old area.
Mean < Median < Mode
62. In a plane, line X is perpendicular to line Y and parallel
then the distribution is
to line Z; line U is perpendicular to both lines V and
(a) symmetric
W; line X is perpendicular to line V. [2015-I]
(b) skewed to the right
(c) neither symmetric nor skewed Which one of the following statements is correct?
(d) skewed to the left (a) Z, U and W are parallel.
54. The age of Mr. X last year was the square of a number (b) X, V and Y are parallel.
and it would be the cube of a number next year. (c) Z, V and U are all perpendicular to W.
What is the least number of years he must wait for (d) Y, V and W are parallel.
EBD_7711
52
63. A cylindrical overhead tank of radius 2 m and height candidates called for the interview, only two are
7 m is to be filled from an underground tank of size 5.5 eligible for the post of Principal while they all are
m × 4 m × 6 m. How much portion of the underground eligible for the post of Vice-Principal. The number of
tank is still filled with water after filling the overhead possible combinations of selectees is [2015-II]
tank completely? [2016-II] (a) 4 (b) 12
(a) 1/3 (b) 1/2 (c) 18 (d) None of the above
(c) 1/4 (d) 1/6 72. A student has to opt for 2 subjects out of 5 subjects for
64. A piece of tin is in the form of a rectangle having a course, namely, Commerce, Economics, Statistics,
length 12 cm and width 8 cm. This is used to Mathematics I and Mathematics II. Mathematics II can
construct a closed cube. The side of the cube is: be offered only if Mathematics I is also opted. The
(a) 2 cm (b) 3 cm [2016-II] number of different combinations of two subjects
(c) 4 cm (d) 7 cm which can be opted is [2015-II]
65. The outer surface of a 4 cm × 4 cm × 4 cm cube is (a) 5 (b) 6
painted completely in red. It is sliced parallel to the (c) 7 (d) 8
faces to yield sixty four 1cm × 1cm × 1cm small 73. A person ordered 5 pairs of black socks and some pairs
cubes. How many small cubes do not have painted of brown socks. The price of a black pair was thrice
faces? [2017-II] that of a brown pair. While preparing the bill, the bill
(a) 8 (b) 16 clerk interchanged the number of black and brown
(c) 24 (d) 36 pairs by mistake which increased the bill by 100%.
66. A group of 630 children is seated in rows for a group What was the number of pairs of brown socks in the
photo session. Each row contains three less children original order? [2015-II]
than the row in front of it.Which one of the following (a) 10 (b) 15
number of rows is not possible ? [2014 - II] (c) 20 (d) 25
(a) 3 (b) 4 74. The number of persons who read magazine X only is
(c) 5 (d) 6 thrice the number of persons who read magazine Y.
67. Twelve people form a club. By picking lots, one of them will The number of persons who read magazine Y only is
host a dinner for all once in a month. The number of dinners thrice the number of persons who read magazine X.
a particular member has to host in one year is [2015-II] Then, which of the following conclusions can be
drawn? [2015-II]
(a) One (b) Zero
1. The number of persons who read both the
(c) Three (d) Cannot be predicted
magazines is twice the number of persons who
68. There are 5 tasks and 5 persons. Task-l cannot be
read only magazine X.
assigned to either person-l or person-2. Task-2 must
2. The total number of persons who read either one
be assigned to either person-3 or person-4. Every
magazine or both the magazines is twice the
person is to be assigned one task. In how many ways
number of persons who read both the magazines.
can the assignment be done? [2015-II]
Select the correct answer using the code given below:
(a) 6 (b) 12 (a) 1 only (b) 2 only
(c) 24 (d) 144 (c) Both 1 and 2 (d) Neither 1 nor 2
69. In a society it is customary for friends of the same sex 75. Four-digit numbers are to be formed using the digits
to hug and for friends of opposite sex to shake hands 1, 2, 3 and 4; and none of these four digits are repeated
when they meet. A group of friends met in a party in any manner. Further, [2016-II]
and there were 24 handshakes. 1. 2 and 3 are not to immediately follow each other
Which one among the following numbers indicates 2. 1 is not to be immediately followed by 3
the possible number of hugs? [2015-II] 3. 4 is not to appear at the last place
(a) 39 (b) 30 4. 1 is not to appear at the first place
(c) 21 (d) 20 How many different numbers can be formed?
70. In a box of marbles, there are three less white marbles (a) 6 (b) 8
than the red ones and five more white marbles than (c) 9 (d) None of the above
the green ones. If there are a total of 10 white marbles, 76. A bag contains 20 balls. 8 balls are green, 7 are
how many marbles are there in the box? [2015-II] white and 5 are red. What is the minimum number
(a) 26 (b) 28 of balls that must be picked up from the bag
(c) 32 (d) 36 blindfolded (without replacing any of it) to be assured
71. A selection is to be made for one post of Principal and of picking at least one ball of each colour? [2017-II]
two posts of Vice-Princ ipal. Amongst the six
53
(a) 17 (b) 16 [2015-II]
(c) 13 (d) 11 (a) 45 km/hr (b) 48 km/hr
77. If 2 boys and 2 girls are to be arranged in a row so (c) 50 km/hr (d) 55 km/hr
that the girls are not next to each other, how many 84. Two pipes A and B can independently fill a tank
possible arrangements are there? [2017-II] completely in 20 and 30 minutes respectively. If both
(a) 3 (b) 6 the pipes are opened simultaneously, how much time
(c) 12 (d) 24 will they take to fill the tank completely? [2015-II]
78. Location of B is north of A and location of C is east of A. (a) 10 minutes (b) 12 minutes
The distances AB and AC are 5 km and 12 km (c) 15 minutes (d) 25 minutes
respectively. The shortest distance (in km) between 85. A and B walk around a circular park. They start at 8
the locations B and C is [2014 - II] a.m. from the same point in the opposite directions. A
(a) 60 (b) 13 and B walk at a speed of 2 rounds per hour and 3
(c) 17 (d) 7 rounds per hour respectively. How many times shall
79. Two cars start towards each other, from two places they cross each other after 8·00 a.m. and before 9.30.
A and B which are at a distance of 160 km. They start a.m.? [2016-II]
at the same time 08 :10 AM. If the speeds of the cars (a) 7 (b) 6
are 50 km and 30 km per hour respectively, they will
(c) 5 (d) 8
meet each other at [2014 - II]
(a) 10 :10 AM (b) 10 :30 AM 86. Four friends A, B, C and D need to cross a bridge. A
(c) 11:10 AM (d) 11:20 AM maximum of two persons can cross it at a time. It is
80. A straight line segment is 36 cm long. Points are to be night and they just have one lamp. Persons that cross
marked on the line from both the end points. From the bridge must carry the lamp to find the way. A
each end, the first point is at a distance of 1cm from pair must walk together at the speed of slower
the end, the second point is at a distance of 2 cm from person. After crossing the bridge, the person having
the first point and the third point is at a distance of 3 faster speed in the pair will return with the lamp
cm from the second point and so on. If the points on each time to accompany another person in the group.
the ends are not counted and the common points are Finally, the lamp has to be returned at the original
place and the person who returns the lamp has to
counted as one, what is the number of points ?
cross the bridge again without lamp. To cross the
[2014 - II]
bridge, the time taken by them is as follows :
(a) 10 (b) 12
A : 1 minute, B : 2 minutes, C : 7 minutes and D : 10
(c) 14 (d) 16 minutes. What is the total minimum time required
81. A and B decide to travel from place X to place Y by by all the friends to cross the bridge? [2016-II]
bus. A has ` 10 with him and he finds that it is 80% of
the bus fare for two persons. B finds that he has ` 3 (a) 23 minutes (b) 22 minutes
with him and hands it over to A. In this context, which (c) 21 minutes (d) 20 minutes
one of the following statements is correct ? [2014 - II]
87. A daily train is to be introduced between station A
(a) Now the money A has is just enough to buy two
and station B starting from each at 6 AM and the
tickets.
journey is to be completed in 42 hours. What is the
(b) A stillneeds ` 2 for buying the tickets. number of trains needed in order to maintain the
(c) After buying the two tickets A will be left with Shuttle Service? [2016-II]
50 paise.
(d) The money A now has is still not sufficient to buy (a) 2 (b) 3
two tickets. (c) 4 (d) 7
82. In a 500 metres race, B starts 45 metres ahead of A, 88. A freight train left Delhi for Mumbai at an average
but A wins the race while B is still 35 metres behind. speed of 40 km/hr. Two hours later, an express train
What is the ratio of the speeds of A to B assuming that left Delhi for Mumbai, following the freight train on
both start at the same time? [2015-II] a parallel track at an average speed of 60 km/hr.
(a) 25: 21 (b) 25: 20 How far from Delhi would the express train meet
(c) 5:3 (d) 5:7 the freight train? [2017-II]
83. Two cities A and B are 360 km apart. A car goes from A (a) 480 km (b) 260 km
to B with a speed of 40 km/hr and returns to A with a
speed of 60 km/hr. What is the average speed of the car? (c) 240 km (d) 120 km
EBD_7711
54
Directions for the following 4 (four) items : The following
Year 1992 1993 1994 1995
graph shows the average profit of two fruit-sellers A and B in
thousands (`) per year from the year 1995 to 2000. Consider the Population
20 21 22 23
graph and answer the 4 (four) items that follow : in lakhs
Income in
1010 1111 1225 1345
crores (`)
Y
Which one of the following statements is correct
6 inrespect of the above data ?
(a) Population increased by 5% or more every year.
5
B (b) Income increased by 10% or more every year.
4 (c) Per capita income was always above ` 5,000.
3 A (d) Per capita income was highest in 1994.
2 95. An automobiles owner reduced his monthly petrol
consumption when the prices went up. The price-
1
consumption relationship is as follows: [2015 - II]
0 X Price (in ` Per litre) 40 50 60 75
1995 1996 1997 1998 1999 2000
Monthly consumption (in litres) 60 48 40 30
Year
If the price goes up to ` 80 per litre, his expected
consumption (in litres) will be
89. In which year is the average profit of A and B same ? (a) 30 (b) 28
[2014 - II] (c) 26 (d) 24
(a) 1995 (b) 1996 96. The graph below depicts the earnings of A and B over
(c) 1997 (d) 1998 the period 2000 to 2010: [2015 - II]
90. What is the difference between the average profit of B
and A in the year 1998 ? [2014 - II]
Earnings

(a) – ` 100 (b) – `1,000 A


(c) + ` 600 (d) – ` 300
91. How much more average profit did A make in the B
year 2000 than in the year 1999 ? [2014 - II]
(a) ` 200 (b) ` 1,000
2000 2007 2010
(c) ` 1,500 (d) ` 2,000 Years
92. What is the trend of the average profit of B from the From the graph, which one of the following can be
year 1997 to the year 2000 ? [2014 - II] concluded?
Consider the following matrix : (a) On the average A earned more than B during this
(a) Non-increasing (b) Non-decreasing period.
(b) On the average B earned more than A during this
(c) Steady (d) Fluctuating period.
93. The following table shows the marks obtained by two (c) The earnings of A and B were equal during this
students in different subjects : [2014 - II] period.
(d) The earnings of A were less as compared to B
Student Maximum Student Maximum during this period.
A Marks B Marks 97. Year-wise variation of the pric e of a certain
English 60 100 80 150 commodity is shown in the following graph:
[2015 - II]
Paychology 70 100 70 100
History 50 100 60 100
Sanskrit 30 50 15 25

The difference in the mean aggregate percentage


marks of the students is
(a) 2.5 % (b) 13.75%
(c) 1.25% (d) Zero
94. The following table gives population and total income
of a city for four years : [2014 - II] The price of the commodity in the year 1990
55
(a) must have been ` 10/- (a) – 9 and 1 (b) – 9 and – 1
(b) must have been ` 12/- (c) 0 and 8 (d) 0 and 9
(c) must have been any where between ` 10/- and ` 20/-
103. A shopkeeper sells an article at ` 40 and gets X% profit.
(d) is higher than that in the year 1991
However, when he sells it at ` 20, he faces same percentage
98. The proportion of expenditure on various items by
of loss. What is the original cost of the article?
two families A and B are represented in the following
[Prelim II-2018]
Bar Charts:
(a) ` 10 (b) ` 20
[2015 - II]
(c) ` 30 (d) ` 40
104. A person bought a refrigerater worth ` 22,800 with 12.5%
interest compounded yearly. At the end of first year he
paid ` 8,650 and at the end of second year ` 9,125. How
much will he have to pay at the end of third year to clear
the debt? [Prelim II-2018]
(a) ` 9,990 (b) ` 10,000
(c) ` 10,590 (d) ` 11,250
105. A lift has the capacity of 18 adults or 30 children. How
many children can board the lift with 12 adults?
[Prelim II-2018]
(a) 6 (b) 10 (c) 12 (d) 15
106 A train 200 metres long is moving at the rate of 40 kmph.
In how many seconds will it cross a man standing near the
railway line? [Prelim II-2018]
(a) 12 (b) 15
From these charts, we can conclude that (c) 16 (d) 18
(a) Family A spent more money on food than Family B. 107. Two persons, A and B are running on a circular track. At
(b) Family B spent more money on food than Family A. the start, B is ahead of A and their positions make an angle
(c) Family A and Family B spent the same amount of 30° at the centre of the circle. When A reaches the point
on food. diametrically opposite to his starting point, he meets B.
(d) The expenditure on food by Family A and Family What is the ratio of speeds of A and B, if they are running
B cannot be compared. with uniform speeds? [Prelim II-2018]
99. A bookseller sold ‘a’ number of Geography textbooks at (a) 6 : 5 (b) 4 : 3
the rate of ` x per book, ‘a + 2’ number of History textbooks (c) 6: 1 (d) 4 : 2
at the rate of ` (x + 2) per book and ‘a – 2’ number of 108. Twelve equal squares are placed to fit in a rectangle of
mathematics textbooks at the rate of ` (x –2) per book. diagonal 5 cm. There are three rows containing four squares
What is his total sale in `? [Prelim II-2018] each. No gaps are left between adjacent squares. What is
(a) 3x + 3a (b) 3ax + 8 the area of each square? [Prelim II-2018]
(c) 9ax (d) x 3 a3 5 7
100. A student has to get 40% marks to pass in an examination. (a) sq. cm (b) sq. cm
7 5
Suppose he gets 30 marks and fails by 30 marks, then what
are the maximum marks in the examination? 25
(c) 1 sq. cm (d) sq. cm
[Prelim II-2018] 12
(a) 100 (b) 120 109. 19 boys turn out for playing hockey. Of these, 11 are
(c) 150 (d) 300 wearing hockey shirts and 14 are wearing hockey pants.
101. If x – y = 8, then which of the following must be true? There are no boys without shirts and/or pants. What is
[Prelim II-2018] the number of boys wearing full uniform?
1. Both x and y must be positive for any value of x and y. [Prelim II-2018]
(a) 3 (b) 5
2. If x is positive, y must be negative for any value of x and y. (c) 6 (d) 8
3. If x is negative, y must be positive for any value of x and y. 110. For a sports meet, a winners’ stand comprising three
Select the correct answer using the code given below. wooden blocks is in the following form :
(a) 1 only (b) 2 only
(c) Both 1 and 2 (d) Neither 1 nor 2 nor 3
102. If X is between – 3 and – 1, and Y is between – 1 and 1,
then X 2 - Y 2 is in between which of the following?
[Prelim II-2018] There are six different colours available to choose from
EBD_7711
56
and each of the three wooden blocks is to be painted such State 14 864 5.46 97.9
that no two of them has the same colour. In how many State 15 497 7.48 62.3
different ways can the winners’ stand be painted? State 16 777 7.03 93.8
[Prelim II-2018] State 17 335 5.8 49.9
(a) 120 (b) 81 State 18 599 7.49 47.84
(c) 66 (d) 36 115. With reference to the above table, which of the following
111. There are 24 equally spaced points lying on the is/are the most logical and rational inference/inferences
circumference of a circle. What is the maximum number of that can be made?
equilateral triangles that can be drawn by taking sets of 1. Higher per capita income is generally associated with
three points as the vertices? [Prelim II-2018] higher Tele-density.
(a) 4 (b) 6 2. Higher GDP growth rate always ensures higher per
(c) 8 (d) 12 capita income.
112. How many diagonals can be drawn by joining the vertices 3. Higher GDP growth rate does not necessarily ensure
of an octagon? [Prelim II-2018] higher Tele-density.
(a) 20 (b) 24 Select the correct answer using the code given below.
(c) 28 (d) 64 [Prelim II-2018]
113 . While writing all the numbers from 700 to 1000, how many (a) 1 only (e) 2 and 3
numbers occur in which the digit at hundred’s place is (b) 1 and 3 (d) 3 only
greater than the digit at ten’s place, and the digit at ten’s 116. With reference to the above table, the following
place is greater than the digit at unit’s place? assumptions have been made :
[Prelim II-2018] 1. Nowadays, prosperity of an already high performing
(a) 61 (b) 64 State cannot be sustained without making further
(c) 85 (d) 91 large investments in its telecom infrastructure.
114. A bag contains 15 red balls and 20 black balls. Each ball 2. Nowadays, a very high Tele-density is the most
is numbered either 1or 2 or 3. 20% of the red balls are essential condition for promoting the business and
numbered 1 and 40% of them are numbered 2. Similarly, economic growth in a State.
among the black balls, 45% are numbered 2 and 30% are Which of the above assumptions is/are valid?
numbered 3. A boy picks a ball at random. He wins if the [Prelim II-2018]
ball is red and numbered 3 or if it is black and numbered (a) 1 only (b) 2 only
1 or 2. What are the chances of his winning? (c) Both 1 and 2 (d) Neither 1 nor 2
[Prelim II-2018]
1 4
(a) (b) Directions (Qs. 117-119) : Study the bar graph and answer the
2 7
questions.
5 12 The following three items are based on the graph given below
(c) (d)
9 13 which shows imports of three different types of steel over a
Directions (Qs. 115-116) : Study the table carefully and answer period of six months of a year. Study the graph and answer the
the question. three items that follow.
The following table gives the GDP growth rate and Tele-density
data of different States of a country in a particular year. Study 44
Thousands of tons imported

the table and answer the two items that follow. 42


40
States Per capita GDP growth Tele-density 38
income ($) rate (%) 36
34
State 1 704 9.52 70.27 32
State 2 419 5.31 35.88 30
State 3 254 10.83 50.07 28
26
State 4 545 9.78 5.94 24
State 5 891 10.8 76.12 22
State 6 1077 11.69 77.5
January February March April May June
State 7 900 8.88 104.86
Coil ($ 320) Sheet ($ 256) Scrap ($ 175)
State 8 395 5.92 6
State 9 720 7.76 82.25
State 10 893 9.55 96.7 The figures in the brackets indicate the average cost per
State 11 363 4.7 57.7 ton over six months period.
State 12 966 7.85 63.8 117. By how much (measured in thousands of tons) did the
State 13 495 9.37 52.3 import of sheet steel exceed the import of coil steel in the
57
first three months of the year? [Prelim II-2018] (a) population growth rate has increased
(a) 11 (b) 15 (b) population growth rate has decreased
(c) 19 (d) 23 (c) growth rate of population has remained stable
118. What was the approximate total value (in $) of sheet steel (d) population growth rate shows no trend
imported over the six months period? [Prelim II-2018] 122. With reference to the above graph, consider the following
(a) 45,555 (b) 50,555 statements considering 1970 as base year :
(c) 55,550 (d) 65,750 1. Population has stabilized after 35 years.
119. What was the approximate ratio of sheet steel and scrap 2. Population growth rate has stabilized after 35 years.
steel imports in the first three months of the year? 3. Death rate has fallen by 10% in the first 10 years.
[Prelim II-2018] 4. Birth rate has stabilized after 35 years.
(a) 1 : 1 (b) 1.2 : 1 Which of the above are the most logical and rational
(c) 1.4 : 1 (d) 1.6 : 1 statements that can be made from the above graph?
120. Consider the following graph : [Prelim II-2018]
(a) 1 and 2 only (b) 1, 2 and 3
(c) 3 and 4 (d) 2 and 4
100%
Expected 123. Average hourly earnings per year (E) of the workers in a
75% Progress
firm are represented in figures A and B as follows:
[Prelim II-2018]
% of work

50%
Actual
E
25% Progress

Hourly earnings
0%
1st April May June July Aug 1st Sep. 40
35
Which one of the following statements is not correct with
reference to the graph given above? 30
[Prelim II-2018]
25
(a) On 1st June, the actual progress of work was less
than expected. 20
(b) The actual rate of progress of work was the greatest 2013 14 15 16 17 Y
during the month of August. Years
(c) The work was actually completed before the expected time. Fig. A
(d) During the period from 1st April to 1st September, at
no time was the actual progress more than the E
expected progress.
Directions for the following 2 (two) items : 40
Consider the following graph in which the birth rate and
death rate of a country are given, and answer the two
Hourly earnings

items that follow. 30

30
Birth and death rates per thousand

20

Birth rate
20 10

Death rate
2013 14 15 16 17 Y
10
Years
Fig. B
From the figures, it is observed that the
0 (a) values of E are different
1970 1980 1990 2000 2010
(b) ranges (i.e., the difference between the maximum
Years and the minimum) of E are different
(c) slopes of the graphs are same
121. Looking at the graph, it can be inferred that from 1990 to (d) rates of increase of E are different
2010 [Prelim II-2018]
EBD_7711
58
124. Consider the following figures A and B :
8 Country B

Population (in millions)


1.2
7 1.0
Cost of production

0.8
(` in lakhs)

6
0.6
5 0.4
0.2
4
0
1950 1970 1990 2010 2030 2050
Years
1000 2000 3000
< 15 15 – 64 64 +
No. of pieces manufactured
Fig. A with reference to the above graphs, which of the following
are the most logical and rational inferences that can be
made?
Selling price per piece

500 1. Over the last two and a half decades, the dependency
ratio for country B has decreased.
450 2. By the end of next two and a half decades, the
dependency ratio of country A will be much less than
400 that of country B.
(` )

3. In the next two decades, the work-force relative to


350
its total population will increase in country B
300 compared to country A.
Select the correct answer using the code given below.
[Prelim II-2018]
1000 2000 3000 (a) 1 and 2 only (b) 2 and 3 only
No. of pieces sold (c) 1 and 3 only (d) 1, 2 and 3
Fig. B 126. The graph given below indicates the changes in key policy
The manufacturing cost and projected sales for a product rates made by the Central Bank several times in a year :
are shown in the above figures A and B respectively.
What is the minimum number of pieces
that should be manufactured to avoid a loss? Key policy rates in % 7.50
[Prelim II-2018] (+.25)
(a) 2000 (b) 2500 (c) 3000 (d) 3500 7.50 7.25
125. Consider the following graphs. The curves in the graphs 6.50
7.00 Repo Rate
indicate different age groups in the populations of two (+.25)
countries A and B over a period of few decades: 6.50 6.25
6.00 CRR
6.00
6.00 6.00
Country A 5.50 (0)
Population (in millions)

1.2 5.50 Reverse Repo Rate


1.0 5.00
0.8 4.50
0.6 4.00
0.4 4.00
3.50
0.2
3.00
0 Jul 2 Jul 27 Sep 16 Nov 2 Dec 16 Jan 25 Mar 17 May 3 Jun 16
1950 1970 1990 2010 2030 2050 2010 2011
Years

< 15 15 – 64 64 + Which one of the following can be the most likely reason
for the Central Bank for such an action?
59
[Prelim II-2018]
(a) Encouraging foreign investment
COMPREHENSION, DECISION
(b) Increasing the liquidity MAKING & INTERPERSONAL SKILLS
(c) Encouraging both public and private savings Directions (For Passages 1-25) : Read the passages and answer
(d) Anti-inflationary stance
the question based on them.
127. The following graph indicates the composition of our tax
revenue for a period of two decades :
PASSAGE - 1
Per cent of gross tax revenue

45
40 It is essential that we mitigate the emissions of greenhouse gases
35
30
and thus avoid some of the worst impacts of climate change that
25 would take place in coming years and decades. Mitigation would
20 require a major shift in the way we produce and consume energy.
15 A shift away from overwhelming dependence on fossil fuels is
10
5
now long overdue, but unfortunately, technological development
0 has been slow and inadequate largely because government policies
have not promoted investments in research and development,
1990-91

1995-96

2003-04

2004-05

2005-06

2006-07

2007-08

2008-09
2009-10

2010-11
(Prov)

(BE)
myopically as a result of relatively low prices of oil. It is now,
therefore, imperative for a country like India treating the opportunity
Years of harnessing renewable energy on a large scale as a national
imperative. This country is extremely well endowed with solar,
Excise Customs Corporate Personal Service wind and biomass sources of energy. Where we have lagged,
tax income tax tax unfortunately, is in our ability to develop and to create technological
With reference to the above graph, which of the following
is/are the most logical and rational inference/ inferences solutions for harnessing these resources.
that can be made? One particular trajectory for carryingout stringent mitigation of
1. During the given period, the revenue from Direct greenhouse gas emissions assessed by the Intergovernmental Panel
Taxes as percentage of gross tax revenue has on Climate Change (IPCC) clearly shows the need for ensuring that
global emissions of greenhouse gases peak no later than 2015 and
increased while that of Indirect Taxes decreased.
reduce rapidly thereafter. The cost associated with such a trajectory
2. The trend in the revenue from Excise Duty
is truly modest and would amount, in the estimation of IPCC, to not
demonstrates that the growth of manufacturing sector
more than 3 percent of the global GDP in 2030. In other words, the
has been negative during the given period.
level of prosperity that the world would have reached without
Select the correct answer using the code given below. mitigation would at worst be postponed by a few months or a year
[Prelim II-2018] at the most. This is clearly not a very high price to pay for protecting
(a) 1 only (b) 2 only hundreds of millions of people from the worst risks associated with
(c) Both 1 and 2 (d) Neither 1 nor 2 climate change. Any such effort, however, would require lifestyles
128. The figure drawn below gives the velocity graphs of two to change appropriately also. Mitigation of greenhouse gas
vehicles A and B. The stright line OKP represents the emissions is not a mere technological fix, and clearly requires
velocity of vehicle A at any instant, whereas the horizontal changes in lifestyles and transformation of a country's economic
straight line. CKD represents the velocity of vehicle B at structure, whereby effective reduction in emissions is brought about,
any instant. In the figure, D is the point where such as through the consumption of much lower quantities of animal
perpendicular from P meets the horizontal line CKD such protein. The Food and Agriculture
1 Organization (FAO) has determined that the emissions from the
that PD = LD : livestock sector amount to 18 percent of the total. The reduction of
2 emissions from this source is entirely in the hands of human beings,
P who have never questioned the impacts that their dietary habits of
consuming more and more animal protein are bringing about.
Mitigation overall has huge co-benefits, such as lower air pollution
C Vehicle B K D and health benefits, higher energy security and greater employment,
Velocity

[Prelims II- 2014]


le A 129. According to the passage, which of the following would
e hic
V help in the mitigation of greenhouse gases ?
1. Reducing the consumption of meat
2. Rapid economic liberalization
O Time L
What is the ratio between the distances covered by vehicles 3. Reducingthe consumerism
A and B in the time interval OL? 4. Modern management practices of livestock
[Prelim II-2018] Select the correct answer using the code given below :
(a) 1 : 2 (b) 2:3 (a) 1,2 and 3 (b) 2, 3 and 4
(c) 3 : 4 (d) 1:1 (c) 1 and 3 only (d) 2 and 4 only
EBD_7711
60
130. Why do we continue to depend on the fossil fuels the poorest sections of the population. It also ensures that those
heavily? who are outside of the bottom quintile do not get ignored. If that
1. Inadequate technological development were done, then those people would in all likelihood drop down
2. Inadequate funds for research and development into the bottom quintile and so would automatically become a
3. Inadequate availability of alternative sources of energy direct target of our policies. Hence the criterion being suggested
Select the correct answer using the code given below : here is a statistical summing up of the idea of inclusive growth,
(a) 1only (b) 2 and 3 only which, in turn, leads to two corollaries : to wish that India must
(c) 1and 3 only (d) 1,2 and 3 strive to achieve high growth and that we must work to ensure
131. According to the passage, how does the mitigation of that the weakest segments benefit from the growth.
greenhouse gases help us ? [Prelims II- 2014]
1. Reduces expenditure on public health 133. The author's central focus is on.
2. Reduces dependence on livestock (a) applauding India's economic growth not only against
3. Reduces energy requirements its own past performance, but against other nations.
4. Reduces rate of global climate change (b) emphasizing the need for economic growth which is
Select the correct answer using the code given below : the sole determinant of a country's prosperity.
(a) 1,2 and 3 (b) 1, 3 and 4 (c) emphasizing inclusive growth where gains of growth
(c) 2, 3 and 4 (d) 1and 4 only are shared widely by the population.
132 What is the essential message of the passage ? (d) emphasizing high growth.
(a) We continue to depend on fossil fuels heavily 134. The author supports policies which will help
(b) Mitigation of the greenhouse gases is imperative (a) develop economic growth.
(c) We must invest in research and development (b) better distribution of incomes irrespective of rate of growth.
(d) People must change their lifestyle (c) develop economic growth and redistribute economic
gains to those getting left behind.
PASSAGE - 2 (d) put an emphasis on the development of the poorest
segments of society.
In recent times, India has grown fast not only compared to its own
135. Consider the following statements :
past but also incomparison with other nations. But there cannot
According to the author, India's economy has grown but
be any room for complacency because it is possible for the Indian
there is no room for complacency as
economy to develop even faster and also to spread the benefits
1. growth eradicates poverty.
of this growth more widely than has been done thus far. Before
2. growth has resulted in prosperity for all.
going into details of the kinds of micro-structural changes that we
Which of the statements given above is/are correct ?
need to conceptualize and then proceed to implement, it is
(a) 1only (b) 2 only
worthwhile elaborating on the idea of inclusive growth that
constitutes the defining concept behind this Government's various (c) Both 1and 2 (d) Neither 1nor 2
economic policies and decisions. A nation interested in inclusive
growth views the same growth differently depending on whether PASSAGE - 3
the gains of the growth are heaped primarily on a small segment or It is easy for the government to control State-owned companies
shared widely by the population. The latter is cause for celebration through nods and winks. So what really needs to be done as a first
but not the former. In other words, growth must not be treated as step is to put petrol pricing on a transparent formula - if the price
an end in itself but as an instrument for spreading prosperity to of crude is x and the exchange rate y, then every month or fortnight,
all. India's own past experience and the experience of other nations the government announces a maximum price of petrol, which
suggests that growth is necessary for eradicating poverty but it anybody can work out from the x and the y. The rule has to be
is not a sufficient condition. In other words, policies for promoting worked out to make sure that the oil-marketing companies can, in
growth need to be complemented with policies to ensure that general, cover their costs. This will mean that if one company can
more and more people join in the growth process and, further, that innovate and cut costs, it will make greater profits. Hence, firms
there are mechanisms inplace to redistribute some of the gains to will be more prone to innovate and be efficient under this system.
those who are unable to partake in the market process and, hence, Once the rule is announced, there should be no interference by
get left behind. the government. If this is done for a while, private companies will
A simple way of giving this idea of inclusive growth a sharper re-enter this market. And once a sufficient number of them are in
form is to measure a nation's progress in terms of the progress of the fray, we can remove the rule-based pricing and leave it truly to
its poorest segment, for instance the bottom 20 per cent of the the market (subject to, of course, the usual regulations of anti-
population. One could measure the per capita income of the bottom trust and other competition laws). [Prelims II- 2014]
quintile of the population and also calculate the growth rate of 136. Consider the following statements :
income; and evaluate our economic success in terms of these According to the passage, an oil company can make greater
measures that pertain to the poorest segment. This approach is profits, if a transparent formula for petrol pricing is announced
attractive because it does not ignore growth like some of the older every fortnight or month, by
heterodox criteria did. It simply looks at the growth of income of 1. promoting its sales.
61
2. undertaking innovation. (a) 1only (b) 2 only
3. cutting costs. (c) Both 1and 2 (d) Neither 1nor 2
4. selling its equity shares at higher prices. 139. Which one of the following statements best implies the need
Which of the statements given above is/are correct ? to shift toward contemporary conservation approach?
(a) 1only (b) 2 and 3 (a) Exploitation of natural resources causes a stress on the
(c) 3 and 4 (d) 1, 2 and 4 Himalayan ecosystem.
137. Consider the following statements : (b) Climate change alters precipitation patterns, causes
According to the passage, private oil companies re-enter episodes of drought and biotic interference.
the oil producing market if (c) The rich biodiversity, including endemic diversity,
1. a transparent rule-based petrol pricing exists. makes the Himalayan region a biodiversity hotspot.
2. there is no government interference in the oil producing (d) The Himalayan biogeographic region should be enabled
market. to adapt to climate change smoothly.
3. subsidies are given by the government 140. What is the most important message conveyed by the
4. regulations of anti-trust are removed. passage ?
Which of the statements given above are correct ? (a) Endemism is a characteristic feature of Himalayan region.
(a) 1and 2 (b) 2 and 3 (b) Conservation efforts should emphasize on biogeographic
(c) 3 and 4 (d) 2 and 4 ranges rather than on some species or habitats.
(c) Climate change has adverse impact on the Himalayan
PASSAGE - 4 ecosystem.
(d) Without Himalayan ecosystem, the life of the
The Himalayan ecosystem is highly vulnerable to damage, both communities of uplands and downstreams will have no
due to geological reasons and on account of the stress caused by sustenance.
increased pressure of population, exploitation of natural resources 141. With reference to the passage, the following assumptions
and other related challenges. These aspects may be exacerbated have been made :
due to the impact of climate change. It is possible that climate 1. To maintain natural ecosystems, exploitation of natural
change may adversely impact the Himalayan ecosystem through resources should be completely avoided.
increased temperature, altered precipitation patterns, episodes of 2. Not only anthropogenic but also natural reasons can
drought and biotic influences. This would not only impact the adversely affect ecosystems.
very sustenance of the indigenous communities in uplands but 3. Loss of endemic diversity leads to the extinction of
also the life of downstream dwellers across the country and ecosystems.
beyond. Therefore, there is an urgent need for giving special Which of the above assumptions is/are correct ?
attention to sustain the Himalayan ecosystem. This would require (a) 1and 2 (b) 2 only
conscious efforts for conserving all the representative systems. (c) 2 and 3 (d) 3 only
Further, it needs to be emphasized that the endemics with restricted
distribution, and most often with specialized habitat requirements, PASSAGE - 5
are among the most vulnerable elements. In this respect the
Himalayan biodiversity hotspot, with rich endemic diversity, is It is often forgotten that globalization is not only about policies
vulnerable to climate change. The threats include possible loss of on international economic relationships and transactions, but has
genetic resources and species, habitats and concomitantly a decrease equally to do with domestic policies of a nation. Policy changes
in ecosystem services. Therefore, conservation of endemic elements necessitated by meeting the internationally set conditions (by
in representative ecosystems/habitats assumes a great significance WTO etc.) of free trade and investment flows obviously affect
while drawing conservation plans for the region. domestic producers and investors. But the basic philosophy
underlying globalization emphasizes absolute freedom to markets
Towards achieving the above, we will have to shift toward
to determine prices and production and distribution patterns, and
contemporary conservation approaches, which include a paradigm
view government interventions as processes that create distortions
of landscape level interconnectivity between protected area
and bring in inefficiency. Thus, public enterprises have to be
systems. The concept advocates a shift from the species-habitat
privatized through disinvestments and sales; sectors and activities
focus to an inclusive focus on expanding the biogeographic range
hitherto reserved for the public sector have to be opened to the
so that natural adjustments to climate change can proceed without
private sector. This logic extends to the social services like
being restrictive. [Prelims II- 2014]
education and health. Any restrictions on the adjustments in
138. Consider the following statements : workforce by way of retrenchment of workers should also be
According to the passage, the adverse impact of climate removed and exit should be made easier by removing any
change on an ecosystem can be a restrictions on closures. Employment and wages should be
1. permanent disappearance of some of its flora and governed by free play of market forces, as any measure to regulate
fauna. them can discourage investment and also create inefficiency in
2. permanent disappearance of ecosystem itself. production. Above all, in line with the overall philosophy of
Which of the statements given above is/are correct ? reduction in the role of the State, fiscal reforms should be
EBD_7711
62
undertaken to have generally low levels of taxation and government
PASSAGE - 7
expenditure should be kept to the minimum to abide by the principle
of fiscal prudence. All these are policy actions on the domestic The conflict between man and State is as old as State history.
front and are not directly related to the core items of the Although attempts have been made for centuries to bring about a
globalization agenda, namely free international flow of goods and proper adjustment between the competing claims of State and the
finance. [Prelims II- 2014] individual, the solution seems to be' still far off. This is primarily
142. According to the passage, under the globalization, government because of the dynamic nature of human society where old values
interventions are viewed as processes leading to and ideas constantly yield place to new ones. It is obvious that if
(a) distortions and inefficiency in the economy. individuals are allowed to have absolute freedom of speech and
(b) optimum use of resources. action, the result would be chaos, ruin and anarchy.
(c) more profitability to industries. 147. The author's viewpoint can be best summed up in which of
(d) free play of market forces with regard to industries. the following statements? [CSAT 2015]
143. According to the passage, the basic philosophy of (a) The conflict between the claims of State and individual
globalization is to remains unresolved.
(a) give absolute freedom to producers to determine prices (b) Anarchy and chaos ar e the obvious results of
and production. democratic traditions.
(b) give freedom to producers to evolve distribution (c) Old values, ideas and traditions persist despite the
patterns. dynamic nature of human society.
(c) give absolute freedom to markets to determine prices, (d) Constitutional guarantee of freedom of speech is not in
production and employment. the interest of society.
(d) give freedom to producers to import and export.
144. According to the passage, which of the following is/are PASSAGE - 8
necessary for ensuring globalization ?
1. Privatization of public enterprises We generally talk about democracy but when it comes to any
2. Expansionary policy of public expenditure particular thing, we prefer a belonging to our caste or community
3. Free play of market forces to determine wages and or religion. So long as we have this kind of temptation, our
employment democracy will remain a phoney kind of democracy. We must be in
a position to respect a man as a man and to extend opportunities
4. Privatization of social services like education and health
for development to those who deserve them and not to those who
Select the correct answer using the code given below :
happen to belong to our community or race. This fact of
(a) 1only (b) 2 and 3 only
favouritism has been responsible for much discontent and ill-will
(c) 1, 3 and 4 (d) 2, 3 and 4
in our country.
145. According to the passage, in the process of globalization
148. Which one of the following statements best sums up the
the State should have
above passage? [CSAT 2015]
(a) expanding role. (b) reducing role.
(a) Our country has a lot of diversity with its many castes,
(c) statutory role. (d) none of the above roles.
communities and religions.
(b) True democracy could be established by providing
PASSAGE - 6
equal opportunities to all.
By 2050, the Earth's population will likely have swelled from seven (c) So far none of us have actually understood the meaning
to nine billion people. To fill all those stomachs - while accounting of democracy.
for shifting consumption patterns, climate change, and a finite (d) It will never be possible for us to establish truly
amount of arable land and potable water – some experts say food democratic governance in our country.
production will have to double. How can we make the numbers
add up? Experts say higher yielding crop varieties and more PASSAGE - 9
efficient farming methods will be crucial. So will waste reduction. The existence/establishment of formal financial institutions that offer
Experts urge cities to reclaim nutrients and water from waste streams safe, reliable and alternative financial instruments is fundamental in
and preserve farmland. Poor countries, they say, can improve crop mobilising savings. To save, individuals need access to safe and
storage and packaging and rich nations could cut back on resource- reliable financial institutions, such as banks, and to appropriate
intensive foods like meat. financial instruments and reasonable financial incentives. Such
146. Which one of the following statements best sums up the access is not always available to all people in developing countries
above passage? [CSAT 2015] like India and more so, in rural areas. Savings help poor households
(a) The population of the world is growing very fast. manage volatility in cash flow, smoothen consumption, and build
(b) Food security is a perennial problem only in developing working capital. Poor households without access to a formal savings
countries. mechanism encourage immediate spending temptations.
(c) The world does not have enough resources to meet the 149. With reference to the above passage, consider the following
impending food scarcity. statements:
(d) Food security is increasingly a collective challenge. 1. Indian financial institutions do not offer any financial
63
instruments to rural households to mobilise their (b) to eradicate hunger in underdeveloped countries
savings. (c) to achieve a balance between the production of food
2. Poor households tend to spend their earnings/savings and non-food items
due to lack of access to appropriate financial (d) to achieve a balance between demand for and supply
instruments. of food
Which of the statements given above is/are correct? 153. According to the above passage, which of the following
[CSAT 2015] helps/help in reducing hunger and starvation in the
(a) 1 only (b) 2 only. developing economies ?
(c) Both 1 and 2 (d) Neither 1 nor 2 1. Balancing demand and supply of food
150. What is the crucial message conveyed in the passage? 2. Increasing imports of food
(a) Establish more banks. [CSAT 2015] 3. Increasing purchasing power of the poor
(b) Increase the Gross Domestic Product (GDP) growth rate 4. Changing the food consumption patterns and
(c) Increase the interest rate of bank deposits practices
(d) Promote financial inclusion Select the correct answer using the code given below:
(a) 1 only (b) 2, 3 and 4 only
PASSAGE - 10 (c) 1 and 3 only (d) 1, 2, 3 and 4
154. The issue of worldwide supply of food has gained
Historically, the biggest challenge to world agriculture has been
importance mainly because of:
to achieve a balance between demand for and supply of food. At
1. overgrowth of the population worldwide
the level of individual countries, the demand-supply balance can
2. sharp decline in the area of food production
be a critical issue for a closed economy, especially if it is a
3. limitation in the capabilities for sustained supply of
populous economy and its domestic agriculture is not growing
food
sufficiently enough to ensure food supplies, on an enduring
Select the correct answer using the code given below:
basis; it is not so much and not always, of a constraint for an
(a) 1 and 2 only (b) 3 only
open, and growing economy, which has adequate exchange
(c) 2 and 3 only (d) 1, 2 and 3
surplues to buy food abroad. For the world as a whole, Spply-
demand balance is always an inescapable prerequisite for warding PASSAGE - 11
off hunger and starvation. However, global availability of
adequate supply does not necessarily mean that food would Accountability, or the lack of it, in governance generally, and civil
automatically move from countries of surplus to counteries of services, in particular, is a major factor underlying the deficiencies
deficit if the latter lack in purchasing power. The uneven in governance and public administration. Designing an effective
distribution of Hunger, starvation, under or malnourishment, etc., framework for accountability has been a key element of the reform
at the world-level, thus owes itself to the presence of empty-pock agenda. A fundamental issue is whether civil services should be
hungry mouths, overwhelmingly confined to the underdeveloped accountable to the political executive of the day or to society at
economies. Inasmuch as ‘a two-square meal’ is of elemental large. In other words, how should internal and external
significance to basic human existence, the issue of worldwide accountability be reconciled? Internal accountability is sought to
supply` of food has been gaining significance, in recent times, be achieved by internal performance monitoring, official
both because the quantum and the composition of demand has supervision by bodies like the–Central–Vigilance Commission–
been undergoing big changes, and because, in recent years, the and–Comptroller and Auditor– General, and judicial review of
capabilities individual countries to generate uninterrupted chain executive decisions. Articles 311 and 312 of the Indian Constitution
of food supplies have come under strain. Food production, provide job security and safeguards to the civil services, especially
marketing and prices, especially price-affordability by the poor in the All India Services. The framers of the Constitution had
the developing world, have become global issues that need global envisaged that provision of these safeguards would result in a civil
thinking and global solutions. service that is not totally subservient to the political executive but
151. According to the above passage, which of, the following are the will have the strength to function in larger public interest. The need
fundamental solutions for the world food security problem? to balance internal and external accountability is thus built into the
1. Setting up more agro-based industries Constitution. The issue is where to draw the line. Over the years,
2. Improving the price affordability by the poor the emphasis seems to have tilted in favour of greater internal
3. Regulating the conditions of marketing accountability of the civil services to the political leaders of the day
4. Providing food subsidy to one and all who in turn are expected to be externally accountable to the society
Select the correct answer using the code given below: at large through the election process. This system for seeking
(a) 1 and 2 (b) 2 and 3 only accountability to society has not worked out, and has led to several
(c) 1, 3 an 4 only (d) 1, 2, and 4 adverse consequences for governance.
152. According to the above passage, the biggest challenge to Some special measures can be considered for improving
world agriculture is: accountability in civil services. Provisions of articles 311 and 312
(a) to find sufficient land for agriculture and to expand should be reviewed and laws and regulations framed to ensure
food processing industries external accountability of civil services. The proposed Civil
EBD_7711
64
Services Bill seeks to address some of these requirements. The PASSAGE - 12
respective roles of professional civil services and the political
executive should be defined so that professional managerial In general, religious traditions stress our duty to god, or to some
functions and management of civil services are depoliticized. For universal ethical principle. Our duties to one another derive from
this purpose, effective statutory civil service boards should be these. The religious concept of rights is primarily derived from our
created at the centre and in the states. Decentralization and relationship to this divinity or principle and the implication it has
devolution of authority to bring government and decision making on our other relationships. This correspondence between rights
closer to the people also helps to enhance accountability. and duties is critical to any further understanding of justice. But,
155. According to the passage, which of the following factor/ for justice to be practiced; viture, rights and duties cannot remain
factors led to the adverse consequences for governance/ formal abstractions. They must be grounded in a community
public administration? (common unity) bound together by a sense of common union
1. Inability of civil services to strike a balance between (communion). Even as a personal virtue, this solidarity is
internal and external accountabilities essential to the practice and understanding of justice.
2. Lack of sufficient professional training to the officers 159. With reference to the passage, the following assumptions
of All India Services have been made :
3. Lack of proper service benefits in civil services 1. Human relationships are derived from their religious
traditions
4. Lack of Constitutional provisions to define the
2. Human beings can be duty bound only if they believe
respective roles of professional civil services vis-a-vis
in god
political executive in this context
3. Religious traditions are essential to practice and
Select the correct answer using the code given below:
understand justice
(a) 1 only (b) 2 and 3 only
Which of these assumption(s) is/are valid?
(c) 1 and 4 only (d) 2, 3 and 4 (a) 1 only (b) 2 and 3 only
156. With reference to the passage, the following assumptions (c) 1 and 3 only (d) 1, 2 and 3
have been made : 160. Which one of the following is the crux of this passage?
1. Political executive is an obstacle to the accountability (a) Our duties to one another derive from our religious
of the civil services to the society traditions
2. In the present framework of Indian polity, the political (b) Having relationship to the divine principle is a great
executive is no longer accountable to the society virtue
Which of these assumptions is/are valid? (c) Balance between rights and duties is crucial to the
(a) 1 only delivery of justice in a society
(b) 2 only (d) Religious concept of rights is primarily derived from
(c) Both 1 and 2 our relationship to god
(d) Neither 1 nor 2
157. Which one of the following is the essential message implied PASSAGE - 13
by this passage? Biomass as fuel for power, heat, and transport has the highest
(a) Civil services are not accountable to the society they mitigation potential of all renewable sources. It comes from
are serving agriculture and forest residues as well as from energy crops. The
(b) Educated and enlightened persons are not taking up biggest challenge in using biomass residues is a long-term reliable
political leadership supply delivered to the power plant at reasonable costs; the key
(c) The framers of the Constitution did not envisage the problems are logistical constraints and the costs of fuel
problems being encountered by the civil services collection. Energy crops, if not managed properly, compete with
(d) There is a need and scope for reforms to improve the food production and may have undesirable impacts on food
accountability of civil services prices. Biomass production is also sensitive to the physical
impacts of a changing climate.
158. According to the passage, which one of the following is not
Projections of the future role of biomass are probably
a means of enhancing internal accountability of civil
overestimated, given the limits to the sustainable biomass supply,
services? unless breakthrough technologies substantially increase
(a) Better job security and safeguards productivity. Climate -energy models project that biomass use
(b) Supervision by Central Vigilance Commission could increase nearly four-fold to around 150 – 200 exajoules,
(c) Judicial review of executive decisions almost a quarter of world primary energy in 2050. However the
(d) Seeking accountability through enhanced maximum sustainable technical potential of biomass resources
participation by people in decision making process (both residues and energy crops) without disruption of food and
forest resources ranges from 80 – 170 exajoules a year by 2050,
and only part of this is realistically and economically feasible. In
65
addition, some climate models rely on biomass-based carbon is a kind of implied anarchy; it is to refuse one's political
capture and storage, an unproven technology, to achieve negative responsibility while enjoying the benefits of a free political
emissions and to buy some time during the first half of the century. society.
Some liquid biofuels such as corn-based ethanol, mainly for 165. This passage relates to [Prelim II-2017]
transport, may aggravate rather than ameliorate carbon emissions on (a) duty to vote
a life-cycle basis. Second generation biofuels, based on ligno- (b) right to vote
cellulosic feedstocks – such as straw, bagasse, grass and wood – (c) freedom to vote
hold the promise of sustainable production that is high-yielding and (d) right to participate in politics
emit low levels of greenhouse gases, but these are still in the R & D
stage. PASSAGE - 15
161. What is/are the present constraint/constraints in using
biomass as fuel for power generation? In a free country, the man who reaches the position of leader
1. Lack of sustainable supply of biomass is usually one of outstanding character and ability. Moreover,
2. Biomass production competes with food production it is usually possible to foresee that he will reach such a position,
3. Bio-energy may not always be low carbon on a life- since early in life one can see his qualities of character. But this
cycle basis is not always true in the case of a dictator, often he reaches his
Select the correct answer using the code given below: position of power through chance, very often through the
(a) 1 and 2 only (b) 3 only unhappy state of his country.
(c) 2 and 3 only (d) 1, 2 and 3 166. The passage seems to suggest that [Prelim II-2017]
162. Which of the following can lead to food security problem? (a) a leader foresees his future position
1. Using agricultural and forest residues as feedstock for (b) a leader is chosen only by a free country
power generation (c) a leader must see that his country is free from despair
2. Using biomass for carbon capture and storage (d) despair in a country sometimes leads to dictatorship
3. Promoting the cultivation of energy crops
Select the correct answer using the code given below: PASSAGE - 16
(a) 1 and 2 only (b) 3 only Many farmers use synthetic pesticides to kill infesting insects.
(c) 2 and 3 only (d) 1, 2 and 3 The consumption of pesticides in some of the developed countries
163. In the context of using biomass, which of the following is/ is touching 3000 grams/hectare. Unfortunately, there are reports
are the characteristic/characteristics of the sustainable that these compounds possess inherent toxicities that endanger
production of biofuel? the health of the farm operators, consumers and the environment.
1. Biomass as a fuel for power generation could meet all Synthetic pesticides are generally persistent in environment.
the primary energy requirements of the world by 2050 Entering in food chain they destroy the microbial diversity and
2. Biomass as a fuel for power generation does not cause ecologcal imbalance. Their indiscriminate use has resulted
necessarily disrupt food and forest resources in development of resistance among insects to insecticides,
3. Biomass as a fuel for power generation could help in upsetting of balance in nature and resurgence of treated
achieving negative emissions, given certain nascent populations. Natural pest control using the botanical pesticides
technologies is safer to the user and the environment because they break down
Select the correct answer using the code given below: into harmless compounds within hours or days in the presence
(a) 1 and 2 only (b) 3 only of sunlight Plants with pesticidal properties have been in nature
(c) 2 and 3 only (d) 1, 2 and 3 for millions of years without any ill or adverse effects on the
164. With reference to the passage, following assumptions have ecosystem. They are easily decomposed by many microbes
been mad : common in most soils. They help in the maintenance of biological
1. Some climate-energy models suggest that the use of diversity of predators and the reduction of environmental
biomass as a fuel for power generation helps in contamination and human health hazards. Botanical pesticides
mitigating greenhouse gas emissions formulated from plants are biodegradable and their use in crop
2. It is not possible to use biomass as a fuel for power protection is a practical sustainable alternative.
generation without disrupting food and forest 167. On the basis of the above passage, the following
resources assumptions have been made :
Which of these assumptions is/are valid? 1. Synthetic pesticides should never be used in modern
(a) 1 only (b) 2 only agriculture.
(c) Both 1 and 2 (d) Neither 1 nor 2 2. One of the aims of sustainable agriculture is to ensure
minimal ecological imbalance.
PASSAGE - 14 3. Botanical pesticides are more effective as compared
to synthetic pesticides.
A successful democracy depends upon widespread interest and
Which of the assumptions given above is/are correct?
participation in politics, in which voting is an essential part. To
(a) 1 and 2 only (b) 2 only
deliberately refrain from taking such an interest, and from voting,
(c) 1 and 3 only (d) 1, 2 and3
EBD_7711
66
168. Which of the following statements is/are correct regarding 170. With reference to the above passage, the following
biopesticides? assumptions have been made:
1. They are not hazardous to human health. 1. The Right to Education guarantees teachers’
2. They are persistent in environment. accountability for the learning process of children.
3. They are essential to maintain the biodiversity of any 2. The Right to Education guarantees 100% enrolment
ecosystem. of children in the schools.
Select the correct answer using the code given below. 3. The Right to Education intends to take full advantage
(a) 1 only (b) 1 and 2 only of demographic dividend.
(c) 1 and 3 only (d) 1, 2 and 3 Which of the above assumptions is/are valid?
[Prelim II-2018]
PASSAGE - 17 (a) 1 only
(b) 2 and 3 only
It is no longer enough for us to talk about providing for (c) 3 only
universal access to education. Making available schooling (d) 1, 2 and 3
facilities is an essential prerequisite, but is insufficient to 171. According to the passage, which one of the following is
ensure that all children attend school and participate in the critical in bringing quality in education?
learning process. The school may be there, but children may [Prelim II-2018]
not attend or they may drop out after a few months. Through (a) Ensuring regular attendance of children as well as
school and social mapping, we must address the entire gamut teachers in school
of social, economic, cultural and indeed linguistic and (b) Giving pecuniary benefits to teachers to motivation
pedagogic issues, factors that prevent children from weaker them
sections and disadvantaged groups, as also girls, from (c) Understanding the socio-cultural background of
regularly attending and complementing elementary education. children
The focus must be on the poorest and most vulnerable since (d) Inculcating learning through activities and discovery
these groups are the most disempowered and at a greatest 172. What is the essential message in this passage?
risk of violation or denial of their right to education. [Prelim II-2018]
The right to education goes beyond free and compulsory (a) The Right to Education now is a Fundamental Right.
education to include quality education for all. Quality is an (b) The Right to Education enables the children of poor and
integral part of the right to education. If the education weaker sections of the society to attend schools.
process lacks quality, children are being denied their right. (c) The Right to Free and Compulsory Education should
The Right of Children to Free and Compulsory Education include quality education for all.
Act lays down that the curriculum should provide for (d) The Government as well a parents should ensure that
learning through activities, exploration and discovery. This all children attend schools.
places an obligation on us to change our perception of
children as passive receivers of knowledge, and to move PASSAGE - 18
beyond the convention of using textbooks as the basis of
examinations. The teaching-learning process must become Some people belive that leadership is a quality which you
stress-free; and a massive programme for curricular reform have at birth or not at all. This theory is false, for the art of
should be initiated to provide for a child-friendly learning leadership can be acquired and can indeed be taught. This
system, that is more relevant and empowering. Teacher discovery is made in time of war and the results achieved can
accountability systems and processes must ensure that surprise even the instructors. Faced with the alternatives of
children are learning, and that their right to learn in a child- going left or right, every soldier soon grasps that a prompt
friendly environment is not violated. Testing and decision either way is better than an endless discussion. A
assessment systems must be reexamined and redesigned firm choice for direction has an even chance of being right
to ensure that these do not force children to struggle while to do nothing will be almost certainly wrong.
between school and tuition centres, and bypass childhood. 173. The author of the passage holds the view that
169. According to the passage, which of the following is/are of [Prelim II-2018]
paramount importance under the Right to education? (a) leadership can be taught through war experience only
1. Sending of children to school by all parents (b) leadership can be acquired as well as taught
2. Provision of adequate physical infrastructure in schools (c) the results of training show that more people acquire
3. Curricular reforms for developing child-friendly learning system leadership than are expected
Select the correct answer using the code given below. (d) despite rigorous instruction few leaders are produced
[Prelim II-2018]
PASSAGE - 19
(a) 1 only
(b) 1 and 2 only In front of us was walking a bare-headed old man in tattered clothes.
(c) 3 only He was driving his beasts. They were all laden with heavy loads of
(d) None of the above clay from the hills and looked tired. The man carried a long whip
67
which perhaps he himself had made. As he walked down the road
PASSAGE - 21
he stopped now and then to eat the wild berries that grew on bushes
along the uneven road. When he threw away the seeds, the bold Set against a rural backdrop, 'Stench of kerosene' is the story of a
birds would fly to peck at them. Sometimes a stray dog watched the couple, Guleri and Manak, who have been happily married for
procession philosophically and then began to bark. When this several years but do not have a child. Manak's mother is desperate
happened, my two little sons would stand still holding my hands to have a grandchild to carry on the family name. Hence, she gets
firmly. A dog can sometimes be dangerous indeed. [Prelim II-2014] Manak remarried in Guleri's absence. Manak, who acts as a
174. The author's children held his hands firmly because reluctant but passive spectator, is meanwhile, informed by a friend
(a) they were scared of the barking dogs. that Guleri, on hearing about her husband's second marriage,
poured kerosene on her clothes and set fire to them. Manak is
(b) they wanted him to pluck berries.
heartbroken and begins to live as if he were a dead man. When his
(c) they saw the whip in the old man's hand.
second wife delivers a son, Manak stares at the child for a long
(d) the road was uneven. time and blurts out, "Take him away ! He stinks of kerosene."
175. The expression "a stray dog watched the procession 180. This is a sensitive issue-based story which tries to sensitise
philosophically" means that the readers about [Prelim II-2015]
(a) the dog was restless and ferocious. (a) Male chauvinism and infidelity
(b) the dog stood aloof, looking at the procession with (b) Love and betrayal
seriousness. (c) Lack of legal safeguards for women
(c) the dog looked at the procession with big, wondering eyes. (d) Influence of patriarchal mindset
(d) the dog stood there with his eyes closed.
PASSAGE - 22
PASSAGE - 20
The very first lesson that should be taught to us when we are
Cynthia was a shy girl. She believed that she was plain and of enough to understand it, is that complete freedom from the
untalented. One day her teacher ordered the entire class to show obligation to work is unnatural, and ought to be illegal, as we
up for audition for the school play. Cynthia nearly died of fright can escape our share of the burden of work only by throwing
when she was told that she would have to stand on stage in front it on someone else's shoulders. Nature ordains that the human
of the entire class and deliver dialogues. The mere thought of it race shall perish of famine if it stops working. We cannot escape
made her feel sick. But a remarkable transformation occurred from this tyranny. The question we have to settle is how much
during the audition. A thin, shy girl, her knees quaking, her stomach leisure we can afford to allow ourselves.
churning in terror, began to stun everyone with her excellent 181. The main idea of the passage is that [Prelim II- 2017]
performance. Her bored classmates suddenly stopped their noisy (a) it is essential for human beings to work
chat to stare at her slender figure on the stage. At the end of her (b) there should be a balance between work and leisure
audition, the entire room erupted in thunderous applause. (c) working is a tyranny which we have to face
[Prelim II-2014] (d) human's understanding of the nature of work is essential
176. Cynthia was afraid to stand on stage because
(a) she felt her classmates may laugh at her. PASSAGE - 23
(b) her stomach was churning. We have hard work ahead. There is no resting for any of us till
(c) she lacked self-confidence. we redeem our pledge in full, till we make all the people of India
(d) she did not like school plays. what destiny intends them to be. We are citizens of a great
177. Cynthia's classmates were chatting because country, on the verge of bold advance, and we have to live up
to that high standard. All of us, to whatever religion we may
(a) it was their turn to act next. belong are equally the children of India with equal rights, privileges
(b) they were bored of the performances. and obligations. We cannot encourage communalism or
(c) Cynthia did not act well. narrowmindedness, for no nation can be great whose people are
(d) the teacher had no control over them. narrow in thought or action.
182. The challenge the author of the above passage throws to
178. Cynthia's knees were quaking because
the public is to achieve. [Prelim II- 2017]
(a) she felt nervous and shy. (a) a high standard of living, progress and privileges
(b) the teacher scolded her. (b) equal privileges, fulfilment of destiny and political
(c) she was very thin and weak. tolerance
(d) she was afraid of her classmates. (c) spirit of adventure and economic parity
179. The transformation that occurred during the audition refers to (d) hard work, brotherhood and national unity
(a) the nervousness of Cynthia. PASSAGE - 24
(b) the eruption of the entire room in thunderous applause. "The individual, according to Rousseau, parts his person and
(c) the surprise on\the faces of her classmates. all his power in common under the supreme direction of the
(d) the stunning performance of Cynthia. General will and in our corporate capacity we receive each
member as an indivisible part of the whole."
EBD_7711
68
183. In the light of the above passage, the nature of General 187. The letters L, M, N, O, P, Q, R, S and T in their order are
Will is best described as [Prelim II-2017] substituted by nine integers 1 to 9 but not in that
(a) the sum total of the private wills of the individuals order. 4 is assigned to P. The difference between P and
(b) what is articulated by the elected representatives of T is 5. The difference between N and T is 3. What is the
the individuals integer assigned to N? [2014 - II]
(c) the collective good as distinct from private wills of (a) 7 (b) 5
the individuals (c) 4 (d) 6
(d) the material interests of the community 188. The number of deaths among the army personnel is 8
in 1000, but among the civilian population it is 20 per
PASSAGE - 25 1000. Which one of the following inferences can be
Though I have discarded much of past tradition and drawn from this statement ? [2014 - II]
custom, and am anxious that India should rid herself of (a) It is better to join the army.
all shackles that bind and contain her and divide her (b) The relationship is fortuitous.
people, and suppress vast numbers of them, and prevent (c) Qualityof Life Index is very high within the armed forces.
the free development of the body and the spirit; though (d) Thegroupscannot becompared dueto their heterogeneity.
I seek all this, yet I do not wish to cut myself off from
189. Given the statement :"Buses are the cause of more
that past completely. I am proud of that great inheritance
accidents than cars, and trucks cause fewer accidents
that has been and is, ours and I am conscious that I
than buses", which of the following conclusions can
too, like all of us, am a link in that unbroken chain
we draw ? [2014 - II]
which goes back to the dawn of history in the immemorial
past of India. (a) There are more buses on the road than trucks.
184. The author wants India to rid herself of certain past bonds (b) Car drivers are more careful than bus drivers.
because [Prelim II-2018] (c) Truck drivers are more skilled than either car or
(a) he is not able to see the relevance of the past bus drivers.
(b) there is not much to be proud of (d) None of the above
(c) he is not interested in the history of India 190. If political leadership fails to emerge, there is Q
(d) they obstruct her physical and spiritual growth likelihood of military taking over power in developing
countries. Radical student groups or labour may try
MENTAL ABILITY, LOGICAL to raise revolution but they are not likely to compete
REASONING & PROBLEM SOLVING with the military. Military intervention, rule, and
withdrawal from politics is closely related to a
185. A question paper must have a question on one of the society's level of political development."
eight poets : A, B, C, D, E, F, G or H. The first four belong
In the context of political development, the assumption
to the medieval period while the rest are considered
in the above passage is that [2014 - II]
modern poets.
(a) political leadership is not an effective instrument.
Generally, modern poets figure in the question paper
in alternate years. Generally those who like H like G (b) military fills in political vacuum.
also; and those who like F like E also. The paper-setter (c) military intervention is inevitable for development.
does not like to ask about F as he has written a book (d) None of the above
on F, but he likes F. Last year, the paper contained a 191. Four persons, Alok, Bhupesh, Chander and Dinesh
question on A. On the basis of the information given, have a total of ` 100 among themselves. Alok and
this year's paper is most likely to contain a question Bhupesh between them have as much money as
on [2014 - II] Chander and Dinesh between them, but Alok has more
(a) C (b) E money than Bhupesh; and Chander has only half the
(c) F (d) H money that Dinesh has. Alok has in fact ` 5 more than
186. In a group of six women there are four dancers, four Dinesh has. Who has the maximum amount of money?
vocal musicians, one actress and three violinists. [2014 - II]
Girija and Vanaja are among the violinists while Jalaja (a) Alok (b) Bhupesh
and Shailaja do not know how to play on the violin. (c) Chander (d) Dinesh
Shailaja and Tanuja are among the dancers. Jalaja, 192. In a row 'A' is in the 11 th position from the left and
Vanaja, Shailaja and Tanuja are all vocal musicians 'B' is in the 10 th position from the right. If ‘A’ and
and two of them are also violinists. If Pooja is an ‘B’ interchange, then 'A' becomes 18th from the left.
actress, who among the following is certainly a dancer How many persons are there in the row other than
and a violinist ? [2014 - II] 'A' and 'B' ? [2014 - II]
(a) Jalaja (b) Pooja (a) 27 (b) 26
(c) Shailaja (d) Tanuja (c) 25 (d) 24
69
193. If the 3rd day of a month is Monday, which one of the number of tries can the cards be turned upside down
following will be the fifth day from 21st of this month? such that all the six cards show number '2' on the
(a) Monday (b) Tuesday upper side ? [2014 - II]
(c) Wednesday (d) Friday (a) 3 (b) 5
(c) 7 (d) This cannot be achieved
DIRECTIONS (Qs. 194-196) : Read the passage given below
200. There are seven persons up on a ladder, A, B, C, D, E, F
and answer the items follow.
and G (not in that order). A is further up than E but is
A, B, C, D, E, F are members of a family. They are engineer, lower than C. B is in the middle. G is between A and B.
stenographer, doctor, draughtsman, lawyer and judge (not E is between B and F. If F is between E and D, the person
in order). A, the engineer is married to the lady on the bottom step of the ladder will be [2014 - II]
stenographer. (a) B (b) F
The judge is married to the lawyer. F, the draughtsman is (c) D (d) E
the son of B and brother of E. C, the lawyer is the daughter- 201. Consider that : [2014 - II]
in-law of D. E is the unmarried doctor. D is the grandmother
1. A is taller than B.
of F. There are two married couples in the family. [2014 - II]
2. C is taller than A.
194. What is the profession of B ?
3. D is taller than C.
(a) Judge (b) Lawyer
(c) Draughtsman (d) Cannot be determined 4. E is the tallest of all.
195. Which of the following is/are a couple/couples ? If they are made to sit in the above order of their
(a) AD only height, who will occupy the mid position ?
(b) BC only (a) A (b) B
(c) Both AD and BC (c) C (d) D
(d) Both AC and BD 202. Consider the following statements : [2014 - II]
196. What is the profession of D? There are six villages A, B, C, D, E and F.
(a) Judge F is 1 km to the west of D.
(b) Stenographer B is 1 km to the east of E.
(c) Doctor A is 2 km to the north of E.
(d) Cannot be determined C is 1 km to the east of A.
197. There are 50 students admitted to a nursery class. D is 1 km to the south of A.
Some students can speak only English and some can Which three villages are in a line ?
speak only Hindi. 10 students can speak both English (a) A, C, B (b) A, D, E
and Hindi. If the number of students who can speak (c) C, B, F (d) E, B, D
English is 21, then how many students can speak 203. Four children are sitting in a row. A is occupying the
Hindi, how many can speak only Hindi and how many seat next to B but not next to C. If C is not sitting next
can speak only English? [2014 - II] to D, who is/are occupying seat/seats adjacent to D ?
(a) 21, 11and 29 respectively [2014 - II]
(b) 28, 18 and 22 respectively (a) B (b) A
(c) 37, 27 and 13 respectively (c) B and A (d) Impossible to tell
(d) 39, 29 and 11 respectively 204. Assume that [2014 - II]
198. Out of a total of 120 musicians in a club, 5% can play 1. thehour and minutehandsof aclockmove without jerking.
all the three instruments, guitar, violin and flute. It so 2. the clock shows a time between 8 o'clock and 9 o'clock.
happens that the number of musicians who can play 3. the two hands of the clock are one above the other.
any two and only two of the above instruments is 30. After how many minutes (nearest integer) will the
The number of musicians who can play the guitar two hands be again lying one above the other ?
alone is 40. What is the total number of those who can (a) 60 (b) 62
play violin alone or flute alone ? [2014 - II] (c) 65 (d) 67
(a) 45 (b) 44 205. Examine the following figure : [2014 - II]
(c) 38 (d) 30
199. Six identical cards are placed on a table. Each card
has number '1' marked on one side and number '2'
marked on its other side. All the six cards are placed
in such a manner that the number '1' is on the upper Which one of the following figures has the above
side. In one try, exactly four (neither more nor less) figure embedded in it?
cards are turned upside down. In how many least
EBD_7711
70
209. With reference to the figure given below, the number
of different routes from S to T without retracing from
U and/or V, is [2014 - II]

(a) (b) (c) (d)


206. Consider the following matrix : [2014 - II]
S T
U V

(a) 3 (b) 6
(c) 9 (d) 18
210. Consider the following figures : [2014 - II]

Which one of the following figures fits into the blank ?


part of the above matrix ?
Change in positions of beads in the four figures above
follows a sequence. Following the same sequence,
(a) (b) which of the figures below should appear as the fifth
figure above ?

(c) (d)
(a) (b)
207. Consider the table given below in which the numbers
bear certain relationship among themselves along the
rows : [2014 - II]
(c) (d)

29 13 18
211. Price is not the same thing as value. Suppose that on a
33 X 19 day the price of everything viz., coal, bread, postage
stamps, a day's labour, the rent of houses, etc. were to
30 27 3 double. Prices then would certainly rise, but values
of all things except one would not."
Which one of the following numbers is the missing
The writer wants to say that if prices of all things
number indicated above by X ?
were doubled [2014 - II]
(a) 19 (b) 15 (c) 14 (d) 8
208. Consider the following matrix with one empty block (a) the values of all things would remain constant.
in the lower extreme corner : [2014 - II] (b) the values of the things sold would be doubled.
(c) the values of the things bought would be halved.
(d) the value of money only would be halved.
212. "Liberty, therefore, is never real unless the Government
can be called to account when it invades rights."
Which one of the following is the best justification of
the above statement ? [2014 - II]
(a) In the realisation that the government can be
brought to book in a court of law
(b) Inidentifying a man as a political unit in a way
Which of the following figures could fit in the empty which distinguishes him from other citizens
block and thus complete the matrix ? (c) In a decentralized society wherein the basic needs
of men can find satisfaction
(d) In the understanding that liberty and restraints
are complementary
213. Six books are labelled A, B, C, D, E and F and are placed
(a) (b) (c) (d) side by side. Books B, C, E and F have green covers
while others have yellow covers. Books A, B and D are
71
new while the rest are old volumes. Books A, B and C 220. The mangroves can shed tons of leaves per acre every
are law reports while the rest are medical extracts. year; fungi and bacteria break down this leaf litter
Which two books are old medical extracts and have and consume it, they then are consumed by tiny
green covers ? [2014 - II] worms and crustaceans, which in turn feed small fish,
(a) B and C (b) E and F which feed larger fish and birds and crocodiles.
(c) C and E (d) C and F Which among the following is the most logical
214. If A runs less fast than B, and B runs as fast but not inference of the above statement? [2015-II]
faster than C; then, as compared to A, C runs[2015-II] (a) Coastal areas cannot have food chains without
(a) slower than A mangroves.
(b) faster than A (b) Mangroves are an essential component of all
(c) with same speed as A marine ecosystems.
(d) Given data is not sufficient to determine (c) Mangroves have a crucial role in some of the
coastal food chains.
215. Examine the following statements: [2015-II]
(d) The composition of marine flora and fauna is
1. Lady's finger is tastier than cabbage.
largely determined by mangroves.
2. Cauliflower is tastier than lady's finger.
221. "By liberty I mean the eager maintenance of that
3. Cabbage is not tastier than peas. atmosphere in which men have the opportunity to be
The conclusion that can be drawn from these their best selves."
statements is that Which one of the following expresses the view implied
(a) peas are as tasty as lady's finger. in the above statement? [2015-II]
(b) peas are as tasty as cauliflower and lady's finger. (a) Liberty is the absence of restraint on human
(c) cabbage is the least tasty of the four vegetables. action.
(d) cauliflower is tastier than cabbage. (b) Liberty is what law permits people to perform.
216. Shahid and Rohit start from the same point in (c) Liberty is the ability to do what one desires.
opposite directions. After each 1 km, Shahid always (d) Liberty is the maintenance of conditions for the
turns left and Rohit always turns right. Which of the growth of human personality.
following statements is correct ? [2015-II] 222. Consider the figures given below: [2015-II]
(a) After both have travelled 2 km, the distance
between them is 4 km.
(b) They meet after each has travelled 3km.
(c) They meet for the first time after each has
travelled 4 km.
(d) They go on without ever meeting again. To fit the question mark, the correct answer is
217. Out of 130 students appearing in an examination, 62
failed in English, 52 failed in Mathematics, whereas
24 failed in both English and Mathematics. The (a) (b)
number of students who passed finally is [2015-II]
(a) 40 (b) 50
(c) 55 (d) 60
218. In a group of persons travelling in a bus, 6 persons (c) (d)
can speak Tamil, 15 can speak Hindi and 6 can speak
Gujarati. In that group none can speak any other
language. If 2 persons in the group can speak two 223. Consider the following matrix: [2015-II]
languages only and one person can speak all the three 3 8 10 2 ? 1
languages, then how many persons are there in the
6 56 90 2 20 0
group? [2015-II]
(a) 21 (b) 22 What is the missing number at '?' in the matrix?
(c) 23 (d) 24 (a) 5 (b) 0
219. In a parking area, the total number of wheels of all the (c) 7 (d) 3
cars (four-wheelers) and scooters/motorbikes (two- 224. What is the missing number 'X' of the series 7, X, 21,
wheelers) is 100 more than twice the number of parked 31, 43? [2015-II]
vehicles. The number of cars parked is [2015-II] (a) 11 (b) 12
(a) 35 (b) 45 (c) 13 (d) 14
(c) 50 (d) 55
EBD_7711
72
225. Four cardboard pieces of specific shapes are shown in 229. All good athletes want to win and all athletes who
the following figure: [2015-II] want to win eat a well-balanced diet; therefore all
athletes who do not eat a well-balanced diet are bad
athletes. [2015-II]
The best conclusion from this statement is that
(a) no bad athlete wants to win.
(b) no athlete who does not eat a well-balanced diet
is a good athlete.
(c) every athlete who eats a well-balanced diet is a good
athlete.
Which one of the following figures given can be formed
(d) all athletes who want to win are good athletes.
by joining these pieces together?
230. Between 6 PM and 7 PM the minute hand of a clock
will be ahead of the hour hand by 3 minutes at
[2015-II]
(a) (b) (a) 6: 15 PM (b) 6: 18 PM
(c) 6: 36 PM (d) 6: 48 PM
231. Two men, Anil and David, and two women, Shabnam
and Rekha are in a sales group. Only two speak Tamil.
(c) (d) The other two speak Marathi. Only one man and one
woman can drive a car. Shabnam speaks Marathi. Anil
speaks Tamil. Both Rekha and David can drive.
226. Four persons A, B, C and D consisting of two married Which of the following statements is true? [2015-II]
couples are in a group. Both the women are shorter (a) Both the Tamil speakers can drive a car.
than their respective husbands. A is the tallest among (b) Both the Marathi speakers can drive a car.
the four. C is taller than B. D is B's brother. In this (c) Both of those who can drive a car speak Marathi.
context, which one of the following statements is not (d) One of those who can drive a car speaks Tamil.
correct ? [2015] 232. A society consists of only two types of people fighters
(a) All four have family ties. and cowards. Two cowards are always friends. A
(b) B is the shortest among the four. fighter and a coward are always enemies. Fighters
(c) C is taller than D. are indifferent to one another. If A and B are enemies,
C and D are friends, E and F are indifferent to each
(d) A is B's husband.
other, A and E are not enemies, while Band F are
227. Consider the following statements : [2015]
enemies. [2015-II]
1. A man had a wife, two sons and daughters in his
Which of the following statements is correct?
family
(a) B, C and F are cowards.
2. The daughters were invited to a feast and the male
members of the family went out to take part in a (b) A, E and F are fighters.
picnic (c) B and E are in the same category.
3. The man's father did not return from his work. (d) A and F are in different categories.
Which of the following statements is true? 233. Each of the six different faces of a cube has been coated
(a) Only the man's wife was left at home. with a different colour i.e., V, I, B, G, Y and O. Following
(b) It is likely that the man's wife was left at home. information is given: [2015-II]
(c) None was left at home 1. Colours Y, O and B are on adjacent faces.
(d) More than one person was left at home. 2. Colours I, G and Y are on adjacent faces.
228. Geeta: Naresh has become a better boxer since he 3. Colours B, G and Y are on adjacent faces.
started meditation. 4. Colours O, V and B are on adjacent faces.
Radha: Impossible, A boxer's most important asset is Which is the colour of the face opposite to the face
his aggressiveness. coloured with O?
Radha's statement reflects her belief that [2015-II]
(a) B (b) V
(a) meditation tends to make a person less aggressive.
(c) G (d) I
(b) meditation has little or no effect on the person
who practises it. 234. Consider the following statements followed by two
(c) Naresh was a poor boxer earlier because he was conclusions:
not aggressive enough. Statements : Some men are great.
(d) Naresh would not have taken to meditation as Some men are wise.
he was a boxer. Conclusion I : Men are either great or wise.
73
Conclusion II : Some men are neither great nor wise 239. What is C’s surname ? [2016-II]
Which one of the following is correct? [2015-II] (a) Riberio (b) Kumar
(a) Only conclusion I is valid (c) Singh (d) Cannot be determined
(b) Only conclusion II is valid 240. What is the colour of the tie ? [2016-II]
(a) Black (b) Blue
(c) Both the conclusions are valid
(c) White (d) Cannot be determined
(d) Neither of the conclusions is valid 241. Who wore the sweater ? [2016-II]
235. Consider the following statements: [2015-II] (a) A (b) B
1. Some claim to have seen UFOs (Unidentified (c) C (d) Cannot be determined
Flying Objects). 242. A person walks 12 km due north, then 15 km due east,
2. Life on other heavenly bodies is considered to be after that 19 km due west and then 15 km due south.
a possibility. How far is he from the starting point?
3. Voyage to space is now an established fact. (a) 5 km (b) 9 km [2016-II]
From the above statements, it may be concluded that (c) 37 km (d) 61 km
(a) UFOs are heavenly bodies 243. A cube has all its faces painted with different colours.
(b) UFOs are sent from other heavenly bodies It is cut into smaller cubes of equal sizes such that the
side of the small cube is one-fourth the big cube. The
(c) Some living species in other heavenly bodies are
number of small cubes with only one of the sides
more intelligent than man
painted is: [2016-II]
(d) Nothing definite can be said about the UFOs (a) 32 (b) 24
236. If ABC × DEED = ABCABC; where A, B, C, D and E are (c) 16 (d) 8
different digits, what are the values of D and E? 244. A military code writes SYSTEM as SYSMET and
[2015-II] NEARER as AENRER. Using the same code,
(a) D = 2, E = 0 (b) D = 0, E = 1 FRACTION can be written as: [2016-II]
(c) D = 1, E = 0 (d) D = 1, E = 2 (a) CARFTION (b) FRACNOIT
237. Usha runs faster than Kamala, Priti runs slower than (c) NOITCARF (d) CARFNOIT
Swati, Swati runs slower than Kamala. Who is the 245. In a class of 60 students, where the number of girls is
slowest runner? [2015-II] twice that of boys, Kamal, a boy, ranked seventeenth
(a) Kamala (b) Priti from the top. If there are 9 girls ahead of Kamal, the
(c) Swati (d) Usha number of boys in rank after him is: [2016-II]
(a) 13 (b) 12
238. A person climbs a hill in a straight path from point ‘O’
(c) 7 (d) 3
on the ground in the direction of north-east and
246. A person X was driving in a place where all roads ran
reaches a point ‘A’ after travelling a distance of 5 km.
either north-south or east-west, forming a grid.
Then, from the point ‘A’ he moves to point ‘B’ in the
Roads are at a distance of 1 km from each other in a
direction of north-west. Let the distance AB be 12 km.
parallel. He started at the intersection of two roads,
Now, how far is the person away from the starting
drove 3 km north, 3 km west and 4 km south. Which
point ‘O’? [2016-II]
further route could bring him back to his starting
(a) 7 km (b) 13 km
point, if the same route is not repeated? [2016-II]
(c) 17 km (d) 11 km
(a) 3 km east, then 2 km south
DIRECTIONS for the following 3 (three) items : Consider (b) 3 km east, then 1 km north
the given information and answer the three items that (c) 1 km north, then 2 km west
follow. (d) 3 km south, then 1 km north
247. Consider the following statement: [2016-II]
When three friends A, B and C met, it was found that each
“We shall go either for a picnic or for trekking”.
of them wore an outer garment of a different colour. In
Which of the following, if true, would falsify this
random order, the garments are: jacket, sweater and tie;
claim?
and the colours are: blue, white and black. Their surnames
(a) We go for a picnic but not for trekking
in random order are : Ribeiro, Kumar and Singh.
(b) Activities such as picnic and trekking are
Further, we know that :
encouraged by the health authorities
1. neither B nor Ribeiro wore a white sweater
(c) We go for trekking and not for picnic
2. C wore a tie
(d) We do not go either for picnic or for trekking
3. Singh’s garment was not white
248. There were 50 faculty members comprising 30 males
4. Kumar does not wear a jacket
and the rest females. No male faculty member knew
5. Ribeiro does not like to wear the black colour
music, but many of the female faculty members did.
6. Each of the friends wore only one outer garment of
The Head of the institution invited six faculty
only one colour
members to a tea party by draw of lots. At the party
EBD_7711
74
it was discovered that no member knew music. The 254. Who is the spouse of R? [2016-II]
conclusion is that: [2016-II] (a) P (b) T
(a) the party comprised male faculty members only (c) Q (d) S
(b) the party comprised only those female faculty 255. Who is the lawyer? [2016-II]
members who could not give renderings in (a) P (b) Q
music (c) R (d) S
(c) the party comprised both male and female 256. Who of the following is definitely a man? [2016-II]
faculty members (a) P (b) S
(d) nothing can be said about the gender (c) Q (d) None of the above
composition of the party 257. There is an order of 19000 quantity of a particular
249. Five people A, B, C, D and E are seated about a round product from a customer. The firm produces 1000
table. Every chair is spaced equidistant from quantity of that product per day out of which 5% are
adjacent chairs. [2016-II] unfit for sale. In how many days will the order be
(i) C is seated next to A completed? [2016-II]
(ii) A is seated two seats from D (a) 18 (b) 19
(iii) B is not seated next to A (c) 20 (d) 22
On the basis of above information, which of the 258. Consider the following statements: [2016-II]
following must be true? 1. Either A and B are of the same age or A is older
1. D is seated next to B than B
2. E is seated next to A 2. Either C and D are of the same age or D is older
3. D and C are separated by two seats than C
Select the correct answer using the code given below: 3. B is older than C
(a) 1 only (b) 1 and 2 only Which of the following conclusions can be drawn
(c) 3 only (d) Neither 1 nor 2 nor 3 from the above statements?
250. There are five hobby clubs in a college —photography, (a) A is older than B
yachting, chess, electronics and gardening. The (b) B and D are of the same age
gardening group meets every second day, the (c) D is older than C
electronics group meets every third day, the chess (d) A is older than C
group meets every fourth day, the yachting group
DIRECTIONS for the following 3 (three) items: Consider
meets every fifth day and the photography group
the given-information and answer the three items that
meets every sixth day. How many times do all the five
follow.
groups meet on the same day within 180 days?
[2016-II] Six boxes A, B, C, D, E and F have been painted with six
(a) 5 (b) 18 different colours viz., violet, indigo, blue, green, yellow
(c) 10 (d) 3 and orange and arranged from left to right (not necessarily
251. There are some nectar-filled flowers on a tree and either kept or painted with the colours in the same order).
some bees are hovering on it. If one bee lands on each Each box contains a ball of any one of the following six
flower, one bee will be left out. If two bees land on games: cricket, hockey, tennis, golf, football and volleyball
each flower, one flower will be left out. The number of (not necessarily in the same order). The golf ball is in violet
flowers and bees respectively are: [2016-II] box and is not in the box D. The box A which contains
(a) 2 and 4 (b) 3 and 2 tennis ball is orange in colour and is at the extreme right.
(c) 3 and 4 (d) 4 and 3 The hockey ball is neither in box D nor in box E. The box C
having cricket ball is painted green. The hockey ball is
DIRECTIONS for the following 5 (five) items: Consider
neither in the box painted blue nor in the box painted
the following information and answer the five items that
yellow. The box C is fifth from right and next to box B. The
follow:
box B contains volleyball. The box containing the hockey
There are five persons in a group — P, Q, R, S and T. The ball is between the boxes containing golf ball and
group has one doctor, one lawyer and one artist. P and S volleyball.
are unmarried students. T is a man married to one of the 259. Which one of the following boxes contains the golf
group members. Q is the brother of P and is neither doctor ball? [2016-II]
nor artist. R is not doctor. (a) F (b) E
252. Who is the doctor? [2016-II] (c) D (d) None of the above
(a) T (b) P 260. Which of the following statements is/are correct?
(c) Q (d) R (a) D is painted yellow [2016-II]
253. Who is the artist? [2016-II] (b) F is painted indigo
(a) P (b) Q (c) B is painted blue
(c) R (d) T (d) All of the above
75
261. The football is in the box of which colour? [2016-II] 1. Some supporters of 'party X' knew Z.
(a) Yellow 2. Some supporters of 'party X', who opposed Z's
(b) Indigo campaign strategy, knew Z.
(c) Cannot be determined as data are inadequate 3. No supporters of 'party X' supported Z 'S
(d) Blue campaign strategy.
262. In a. question paper there are five questions to be Which of the statements given above is/are not
attempted and answer to each question has two correct?
choices - True (T) or False (F). It is given that no two (a) 1 only (b) 2 and 3 only
candidates have given the answers to the five (c) 3 only (d) 1, 2 and 3
questions in an identical sequence. For this to happen 267. If second and fourth Saturdays and all the Sundays
the maximum number of candidates is: [2016-II] are taken as only holidays for an office, what would
(a) 10 (b) 18 be the minimum number of possible working days
(c) 26 (d) 32 of any month of any year? [2017-II]
263. A ate grapes and pineapple; B ate grapes and oranges; (a) 23 (b) 22
C ate oranges, pineapple and apple; D ate grapes, (c) 21 (d) 20
apple and pineapple. After taking fruits, B and C fell 268. Four tests-Physics, Chemistry, Mathematics and
sick. In the light of the above facts, it can be said that Biology are to be conducted on four consecutive days,
the cause of sickness was: [2016-II] not necessarily in the same order. The Physics test is
(a) Apple (b) Pineapple held before the test which is conducted after Biology.
(c) Grapes (d) Oranges Chemistry is conducted exactly after two tests are
264. Consider the following statements. [2016-II] held. Which is the last test held? [2017-II]
1. The rate of population growth is increasing in the country (d) Physics (b) Biology
2. The death rate is declining faster in the country (c) Mathematics (d) Chemistry
compared to birth rate 269. Consider the following : [2017-II]
3. The birth rate is declining faster in the country Statement :
compared to death rate Good voice is a natural gift but one has to keep
4. Rural-urban migration is taking place regularly practising to improve and excel well in the field of
in the country music.
Which one of the following conclusions may be true Conclusions :
in the light of the above facts? 1. Natural gifts need nurturing and care.
(a) The rate of population growth is increasing due II. Even though one's voice is not good, one can
to rural-urban migration keep practising.
(b) The rate of population growth is increasing due Which one of the following is correct in respect of
to decline in death rate only the above statement and conclusions?
(c) The rate of population growth is increasing due (a) Only conclusion I follows from the statement.
to increase in birth rate only (b) Only conclusion II follows from the statement.
(d) The rate of population growth is increasing due
(c) Either conclusion I or conclusion II follows from
to faster decline in death rate than in birth rate
the statement.
DIRECTIONS for the following 2 (two) items : Consider (d) Neither conclusion I nor conclusion II follows
the given information and answer the two items that from the statement.
follow. 270. "Rights are certain advantageous conditions of social
well-being indispensable to the true development of
No supporters of 'party X', who knew Z and supported
the citizen." [2017-II]
his campaign strategy, agreed for the alliance with 'party
In the light of this statement, which one of the
Y'; but some of them had friends in 'party Y'.
following is the correct understanding of rights?
265. With reference to the above information, which one
among the following statements must be true? (a) Rights aim at individual good only.
(b) Rights aim at social good only.
[2017-II]
(a) Some supporters of 'party Y' did not agree for (c) Rights aim at both individual and social good.
the alliance with the 'party X'. (d) Rights aim at individual good devoid of social
(b) There is at least one supporter of 'party Y' who well-being.
knew some supporters of 'party X' as a friend. 271. Consider the following : [2017-II]
(c) No supporters of 'party X' supported Z's A+ B means A is the son of B.
campaign strategy. A– B means A is the wife of B.
(d) No supporters of 'party X' knew Z. What does the expression P + R - Q mean?
266. With reference to the above information, consider (a) Q is the son of P. (b) Q is the wife of P.
the following statements : [2017-II] (c) Q is the father of P. (d) None of the above
EBD_7711
76
272. In a group of six women, there are four tennis players, (a) 1 and 2 are true. (b) 3 is true.
four postgraduates in Sociology, one postgraduate (c) 2 is false. (d) 1 is false.
in Commerce and three bank employees. Vimala 278. Consider the following relationships among
and Kamla are the bank employees while Amala members of a family of six persons A, B, C, D, E and
and Komala are unemployed. Komala and Nirmala F: [2017-II]
are among the tennis players. Amala, Kamla, l. The number of males equals that of females.
Komala and Nirmala are postgraduates in Sociology 2. A and E are sons of F.
of whom two are bank employees. If Shyamala is a 3. D is the mother of two, one boy and one girl.
postgraduate in Commerce, who among the 4. B is the son of A.
following is both a tennis player and a bank 5. There is only one married couple in the family
employee? [2017-II] at present.
(a) Amala (b) Komala Which one of the can be drawn from following
(c) Nirmala (d) Shyamala inferences the above?
273. P = (40% of A) + (65% of B) and Q = (50% of A)+ (50% (a) A, B and C are all females.
of B), where A is greater than B. (b) A is the husband of D.
In this context, which of the following statements is (c) E and F are children of D.
correct? [2017-II] (d) D is the daughter of F
(a) P is greater than Q. 279. Consider the following : [2017-II]
(b) Q is greater than P. A, B, C, D, E, F, G and H are standing in a row facing
(c) P is equal to Q. North.
(d) None of the above can be concluded with certainty. B is not neighbour of G.
274. Consider the given statem ent and the two F is to the immediate right of G and neighbour of E.
conclusions that follow : [2017-II] G is not at the extreme end.
Statement : A is sixth to the left of E.
Morning walk is good for health. H is sixth to the right of C.
Conclusions : Which one of the following is correct in respect of
1. All healthy people go for morning walk. the above?
2. Morning walk is essential for maintaining good (a) C is to the immediate left of A.
health. (b) D is immediate neighbour of B and F.
What is/are the valid conclusion/ conclusions? (c) G is to the immediate right of D.
(a) 1 only (b) 2 only (d) A and E are at the extreme ends.
(c) Both 1 and 2 (d) Neither 1 nor 2 280. In a certain code, '256' means 'red colour chalk', '589'
275. Six boys A, B, C, D, E and F play a game of cards. means "green colour flower' and '254' means 'white
Each has a pack of 10 cards. F borrows 2 cards from colour chalk'. The digit in the code that indicates
A and gives away 5 to C who in turn gives 3 to B 'white' is [2017-II]
while B gives 6 to D who passes 1 to E. Then the (a) 2 (b) 4
number of cards possessed by D and E is equal to (c) 5 (d) 8
the number of cards possessed by [2017-II] 281. The average rainfall in a city for the first four days
(a) A, B and C (b) B, C and F was recorded to be 0.40 inch. The rainfall on the last
(c) A, B and F (d) A, C and F two days was in the ratio of 4:3. The average of six
276. Which of the following statements is/are correct days was 0.50 inch. What was the rainfall on the
regarding biopesticides? [2017-II] fifth day? [2017-II]
1. They are not hazardous to human health. (a) 0.60 inch (b) 0.70 inch
2. They are persistent in environment. (c) 0.80 inch (d) 0.90 inch
3. They are essential to maintain the biodiversity
of any ecosystem. DIRECTIONS for the following 3 (three) items : Consider
Select the correct answer using the code given below. the given information and answer the three items that
(a) 1 only (b) 1 and 2 only follow.
(c) 1 and 3 only (d) 1, 2 and 3 A, B, C, D, E, F and G are Lecturers from different cities-
277. Examine the following statements : [2017-II] Hyderabad, Delhi, Shillong, Kanpur, Chennai, Mumbai
1. All colours are pleasant. and Srinagar (not necessarily in the same order) who
2. Some colours are pleasant. participated in a conference. Each one of them ' is
3. No colour is pleasant specialized in a different subject, viz, Economics,
4. Some colours are not pleasant. Commerce, History, Sociology, Geography, Mathematics
Given that statement 4 is true, what can be de?nitely and Statistics (not necessarily in the same order). Further
concluded? 1. Lecturer from Kanpur is specialized in Geography
77
2. Lecturer D is from Shillong (a) E (b) D
3. Lecturer C from Delhi is specialized in Sociology (c) A (d) B
4. Lecturer B is specialized in neither History nor 289. Lakshmi, her brother, her daughter and her son are badminton
Mathematics players. A game of doubles is about to begin:
5. Lecturer A who is specialized in Economics does (i) Lakshmi’s brother is directly across the net from
not belong to Hyderabad her daughter.
6. Lecturer F who is specialized in Commerce (ii) Her son is diagonally across the net from the worst
belongs to Srinagar player’s sibling.
7. Lecturer G who is specialized in Statistics (iii) The best player and the worst player are on the same
belongs to Chennai side of the net.
282. Who is specialized in Geography? [2017-II] Who is the best player? [Prelim II-2018]
(a) B (a) Her brother (b) Her daughter
(b) D (c) Her son (d) Lakshmi
(c) E 290. If Pen < Pencil, Pencil < Book and Book > Cap, then which
(d) Cannot be determined as data are inadequate one of the following is always true? [Prelim II-2018]
283. To which city does the Lecturer specialized in (a) Pen > Cap (b) Pen < Book
Economics belong? [2017-II] (c) Pencil = Cap (d) Pencil > Cap
(a) Hyderabad 291. Consider the following three-dimensional figure:
(b) Mumbai
(c) Neither Hyderabad nor Mumbai
(d) Cannot be determined as data are inadequate
284. Who of the following belongs to Hyderabad?
[2017-II]
(a) B
(b) E
(c) Neither B nor E
(d) Cannot be determined as data are inadequate
285. In a school, there are five teachers A, B, C, D and E,
A and B teach Hindi and English. C and B teach
English and Geography. D and A teach Mathematics How many triangles does the above figure have?
and Hindi. E and B teach History and French. Who [Prelim II-2018]
teaches maximum number of subjects. [2017-II] (a) 18 (b) 20
(a) A (b) B (c) 22 (d) 24
(c) D (d) E 292. Consider the following sum :
• + 1• + 2• + •3 + •1 = 21•
DIRECTIONS for the following 3 (three) items : Consider In the above sum, • stands for [Prelim II-2018]
the given information and answer the three items that (a) 4 (b) 5
follow. (c) 6 (d) 8
Eight railway stations A, B, C, D, E, F, G and H are 293. A solid cube of 3 cm side, painted on all its faces, is cut
connected either by two way passages or one way up into small cubes of 1 cm side. How many of the small
passages. One way passages are from C to A, E to G, cubes will have exact1y two painted faces.
B to F, D to H, G to C, E to C and H to G. Two way [Prelim II-2018]
passages are between A and E, G and B, F and D, and (a) 12 (b) 8
E and D. (c) 6 (d) 4
286. While travelling from C to H, which one of the 294. Consider the following pattern of numbers:
following stations must be passed through. [2017-II]
(a) G (b) E 8 10 15 13
(c) B (d) F 6 5 7 4
287. In how many different ways can a train travel from 4 6 8 8
F to A without passing through any station more
6 11 16 ?
than once? [2017-II]
(a) 1 (b) 2 What is the number at ? in the above pattern?
(c) 3 (d) 4 [Prelim II-2018]
288. If the route between G and C is closed, which one of the
following stations need not be passed through while (a) 17 (b) 19 (c) 21 (d) 23
travelling from H to C? [2017-II]
EBD_7711
78
Directions for the following 3 (three) items: Rotated positions 299. Consider the following figures :
of a single solid are shown below. The various faces of the solid
are marked with different symbols like dots, cross and line.
Answer the three items that follow the given figures.
[Prelim II-2018]

(I) (II) (III)

(I) (II) (III) (IV)


295. What is the symbol on the face opposite to that containing
a single dot?
(a) Four dots (b) Three dots
(c) Two dots (d) Cross (IV) (V) (VI)
296. What is the symbol on the face opposite to that containing
two dots?
In the figures (I) to (VI) above, some parts are shown to
(a) Single dot (b) Three dots change their positions in regular directions. Following the
(c) Four dots (d) Line same sequence, which of the figures given below will
297. What is the symbol on the face opposite to that containing appear at (VII) stage? [Prelim II-2018]
the cross?
(a) Single dot (b) Two dots
(c) Line (d) Four dots
298. Consider the figures given below : (a) (b)

(c) (d)

Directions for the following 6 (six) Items: Read the information


given below and answer the six items that follow.
A, B, C and D are students. They are studying in four different
cities, viz., P, Q, Rand S (not necessarily in that order). They are
studying in Science college, Arts college, Commerce college and
Engineering college (not necessarily in that order), which are
situated in four different States, viz., Gujarat, Rajasthan, Assam
? and Kerala (not necessarily in that order). Further, it
is given that- [Prelim II-2018]
(i) D is studying in Assam
(ii) Arts college is located in city S which is in Rajasthan
(iii) A is studying in Commerce college
(iv) B is studying in city Q
To fit the question mark, the correct answer is
(v) Science college is located in Kerala
[Prelim II-2018]
300. A is studying in
(a) (b) (a) Rajasthan (b) Gujarat
(c) city Q (d) Kerala
301. Science college is located in
(c) (d) (a) city Q (b) city S
(c) city R (d) city P
302. C is studying in
(a) Science college (b) Rajasthan
(c) Gujarat (d) city Q
79
303. Which one of the following statements is correct? (b) Meat-producing industry violates the laws against
(a) D is not studying in city S. cruelty to animals.
(b) A is studying in Science college. (c) Mass production of meat through industrial farming
(c) A is studying in Kerala. is undesirable and should be stopped immediately.
(d) Engineering college is located in Gujarat. (d) Environmental cost of meat production is
304. Which one of the following statements is correct regarding unsustainable when it is produced through industrial
Engineering college? farming.
(a) C is studying there. A male tiger was removed from Pench Tiger Reserve and
(b) B is studying there. was relocated in Panna National Park. Later, this tiger
(c) It is located in Gujarat. trekked toward his home 250 miles away. The trek of this
(d) D is studying there. solitary tiger highlights a crisis. Many wildlife reserves
305. Which one of the following statements is correct? exist as islands of fragile habitat in a vast sea of humanity,
(a) Engineering college is located in Assam. yet tigers can range over a hundred miles, seeking prey,
(b) City Q is situated in Assam. mates and territory. Nearly a third of India’s tigers live
(c) C is studying in Kerala. outside tiger reserves, a situation that is dangerous for
(d) B is studying in Gujarat. both human and animal. Prey and tigers can only disperse
if there are recognized corridors of land between protected
areas to allow unmolested passage.
309. Which of the following is the most rational and crucial
Directions for the following 2 (two) items: Read the following message given by the passage? [Prelim II-2018]
information and answer the two items that follow. (a) The conflict between man and wildlife cannot be
The plan of an office block for six officers A, B,C, D, E and F is resolved, no matter what efforts we make.
as follows : Both B and C occupy offices to the right of the (b) Safe wildlife corridors between protected areas is an
corridor (as one enters the office block) and A occupies on the essential aspect of conservation efforts.
left of the corridor. E and F occupy offices on opposite sides of (c) India needs to declare more protected areas and set
the Corridor but their offices do not face each other. The offices up more tiger reserves.
of C and D face each other. E does not have a corner office. Fs (d) India’s National Parks and Tiger Reserves need to be
office is further down the corridor than A’s, but on the same side. professionally managed.
[Prelim II-2018] 310. With reference to the above passage, the following
306. If E sits in his office and faces the corridor, whose office assumptions have been made:
is to his left? 1. The strategy of conservation of wildlife by relocating
(a) A (b) B them from one protected area to another is not often
(c) C (d) D successful.
307. Who is/are F’s immediate neighbour/neighbours? 2. India does not have suitable legislation to save the
(a) A only (b) A and D tigers, and its conservation efforts have failed which
(c) C only (d) Band C forced the tigers to live outside protected areas.
Which of the above assumptions is/are valid?
Directions for the following 3 (three) items : Read the following
[Prelim II-2018]
two passages and answer the items that follow. Your answers
(a) 1 only (b) 2 only
to these items should be based on the passages only.
(c) Both 1 and 2 (d) Neither 1 nor 2
The quest for cheap and plentiful meat has resulted in
Directions for the following 8 (eight) items : Read the following
factory farms where more and more animals are squeezed
eight passages and answer the items that follow. Your answers
into smaller lots in cruel and shocking conditions. Such
to these items should be based on the passages only.
practices have resulted in many of the world’s health
pandemics such as the avian flu. Worldwide, livestock are All actions to address climate change ultimately involve
increasingly raised in cruel, costs. Funding is vital in order for countries like India to
cramped conditions, where animals spend their short lives design and implement adaptation and mitigation plans and
under artificial light, pumped full of antibiotics and growth projects. The problem is more severe for developing
hormones, until the day they are slaughtered. Meat countries like India, which would be one of the hardest hit
production is water-intensive. 15000 litres of water is needed by climate change, given its need to finance development.
for every kilogram of meat compared with 3400 litres for rice, Most countries do indeed treat climate change as real
3300 litres for eggs and 255 litres for a kilogram of potatoes. threat and are striving to address it in a more comprehensive
308. What is the most rational and crucial message given by and integrated manner with the limited resources at their
the passage? [Prelim II-2018] disposal.
(a) Mass production of meat through industrial farming 311. With reference to the above passage, the following
is cheap and is suitable for providing protein nutrition assumptions have been made :
to poor countries. 1. Climate change is not a challenge for developed
countries.
EBD_7711
80
2. Climate change is a complex policy issue and also a 314. Which one of the following is the most important
development issue for many countries. implication of the passage? [Prelim II-2018]
3. Ways and means of finance must be found to enable (a) India can have territorial claims in the Arctic territory
developing countries to enhance their adaptive and free access to its resources.
capacity. (b) Melting of summer ice in the Arctic leads to changes
Which of the above assumptions is/are valid? in the geopolitics.
(a) 1 and 2 only (b) 3 only (c) The Arctic region will solve the world’s future problem
(c) 2 and 3 only (d) 1, 2 and 3 of resource crunch.
Cooking with biomass and coal in India is now recognized (d) The Arctic region has more resources than Antarctica.
to cause major health problems, with women and children Being a member of the WTO, India is bound by the
in poor populations facing the greatest risk. There are agreements that have been signed and ratified by its
more than 10 lakh premature deaths each year from members, including itself. According to Articlc 6 of the
household air pollution due to polluting cooking fuels Agriculture Agreement, providing minimum support prices
with another 1.5 lakh due to their contribution to general for agricultural products is considered distorting and is
outdoor air pollution in the country. Although the fraction subject to limits. The subsidy arising from ‘minimal
of the Indian population using clean cooking fuels, such supports’ cannot exceed 10 per cent of the value of
as LPG natural gas and electricity, is slowly rising, the agricultural production for developing countries. PDS in
number using polluting solid fuels as their primary cooking India entails minimum support prices and public
fuel has remained static for nearly 30 years at about 70 stockholding of food grains. It is possible that, in some
crore. years, the subsidy to producers
312. Which of the following is the most crucial and logical will exceed 10 per cent of the value of agricultural
inference that can be made from the above passage? production.
[Prelim II-2018] 315. What is the crucial message conveyed by the above
(a) Rural people are giving up the use of polluting solid passage? [Prelim II-2018]
fuels due to their increasing awareness of health (a) India should revise its PDS.
hazards. (b) India should not be a member of WTO.
(b) Subsidizing the use of clean cooking fuels will solve (c) For India, food security collides with trade.
the problem of India’s indoor air pollution. (d) India provides food security to its poor.
(c) India should increase its import of natural gas and India’s educational system is modelled on the mass
produce more electricity. education system that developed in the 19th century in
(d) Access to cooking gas can reduce premature deaths Europe and later spread around the world. The goal of the
in poor households. system is to condition children as ‘good’ citizens and
Scientific knowledge has its dangers but so has every productive workers. This suited the industrial age that
great thing. Over and beyond the dangers with which it needed the constant supply of a compliant workforce with
threatens the present, it opens up as nothing else can, the a narrow set of capabilities. Our educational institutes
vision of a possible happy world; a world without poverty, resemble factories with bells, uniforms and batch-
without war, with little illness. Science, whatever unpleasant processing of learners, designed to get learners to conform.
consequences it may have by the way, is in its very nature But, from an economic point of view, the environment
a liberator. today is very different. It is a complex, volatile and globally
313. Which one of the following is the most important interconnected world.
implication of the passage? [Prelim II-2018] 316. With reference to the above passage, the following
(a) A happy world is a dream of science. assumptions have been made:
(b) Science only can build a happy world, but it is also 1. India continues to be a developing country essentially
the only major threat. due to its faulty education system.
(c) A happy world is not possible without science. 2. Today’s learners need to acquire new-age skill-sets.
(d) A happy world is not at all possible with or without 3. A good number of Indians go to some developed
science. countries for education because the educational
The Arctic’s vast reserves of fossil fuel, fish and minerals systems there are a perfect reflection of the societies
are now accessible for a longer period in a year. But unlike in which they function.
Antarctica, which is protected from exploitation by the Which of the above assumptions is/are valid?
Antarctic Treaty framed during the Cold War and is not [Prelim II-2018]
subject to territorial claims by any country, there is no (a) 1 and 3 only (b) 2 only
legal regime protecting the Arctic from industrialization, (c) 2 and 3 only (d) 1, 2 and 3
especially at a time when the world craves for more and The practice of dieting has become an epidemic; everyone
more resources. The distinct possibility of ice-free summer is looking out for a way to attain that perfect body. We
has prompted countries with Arctic coastline to scramble are all different with respect to our ethnicity, genetics,
for great chunks of the melting ocean. family history, gender, age, physical and mental and
81
spiritual health status, lifestyles and preferences. Thereby (c) India should take strict measures to control its rapid
we also differ in what foods we tolerate or are sensitive population growth.
to. So we really cannot reduce so many complexities into (d) India’s farming communities should switch over to
one diet or diet book. This explains the failure of diets other occupations to improve their economic
across the world in curbing obesity. Unless the reasons conditions.
for weight gain are well understood and addressed and Many pathogens that cause food borne illnesses are
unless habits are changed permanently, no diet is likely to unknown. Food contamination can occur at any stage
succeed. from farm to plate. Since most cases of food poisoning
317. What is the most logical and rational inference that can go unreported, the true extent of global foodborne illnesses
is unknown. Improvements in international monitoring have
be made from the above passage? [Prelim II-2018]
led to greater public awareness, yet the rapid globalization
(a) Obesity has become an epidemic all over the world.
of food production increases consumers’ vulnerability by
(b) A lot of people are obsessed with attaining a perfect making food harder to regulate and trace. “We have the
body. world on Our plates”, says an official of WHO.
(c) Obesity is essentially an incurable disease. 320. Which of the following is the most logical corollary to
(d) There is no perfect diet or one solution for obesity. the above passage? [Prelim II-2018]
Monoculture carries great risks. A single disease or pest (a) With more options for food come more risks.
can wipe out swathes of the world’s food production, an (b) Food processing is the source of all foodborne
alarming prospect given that its growing and wealthier illnesses.
population will eat 70% more by 2050. The risks are (c) We should depend on locally produced food only.
magnified by the changing climate. As the planet warms (d) Globalization of food production should be curtailed.
and monsoon rains intensify, farmlands in Asia will flood. I am a scientist, privileged to be some body who tries to
North America will suffer more intense droughts, and crop understand nature using the tools of science. But it is also
diseases will spread to new latitudes. clear that there are some really important questions that
318. Which of the following is the most logical, rational and science cannot really answer, such as : Why is there
crucial message given by the passage? something instead of nothing? Why are we here? In those
[Prelim II-2018] domains, I have found that faith provides a better path to
(a) Preserving crop genetic diversity is an insurance answers. I find it oddly anachronistic that in today’s culture
against the effects of climate change. there seems to be a widespread presumption that scientific
and spiritual views are incompatible.
(b) Despite great risks, monoculture is the only way
321. Which of the following is the most logical and rational
to ensure food security in the world.
inference that can be made from the above passage?
(c) More and more genetically modified crops only [Prelim II-2018]
can save the world from impending shortages of (a) It is the faith and not science that can finally solve
food. all the problems of mankind.
(d) Asia and North America will be worst sufferers from (b) Science and faith can be mutually complementary if
climate change and the consequent shortage of food. their proper domains are understood.
Directions for the following 4 (four) items : Read the following (c) There are some very fundamental questions which
four passages and answer the items that follow. Your answers cannot be answered by either science or faith.
to these items should be based on the passages only. (d) In today’s culture, scientific views are given more
importance than spiritual views.
Global population was around 1.6 billion in 1990—today
Today, the top environmental challenge is a combination
it is around 7.2 billion and growing. Recent estimates on of people and their aspirations. If the aspirations are more
population growth predict a global population of 9.6 billion like the frugal ones we had after the Second World War,
in 2050 and 10.9 billion in 2100. Unlike Europe and North a lot more is possible than if we view the planet as a giant
America, where only three to four percent of population shopping mall. We need to get beyond the fascination
is engaged in agriculture, around 47 percent of India’s with glitter and understand that the planet works as a
population is dependent upon agriculture. Even if India biological system.
continues to do well in the service sector and the 322. Which of the following is the most crucial and logical
manufacturing sector picks up, it is expected that around Inference that can be made from the above passage?
2030 when India overtakes China as the world’s most [Prelim II-2018]
populous country, nearly 42 per cent of India’s population (a) The Earth can meet only the basic needs of humans
will still be predominantly dependent on agriculture. for food, clothing and shelter.
319. Which of the following is the most logical and rational (b) The only way to meet environmental challenge is to
inference that can be made from the above passage? limit human population.
[Prelim II-2018] (c) Reducing our consumerism is very much in our own
(a) Prosperity of agriculture sector is of critical importance interest.
to India. (d) Knowledge of biological systems can only help us
(b) Indian economy greatly depends on its agriculture. save this planet.
EBD_7711
82

HINTS & SOLUTIONS


Explanations

1. (c) Total amount received is = 4 hrs. 48 min


i.e., 12 : 48 hrs
420 420 420
= ´5+ ´3+ ´2 5. (c) Let amount in each installment = x
2 3 6 Let total amount paid = L
= 210 × 5 + 140 × 3 + 70 × 2 According to Question.
= 1050 + 420 + 140 = ` 1610
2. (c) Let C.P of 1st goat is ` 100. 60
\ 18x = ´L
\ S.P. of 1st goat is ` 110 (10% profit) 100
\ S.P. of 2nd goat is ` 110 (Same S.P.)
L 100 ´ 18
Þ = = 30 = no. of installments.
100 x 60
\ C.P. of 2nd goat = ´ 110 (10% loss)
90 6. (b) Different sums of money can be formed by taking
one, two, three and all the four notes together.
1100
= No. of different sums = 4C1 + 4C2 + 4C3 + 4C4
9 = 4 + 6 + 4 + 1 = 15
1100 2000 7. (b) Given, n (U) = 100
\ Total C.P. = 100 + = Number of students who play cricket = 60
9 9
i.e. n(C) = 60
Total S.P. = 2 × 110 = 220
Number of students who play football = 30
æ 2000 ö i.e. n(F) = 30
20
ç 9 - 220 ÷ ´ 100 Number of students who play both the games = 10
è ø = 9 ´ 100 = 1% loss
\ Loss % = i.e. n(C Ç F) = 10
2000 2000
To find : n(C¢Ç F¢) = ?
9 9 we know,
Shortcut method: Loss % n(C È F) = n(C) + n(F) – n(C Ç F)
2
= 60 + 30 –10 = 80
æ common gain and loss% ö 100 n(C¢ Ç F¢) = n(C È F)¢ = n(U) – n(C È F)
=ç ÷ = = 1%
è 10 ø 100 = 100 – 80 = 20
8. (c) According to question, A, B, C and D each carry
3. (b) Time gap after which they will first hit the target
` 100.
is given by LCM of 6, 7, 8, 9, 12.
A ¾¾®
+20
B ¾¾®
-10
C ¬¾¾
-30
D
2 6, 7, 8, 9, 12
80 120 140 70
3 3, 7, 4, 9, 6
Here, option (c) is not correct. C has more than
2 1, 7, 4, 3, 2 what A and D have together.
1, 7, 2, 3, 1
LCM = (12 × 42) sec. 9. (c)
\ In 1 hr (= 3600 sec) no. of time they will hit together is
3600 50 1
= = = 7 times 45%
12 ´ 42 7 7 55%
30%
= 7 times in an hour. Magazine Magazine
4. (b) LCM of 18, 24, 32 A B
10%
LCM of 9, 12, 16
LCM = 3 × 4 × 3 × 4 25% 15%
= 144 min
144 2 40%
= = 2 hr
60 5 Magazine
= 2 hrs 24 min C
\ Bell will ring together again after 2 × (2 hrs 24 min)
83
Number of population who read the magazine \x =yÞy=x
= (45 + 55 + 40) – (30 + 25 +15) + 10 Now, put the value of y in equation (i), we get
= 140 – 70 + 10 Þ x = 6000
= 80% \ 4x = 24000
Number of population who do not read magazine 3x = 18000
= 100 – 80 = 20% Monthly Incomes of Peter and Paul are ` 24000
10. (a) Let the capacity of each of the equal glass = x litre and ` 18000 respectively.
Each glass containing x/3l and x/4l of milk. Sol. 14 & 15. E A C B/D F D/B
The quantities of milk in first and the second 22 21 20 19 18 17
x x 14. (b) F is 18 years old.
glasses were and respectively
3 4 15. (b) Two orders is possible in increasing age.
16. (d) Let ‘g’ be the cost of goats.
æx xö
= ç + ÷ 4g < Cow < 5g
è3 4ø Now, Given a goat price = ` 600
4 x + 3x 7 x 2400 < Cow < 3000 ... (i)
= = l Given, a goat price = ` 800
12 12
3200 < Cow < 4000 ... (ii)
So, the quantities of water in the first and the Hence, it can be seen that from above equations
second glasses were that cow cost is between ` 2400 and ` 4000.
æ xö æ xö 17. (c) Let initial population be100
= ç x - ÷ l and ç x - ÷ l Men Women
è 3ø è 4ø
¯ 60% ¯ 40%
2x 3x 60 40
= +
3 4 ¯ 70% ¯ 75%
42 30
8x + 9x 17x
= = l ¯ 80% ¯ 70%
12 12 33 21
7x 17x It is clear that more men cleared the examination
Ratio of milk and water = : than women.
12 12
18. (a) Given, length = x1m and breadth = x2m
7 Also, x1 + x2 = 40 where x1 and x2 are variables
=
or 7 : 17
17 We know that, of all the rectangles, a square has
11. (b) Let each question carry x marks. the largest area.
According to question For the given rectangle to be a square x1 = x2
So, x1 + x2 = 40 Þ x1 = x2 = 20 m
50
Þ 8x ´ = 40 \ Maximum area of the given rectangle = x 1x2
100 = (20 × 20)m2 = 400 m2
40 19. (b) Let x1, x2, x3, x4 and x5 be the present ages of the
x= = 10 family comprising of 5 members
4
3 years ago,
100 (x1 + x2 + x3 + x4 + x5) – 3 × 5 = 80
Numbers of question in the test = = 10
10 Þ (x1 + x2 + x3 + x4 + x5) – 15 = 80 ...(i)
12. (d) Let the son’s age = x Now, let the age of new born baby be x 6.
Age of Father = 9x Given,
Age of Mother = 8x Average age of family today = Average age of
Now, According to question family 3 years ago
8x + 9x = 51 (x1 + x 2 + x 3 + x 4 + x 5 ) - 15
Þ 17x = 51 Þ
5
x = 3 years
13. (a) Let the Income of Peter and Paul are 4x and 3x x1 + x 2 + x 3 + x 4 + x 5 + x 6
=
Let the Expenses of Peter and Paul are 3y and 2y 6
So, According to question
80 15 + 80 + x 6
4x – 3y = 6000 ... (i) Þ = [from (i)]
3x – 2y = 6000 ... (ii) 5 6
Now, From equation (i) and equation (ii) Þ x6 = 1
4x – 3y = 3x – 2y \ Baby’s age = 1 year
EBD_7711
84
20. (d) Let the basic pay of the first person be ` x and
æ 60 ö 40
that of second person be ` y. S’s 8 days work = çè1 - ÷ of work = of work
According to the question, 100 ø 100
Emolument of both the persons are same. 40 1 5
So, x + 0.65x = y + 0.8y Þ S¢s 1 day work ´ of work = of work
100 8 100
x 1.8 So, Ram’s 1 day work = (R + S)¢s 1 day work – S¢s
Þ x(1 + 0.65) = y(1 + 0.8) Þ =
y 1.65 1 day work

x 12 æ 15 5 ö
= or 12 : 11 = çè - ÷ of work
Þ 100 100 ø
y 11
21. (b) A person is standing on the first step from the 1
bottom of a ladder. = of work]
10
Now, he has to climb 4 more steps to reach exactly \ Ram will take 10 days to complete the entire job alone.
the middle step, 24. (b) By looking at all the options, we observe that
option (b) is not necessarily true.
9
We know that, a number is divisible by 10 iff it
8
has 0 at the unit’s place.
7
(R + S) may or may not have 0 at the unit’s place.
6
Middle Therefore, it may or may not be divisible by 10.
5 4 Thus, it is not necessarily true.
4 more steps 25. (a) From 100 to 199, there are 10 numbers ending
3 with 2. They are 102, 112, 122, 132, 142, 152, 162,
2 172, 182, 192.
Step 1 And from 200 to 300, there are 100 numbers
beginning with 2. They are 200, 201, 202...,299.
So, it is clear that the ladder have 9 steps. \ There are 110 numbers between 100 and 300
which either begin with or end with 2.
C
22. (b) B 26. (d) W can do 25% of a work in 30 days.
1
Þ W can do of a work in 30 days
4
12m \ W can do the complete work in 120 days.
1
X can do of the work in 10 days
4
\ X can do the complete work in 40 days.
A Y can do 40% of the work in 40 days.
A 5m D
Þ Y can do 100% of the work in 100 days
Fig. Vertical Trunk Fig. Broken Trunk \ Y can do the complete work in 100 days.
According to pythagoras theorem,
CD2 = AD2 + AC2 1
Z can do of the work in 13 days.
3
Þ CD2 = 52 + 122 Þ CD2 = 25 + 144
\ Z can do the complete work in 39 days.
Þ CD = 169 = 13 m Hence, it is clear from above results that Z will
Now, AB = AC + CD (Q CD is the broken part of complete the work first.
the trunk AB touching ground at D) 27. (a) Average monthly income of a person in family of
Þ AB = (12 + 13) m = 25 m 5 = ` 10,000
23. (c) Let the work done by Ram and Shyam be R and S \ Monthly income of family = ` 10,000 × 5 = `
respectively. 50,000
After Increament
60
Given, (R+S)¢s 4 days work = of work Increase in monthly income of one person
100
1, 20, 000
60 1 15 = = ` 10, 000
Þ (R + S)¢s 1 day work = ´ of work = of 12
100 4 100 Now, the average monthly income of a person in
work 50, 000 + 10, 000
After Ram takes leave : family = = ` 12,000
5
85
Þ 1.1 x = 693 Þ x = 630
28. (c) \ Cost price of the toy = ` 630
MP = 770
90
SP after discount = ´ 770
5m 3m 3m 3m 3m 3m 100
Distances covered by the competitor to collect the 100 90
apples in the bucket are as follows: CP getting 10% gain = ´ ´ 770 = 630
110 100
1st apple : 2(5) = 10 m
2nd apple : 2(5 + 3) = 16 m 31. (a) Class start at 11:00 am
3rd apple : 2(5 + 2 × 3) = 22 m Class ends at 02:27 pm
4th apple : 2(5 + 3 × 3) = 28 m So, duration of class = 3h 27min = 207 min
5th apple : 2(5 + 4 × 3) = 34 m There will be 4 periods.
6th apple : 2(5 + 5 × 3) = 40 m After every period, there will be a 5 min break.
Therefore, total distance covered = 10 + 16 + 22 + Here, only 3 breaks will be taken as there are only
28 + 34 + 40 = 150 m 4 periods and end of 4th period means end of the
class. 4th break is not needed to be considered.
29. (c) So, duration of the periods = 207 – 3×5 = 192 min.
192
\ Time assigned to each period = = 48min .
4
R B Y W 32. (d) Vessel A : 30g sugar mixed in 180ml water.
So, the concentration of sugar in vessel A is
30 1
g / ml i.e., g / ml .
180 6
Vessel B : 40g sugar mixed in 280ml water.
We know that, area of a circle = pr2 So, the concentration of sugar in vessel B is
So, Red band area = p(0.2)2 = 0.04p
40 1
All the other than red are in the form of a ring. So, g / ml i.e., g / ml .
Blue band area = p((0.3)2 – (0.2)2) 280 7
= p((0.3 – 0.2) (0.3 + 0.2)) = 0.05 p Vessel C : 20g sugar mixed in 100ml water.
Similarly, yellow band area = 0.07 p So, the concentration of sugar in vessel C is
And white band area = 0.09 p 20 1
\ Required probability g / ml i.e. g / ml .
100 5
0.04p More the concentration of sugar, more will be the
= = 0.16
( 0.04 + 0.05 + 0.07 + 0.09 ) p sweetness.
Alternate Method: Therefore, in terms of sweetness : C > A > B
Area of red circle = p(20)2 33. (b) Let the number of students in the class be x.
Total c ollec tion without considering the
1 additional contribution by one student = 443 – 2
Radius of Archry board = m = .50m
2 = ` 441
Area of Archry Board = p(.50)2 So, x × x = 441
Þ x2 = 441 Þ x = 441 = 21
p(.20) 2 0.400
P(Î) = 2
= = 0.16 34. (b) Questions correctly answered by Anita are as
p.(50) 0.2500
follows:
30. (c) Marked price (M.P.) of toy = ` 770 Arithmetic – 70% of 10 = 7
10 Algebra – 40% of 30 = 12
Discount = 10% of MP = ´ 770 = ` 77 Geometry – 60% of 30 = 18
100
\ Total questions correctly answered = 7 + 12 +
Price after discount = ` (770 – 77) = ` 693 18 = 37
Let cost price (C.P.) of toy be ` x. Anita had to attempt 60% of 70 questions i.e., 42
According to Question, questions correctly to pass the test.
10 \ Number of more questions required to be
693 – x = of x answered = 42 – 37 = 5
100
Þ 693 – x = 0.1 x
EBD_7711
86
= 4.5 cm.
3
35. (b) Here, ´ x = 18 Hence, Length of pillar A = 5 × 39 + 6 = 201 cm
4 Length of pillar B = 4 × 39 +7 = 163 cm
x = 24 Length of pillar C = 4.5 × 39 + 6.5 = 182 cm
2 Length of shortest pillar ® B. 163 cm.
Boys = 24 and ´ y = 24 41. (c) Remaining names after the removal of failed students
3
= 52 – 15 = 37.
y = 36
Ramesh’s position from top = 22.
Total students = 36
Hence from bottom Ramesh is (37 + 1 – 22)th = 16th.
The number of girls in the class = 36 – 24 =12.
42. (c) Let the original price of phone = `100
36. (b) Let the number of employees in the company be
100 and the number of male employees in the 110
company be x. SP of Gopal = 100 ´ = `110
100
So, the number of female employees = 100 – x. SP of Ram = 110 – 10% of 110 = `99
According to the question, Total Profit of gopal = `100 – `99 = 1
5200x + 4200 (100 - x) But profit was made as CP hence, 1% profit
= 5000 43. (c) The weight of the 5th person = sum of first 5 person
100
+ Sum of last five person– Sum of weight of 9 persons.
Þ 52x + 42 (100 - x) = 5000 = 5 × 45 + 5 × 55 – 9 × 50 kg = 50 kg
Þ 52x + 4200 - 42x = 5000 44. (d) None of the above statement is correct.
45. (b) 12% earn less than ` 30,000.
Þ 10x = 800 Þ x = 80 6% earn more than ` 2,00,000.
So, there are 80% male employees in the company. 22% earn more than ` 1,00,000.
37. (b) Let the third number Z = 100 So, between ` 30,000 – ` 1, 00, 000 there are (100
So, X = 80, Y = 72 – 22 – 12)% = 66% household.
\ Percentage by whic h Y is less than X 66% household is equal to 990
80 - 72 Hence, Number of household = 1500.
= ´ 100 = 10% Number of household between 1,00,000 and 2,00,000
80
38. (d) Population of the Community after every year 16
= 1500 ´ = 240.
100
2 46. (b) At 5 O’clock the clock will strike 5 times. It’s
= of the previous year. Hence, the population
3 given that it takes 12 seconds to do so.
Now, at 10’O clock the clock will strike 10 times.
6
æ2ö Hence, the time taken by it to strike = 12 × 2 = 24.
of the community after 6 year = ç ÷ of the 47. (a) Since 39 is the mean of first five numbers, first
è3ø
original population of the Community. five numbers are
35 , 37, 39, 41 and 43 and next 8 numbers are
æ 2ö
6
64 45, 47, 49, 51, 53, 55, 57 and 59.
ç ÷ = th part.
è 3ø 729 (13 + 1)
Hence, mean = , 7th number or middle
39. (b) A + B > C + D .....(i) 2
A+C=B+D .....(ii) number = 47.
B+D 48. (a) Let the original amount be 150 ml.
A= According to question.
2
50 ml of mixture + 50ml of pure milk
B+ D Þ 25 ml of Milk + 25ml of water + 50 ml of pure
So, C = or, A = C ....(iii)
2 milk.
From (i) and (iii) Hence, % of water in new mixture
B>D ....(iv)
25
Using (i), (ii), (iii) and (iv), = ´ 100 = 25% .
B > A = C > D. 100
Hence, B’s income is highest. 49. (c)
40. (b) Distance travelled by A in one chance = (6 –1) cm
= 5 cm
Distance travelled by B in one chance = (7 – 3) cm
= 4 cm
Distance travelled by C in one chance = (6.5 – 2) cm
87
Number of squares and rectangles in a 3 × 3 grid 59. (d) Let the two digit numbers are of the form ab.
= 13 + 23 +33 = 36. So, 10a + b – (10b + a) = 27
50. (d) Randhir > (Kunal + Debu) ------ ... (i) Þ 9a – 9b = 27
(Kunal + Shankar) > Randhir ----- ... (ii)
Sonal > Shankar ------ ... (iii)
Þ a–b =3
Neha > Randhir ------ ... (iv) (a, b) Þ (9, 6), (8, 5), (7, 4) (6, 3) (4,1), (5, 2)
From (i), (ii), (iii) and (iv) we can not conclude Hence, 96 , 85 , 74 , 63 , 41, 52 and 25, 14, 36,
who optained the highest 47, 58, 69 are the required numbers.
mark hence, Data are inadequate. 60. (b) Number of digits used For 1 – 9 = 9 × 1 = 9
51. (b) Let the numbers are of the form abc. Number of digits used For 10 – 99 = 90 × 2 = 180
So, According to question, Number of digits used for 100 – 150 = 51 × 3 = 153
100 a + 10b + c = 7K ....(i) Hence, 153 + 180 + 9 = 342 digits.
100c + 10b + a = 7m .....(ii) 61. (b) Let initial dimensions be, l & b
From, (i) – (ii) \ Final length is 1.4 l
99a – 99c = 7 (k–m) Final breadth is 0.8 b
99 (a – c) = 7n \ Final area is = 1.4 l × 0.8 b
a–c =7 = 1.12 lb
a = 9, c = 2 \ Area is increased by 12%.
a = 8, c = 1 40 ´ ( -20 )
Hence, 4 numbers, 259, 952, 168 and 861. Shortcut Method : + 40 – 20 +
100
52. (c) — —8
= 20 – 8 = 12%
There are 9 values (1 to 9) for hundreds place
Therefore, the area of the new garden increased by 12%
digit. While 10 values (0 to 10) for ten’s place
digit. 62. (d) X U Z
Hence, 9 × 10 = 90 Numbers. U
53. (d) Skewed to the left.
54. (b) Mr. x’s present age 26, because 25 was a perfect W
square and the next year would be a perfect
cube. V
Next cube number ® 64 Y
Hence, minimum years required = 64 – 26 = 38
years. From above diagram, it is clear that Y, V and W
are parallel.
55. (a) Let the work done by Q in 1 day = x units.
63. (a) Volume of cylindrical overhead tank = pr 2h =
So the work done by P in 1 day = 3x units
22
3x + x ´ (2)2 ´ 7 = 88 m3
Work done by R in 1 day = = x units 7
4 Volume of underground tank = (5.5 × 4 × 6)m 3 =
P Q R 132m 3
Hence, the ratio of earnings = 3 x : x : x Portion of underground tank still filled with
= 3 : 1 : 1. water after filling the overhead tank completely.
56. (d) 94 divided by 49 leaves the largest remainder 45. =
Volume of underground tank – Volume of overhead tank
57. (b) Let the monthly incomes of x and y be 4x and 3x. Volume of underground tank

4x – 6000 3 132 - 88 44 1
So, = = = =
3x – 6000 2 132 32 3
64. (c) Area of rectangle = 12 × 8 cm2 = 96 cm2
Þ 8x – 12000 = 9x – 18000 Rectangle is used to construct a closed cube.
\ x = ` 6,000 Surface area of cube formed = Area of given
Total income of x and y = (3 + 4)x rectangle.
= ` 6,000 × 7 = ` 42,000 6 (side)2 = 96 cm2
58. (a) 9 = (8)2 + (1)2 + (d)2 Þ side = 16 Þ side = 4 cm.
65. (a) Number of Colour less cubes = (4 – 2)3 = 8.
Þ 81 = 64 + 1 + (d)2 66. (d) Let no. of column = x, no. of rows = y
Þ (d)2 = 16 \ xy = 630 – [3 × 1 + 3 × 2 + ......... + 3 × (y – 1)]
d = 4. = 630 – 3 [1 + 2 + ........ + (y – 1)]
EBD_7711
88
Total number of ways = 2C + 5C
3 ( y - 1) y 1 2
xy = 630 – 5´ 4
2 = 2+
621 2´1
(a) If y = 3, then 3x = 630 – 9 Þ x = = 207 = 2 + 10 = 12
3 Number of possible combinations of selectres
612 = 2 × 10 = 20
(b) If y = 4, then 4x = 630 – 18 Þ x = = 153 72. (c) It mathematics I is not opted, then two subjects
4
out of four subjects have to be opted for.
600 \ Number of ways in which two subjects can be
(c) If y = 5, then 5x = 630 – 30 Þ x = = 120
5
4´3
67. (d) We cannot predict the number of dinners for a opted for =6
particular member from the given data. It may 2
be possible that by choosing members from If mathematics II is opted, then it can be offered
only if mathematics I is also opted for Number of
picking lots, one may have to host a dinner more ways in which two subjects can be opted for
than one times. = 6 +1 = 7.
68. (c) Here are five persons, and 5 tasks 73. (d) Let number of a pairs of brown socks = y
So, When T2 task is fixed for person 3 Price of brown socks = x
Task Price of black socks = 3x
According to question
T2 Þ 5 ´ 3x + yx = 100 ... (i)
1 2 3 4 5 Now, clerk has interchanged socks pairs then
For Task 1 no. of ways = 2 price is increased by 100%
Task 2 no. of ways = 1 (15 x + yx) ´ 100
Task 3 no. of ways = 3 3xy + 5x = (15 x + yx) +
100
Task 4 no. of ways = 3 Þ 3xy + 5x = 30x + 2xy ... (ii)
Task 5 no. of ways = 3 Þ 30x + 2yx = 3xy + 5x
Total no. of ways for condition = 3 + 3 + 3 + 2 + 1 Þ 25x = xy
= 12 y = 25
Condition II \ So, number of brown socks = 25
When task T2 is given to be person 4 74. (*) Data Inconsistent.
75. (a)
Task 76. (b) Considering the worst case First 15 picked balls
T2 are red and white. Hence in 16th time it is
1 2 3 4 5 assured that 3 different colour balls exist.
No. of ways for Task T1 = 2 77. (c) — B1 — B2 —
No. of ways for Task T2 = 1 2 boys can take their seats in 2! ways and 3 girls
No. of ways for Task T3 = 3 can take the remaining 3 seats in 3C2 ´ 2! ways.
No. of ways for Task T4 = 3 Hence, 2! ´ 3C2 ´ 2! = 12 ways.
No. of ways for Task T5 = 3 78. (b)
Total number of ways for condition II B
= 3 + 3 + 3 + 2 +1
= 12
Total number of ways for condition I and II = 12 + 12
2
= 24 Shortest Distance, BC = 5 + 12 = 13 km
2
69. (c) Let x be the number of women. 5
Let y be the number of men.
Total number of hand shakes = xy = 24
Then, the possible factors of x and y are x = 6 or 4, A C
y = 4 or 6 12
Number of hugs = x C 2 + y C 2
79. (a) 50 kmph 30 kmph
= 6 C2 +4 C2 C
A B
6´5 4´3
= +
2´1 2
= 15 + 6 = 21
70. (b) White Marbles Red Marbles 160 km
10 13 Suppose the cars meet at point C after ‘t’ hrs.
White Marbles Green Marbles \ AC = 50 t and BC = 30 t
10 . 0 5 \ 50 t + 30 t = 160
Now, total number of Marbles = 5 + 10 + 13 = 28 160
71. (d) Number of ways to select Principal = 2C1 t= = 2 hrs.
Number of ways to select Vice Principal = 5C2 80
So, the cars will meet at 10 : 10 AM
89
80. (c) 1 2 3 .......... n 2 ´ 40 ´ 60
=
60 + 40

2 ´ 40 ´ 60
36 cm =
100
\ 1 + 2 + 3 + ........ n = 36
= 48 km/hr
n ( n + 1) 30 ´ 20
= 36 84. (b) Time taken by both tanks =
2 30 + 20
n(n + 1) = 72
30 ´ 20
Þ n=8 =
50
\ Leaving the end points the no. of points starting
from A is 7. = 12 min
85. (a) Here, A and B are moving in opposite directions.
Similarly starting from B, the no. of points will be 7.
So, Relative speed = 2 + 3 = 5 rounds/hour So, they
And also no. of the points between A & B will coincide. cross each other 5 times in an hour and 2 times in
\ Total no. of points half an hour.
= 7 + 7 = 14 Hence, they cross each other 7 times before 9 : 30
81. (c) Let cost of 1 ticket is ` x. a.m.
80 86. (a) Here, A takes the shortest time to cross the bridge
\ for A, 10 = ´ 2x i.e. 1 min. And, D takes 10 min, C takes 7 min and
100
B takes 2 min to cross the bridge.
50 25 So, 4 friends can cross the bridge in minimum
Þ x= = = ` 6.25
8 4 time in the following ways.
Now B gives ` 3 to A. 1. A + B crossing and A returning : (2 + 1) min
So, A has 13 `. 2. A + C crossing and A returning : (7 + 1) min
Cost of 2 tickets = ` 12.50, which is more than enough 3. A + D crossing and A returning : (10 + 1) min
to buy 2 tickets.
4. A crossing the bridge for the last time : 1 min.
\ (a) Rules out
Hence, total minimum time = 3 + 8 + 11 + 1 = 23
(b) is clearly ruled out min.
(c) is true, because after buying 2 tickets.
A is left with ` 13 – 12.50 = 50 paise 87. (c)
82. (a) Total distance = 500m A B
st
Distance covered by A = 500m 6 am Fig. 1 day 6 am
Distance covered by B = 500 – 45 – 35
= 500 – 80 Train 2 Train 1
A B
= 420 nd
6 am Fig. 2 day 6 am
distance of A
speedof A time Above given figures show that train 1 leaves
= station A on 1st day and train 2 leaves station B
speed of B distance of B
time
on 1st day. Both the trains don’t reach their
destination even on the 2nd day after 24 hours as
500 time the journey completes in 42 hours.
= ´
time 420 \ Two more trains need to be introduced at both the
stations i.e., station A and B. So, 4 trains are needed in
500 order to maintain the shuttle service.
=
420 88. (c) Distance travelled by freight train in 2 hour = 2 ×
25 40 = 80 km.
=
= 25 : 21 Relative speed = 60 km/hr – 40 km/hr = 20 km/hr
21
Hence, time taken by express train to meet
83. (b) Average speed of two cars at a speed V1 and V2
80 km
2V1 V2 Freight train = = 4 hr.
km/hr = 20 km / hr
V1 + V2
So, Distance travelled = 4 × 60 or 6 × 40 = 240 km.
EBD_7711
90
89. (b) From the given options, in 1996, the graphs of A 101000
& B intersect, which shows, same average profit. In 1992 ® p.c.i = = 5050 > 5000
20
\ Ans - 1996
90. (c) From the graph, we observe that the difference 111100
between points A & B is more than half of 1000, but In 1993 ® p.c.i = > 5000
21
less than 1000.
122500
In 1994 ® p.c.i = > 5000
22

4 B 134500
In 1995 ® p.c.i = > 5000
3 A 23
Option (c) is correct.
95. (a) The product of price (in `/litre) and the monthly
consumption (in litres) in constraint is equal to 2400.
Expected consumption when the price goes up to
\ 500 < |Difference between A and B| < 1000. 2400
` 80 per litre = = 30 litres
From the option, + 600 satisfies this condition. 80
91. (d) In 2000, Profit of A = 6000 96. (a) From given years 2000 to 2007
In 1999, profit of A = 4000 Earnings of A > Earnings of B
\ Difference = ` 2000 So,
92. (d) Fluctuating Average of A > Average of B
93. (d) For A Agg marks = 60 + 70 + 50 + 30 = 210 Hence, It can be conducted that the average A
Agg max marks = 100 + 100 + 100 + 50 = 350 earned more than B during this period.
210 97. (c) From the graph commodity price are between `
% Agg marks of A = ´ 100 = 60% 10 and ` 20.
350
For B: 50
98. (c) A spent on food = 20, 000 ´
Agg marks = 80 + 70 + 60 + 15 = 225 100
Agg max marks = 150 + 100 + 100 + 25 = 375 = `10000
225 10
% Agg marks of B = ´ 100 = 60% B spent on food = 1,00, 000 ´
375 100
Difference in agg percentage = 0 = `100, 00
Note: Difference in agg percentage is same as difference So, Both A and B spent same amount on the food.
in mean agg percentage. 99. (c) Total passing marks = 30 + 30 = 60
94. (c) Rule out options:- Percentage of passing marks = 40%
Option (a): - We clearly see that from 1993 to 94, Let total marks be x
æ 1 ö According to the question
increase in pop is less than 5% ç ´ 100 < 5 ÷
è 21 ø 40
x´ = 60
Option (b): - from year 1994 to 1995 100

1345 - 1225 60 ´ 100


Increase in income = ´ 100 \x= = 150
1225 40
100. (*) At 5’ o’clock it strikes 5 times. Total gaps between
120 480 1 to 5.
= ´4 = < 10
49 49 (1 – 2 – 3 – 4 – 5) Total time for 4 gaps.
\ Not true 12
Option (c): per capita income (p.c.i.) So, time taken for each gap = = 3sec.
4
Total Income in an year At 10’o clock, it strikes 10 times. Total gaps = 9
= Hence, 9 × 3 = 27 seconds.
Total population in that year.
101. (d) Checking statement 1
x–y=8
Both x and y must be positive for any value of x and y.
91
It is not true because negative values of x and y or 105. (b) 18 Adults = 30 children (given)
positive x and negative y is also possible 5
Checking statement 2 3 Adults = 5 children \ 1 adult = children
3
If x is positive, y must be negative for any value of 12 Adults + x children = 18 Adults
x and y.
5 5
It is alos not true because positive values of x and 12 × children + x children = 18 × children
3 3
y are also possible.
x = 10 children
Checking statement 3 106. (d) Length of train = distance = 200 m
If x is negative, y must be positive for any value of x and y.
5 100
Here if x is negative then only negative value of y is Speed of train = 40 × = m/s
18 9
possible.
Hence, neithter 1 nor 2 nor 3 100
Required time = 200 ÷ = 18 sec
102. (d) From question, 9
x Î[–3, –1] Þ 1£ x2 £ 9 107. (a) B is 30° head to A
Total angle covered by A =180°
and y Î [–1, 1] Þ 0 £ 42 £ 1
Total angle covered by B = 180°–30° = 150°
1 – 1 £ x2 – y2 £ 9 – 0 Required ratio = 180° : 150 = 6 : 5
0 £ x 2 – 42 £ 9 108. (c) Diagonal of rectangle = 5 cm
Thus (x2 – 42) Î [0, 9]
103. (c) Selling price of article = 40 l 2 + b2 = 5
Profit = x% then velue of l and b must be 4 and 3
Area of ractangle = 4 × 3 = 12 cm2
100 4000
Cost price = 40 × = Area of square be a2
100 + x 100 + x 12 × a2 = 12
Again
\ a2 = 1 sq.cm
Selling price of article = 20
Loss = x% 109. (c) Players wearing
Hockey Shirts and Pants (S+P)
100 2000
Cost price = 20 × = 14-x x 11-x
100 - x 100 - x
Now
Players wearing hockey Pants (P)
4000 2000 Players wearing hockey Shirts (S)
=
100 + x 100 - x Using venn diagram the given information can be
100 - x 1 representd as follows:
= Let the number of players wearing both hockey shirt
100 + x 2
and pant be x
Þ 100 + x = 200 – 2x
\ n (st+1p) = x, n (s) = 14 – x
1 n (p) = 11 – x
Þ 3x = 100 \ x = 33
3 n (T) = 19 (\ T = Total number of players)
100 \ n (T) = n (s) + n (p) + n (S + P)
300 \ 19 = 14 – x + 11 –x + x
\ Cost price = 40 × 100 + 100 = 40 × = `30
3 400 \ 19 = 25 – x
104. (d) Amount to be paid after 1 year \ x = 25 – 19 = 6

12.5 6!
= 22800 + 22800 × = 22800 + 2850 = 25650 110. (a) Required number of ways = = 120
100 3!
He paid 8650 then amount left 111. (c) Each equilateral triangle is made by joining the three
= 25650 – 8650 = 17000 points as in the figure given below. Circle is divided
Amount to be paid after 2 years into three sectors.

12.5
=17000 + 17000 × = 17000 + 2125 = 19125
100
He paid 9125 at the end of 2nd year then remaining
amount = 19125 – 9125 = 10000
Amount to be paid at the end of 3rd year
12.5 24
= 10000 + 10000 × = ` 11250 Hence, required number of triangles = =8
100 3
EBD_7711
92
112. (a) Number of diagonals in an octagon = 8 C2 – 8 Total scrap steel imported in first three months
= 32 + 34 + 32 = 98
8´ 7 Required ratio = 113 : 98 = 1.2:1
= – 8 = 20
2 120. (d) According to the given graph, during mid of the
113. (c) Hundred Corresponding Corresponding August, actual progress was more than expected
Digit Ten’s Digit Unit’s Digit progress.
7 1 0 ü 121. (a) According to the graph, death rate is decreased faster
2 0, 1 ï
ï than birth rate, therefore population growth rate has
3 0, 1, 2 ïï increased.
4 ý 21 122. (d) After 2005 birth rate and population growth has no
0, 1, 2, 3 ï
5 0, 1, 2, 3, 4 ï change.
ï 123. (c) We can see from graph slopes of both graphs are same
6 0, 1, 2, 3, 4, 5 ïþ
124. (a) Manufacturing cost of 1000 pieces = 600000
8 1 0 ü
2 ï Cost of 1 piece when 1000 pieces manufactured
0, 1 ï
3 0, 1, 2 ï 600000
4 ï = = 600
0, 1, 2, 3 ý 28 1000
5 0, 1, 2, 3, 4 ï
ï Selling price of 1 piece when 1000 pieces manufactured
6 0, 1, 2, 3, 4, 5 ï = 400
7 ï
0, 1, 2, 3, 4, 5, 6þ Manufacturing cost of 2000 pieces = 700000
9 1 0 ü Cost of 1 piece when 2000 pieces manufactured
2 0, 1 ï
3 ï 700000
0, 1, 2 ï = = 350
4 ï 2000
0, 1, 2, 3 ï
5 ý 36 Selling price of 1 piece when 2000 pieces manufactured
0, 1, 2, 3, 4 ï = 350
6
0, 1, 2, 3, 4, 5 ï
7 ï Hence, required number of pieces = 2000
8 0, 1, 2, 3, 4, 5, 6 ï
125. (c) Dependency Ratio
0, 1, 2, 3, 4, 5, 6, 7 ïþ
So, total numbers = 21 + 28 + 36 = 85 æ Sum of the number of people aged less ö
114. (b) Total number of balls 15 + 20 = 35 ç ÷
è than is 15 years and more than 64 years ø ´ 100
Total number of red balls numbered 3 =
( Number of people aged from 15 to 64 years )
æ 100 - 20 - 40 ö 40
= 15 × çè ÷ø = 15× =6 1. In country B, over the last two and a half decades
100 100
(2025 to 2050), the sum of the number of people aged
Total number of black ball numbered 1 or 2
less than 15 years and more than 64 years is slightly
æ 100 - 30 ö 70 increased but number of people aged from 15 to 64
= 20 × çè ÷ = 20 × = 14
100 ø 100 years increased tremendeously. Therefore
6 + 14 4 dependency radio of country B has decreased over
Required chances = =
35 7 the last two and half decades.
115. (d) According to the table tele-density is similar to others
116. (d) There are many points related to GDP of state and Country A
tele-density is similar to others according to table.
Population (in millions)

1.2
117. (c) Total import of sheet steel in first three months 1.0
= 40 + 37 + 36 = 113 0.8
Total import of coil steel in first three months 0.6
= 30 + 31 + 33 = 94 0.4
Required velue = 113 – 94 = 19. 0.2
0
118. (c) Total sheet steel important over six months period
1950 1970 1990 2010 2030 2050 2070 2090
= 40 + 37 + 36 + 36 + 34 + 34 = 217
Total velue = 217 × 256 = 55552 » 55550
119. (b) Total sheet steel imported in first three months
= 40 + 37 + 36 =113
93
129. (c) According to the passage there should be changes in
Country B lifestyle; especially the overuse of technology should
Population (in millions)

1.2
be stopped at all costs as stated in statement (3). There
1.0
should be less consumption of animal protein that is
0.8
0.6
meat. The FAQ or the Food Administration
0.4 Organization claims that nearly 18% of greenhouse
0.2 gases are emitted from livestock. So statement (1) is
0 also true.
1950 1970 1990 2010 2030 2050 2070 2090 130. (a) As Indians we continue to depend on fossil fuel heavily
because of the lack of investment in research and
2. By the end of next two and a half decades (i.e. 2075), development in other forms of energy which are so
sum of the number of people aged less than 15 years readily available. The lack of technological development
and more than 64 years of country A is more than prevents us from harnessing the resources of wind,
that of country B where as number of people of aged solar and biomass energy, which are readily available.
from 15 to 64 years of country A is less than that of 131. (d) According to the passage the mitigation of greenhouse
country B. gases cause lower air pollution, this will lead to better
Hence by the end of next two and a half decades, the health among the public, as a result there will be less
dependency ratio of country A will be more than that expenditure on public health as stated in statement (1);
scientists have already predicted that the emission of
of country B.
greenhouse gases will have disastrous impact on climate
3. In the next two decades (i.e. 2050-2070), graph of change in the next 30 years and worst still after that. So
work-force (i.e. population of aged from 15 years to statement (4) is also correct.
64 years) of country A is going lower and that of 132. (b) The essential message conveyed by the passage is
country B is going higher. that it is absolutely essential to reduce to a very great
Therefore, in the next two decades, the work-force extent the emission of greenhouse gases into the
relative to its total population will increase in country atmosphere.
B as compared to country A. 133. (c) In this passage the author wants to convey that the
best way to ensure the prosperity and further economic
126. (d) Repo rate and reverse repo rate is directly related to
growth of the nation is by implementing inclusive
Anti-inflationary stance
growth where the benefits of growth are shared by the
127. (a) entire population and not by certain segments only.
1. Corporate tax (……) and personal tax (-----) are direct 134. (c) To make the process of inclusive growth more effective
taxes. From graph it is clear that corporate tax is the author suggests that we should not only
increasing tremendeously whereas personal tax concentrate on the poorer sections of society, but also
increased slightly. ensure that the rich and middle class stand to gain
Exercise ( ), customs ( ) and service tax ( ) certain points. In this way it will be possible to
redistribute the economic gains made by these sections
are indirect taxes. From the graph, it is clear that
of society to those getting left behind.
exercise and customs decreased tremendeously
whereas service tax is increased. 135. (d) According to the passage eradication of poverty is not
a sufficient condition for growth. So statement (1) is
Therefore, during the given period, the revenue from ruled out. Growth has to be treated as an instrument for
direct taxes as percentage of gross tax revenue has ensuring prosperity for all. But this is yet to be attained.
increased while that of indirect taxes decreased. So India has to strive on. So statement (2) is also ruled
2. From the graph, it is clear that exercise duty is over out.
all decreased tremendeously. But after 2010, exercise 136. (b) If the price of crude oil is x and the exchange rate is y,
duty increases slightly. This traineds of exercise duty and if such a transparent formula is set in place by the
has many reasons. One of then may be decreasing government the oil companies can make profits by
rate of exercise duty to promote manufacturing sector innovating within the rules and regulations of anti- trust
thats why after 2010 exercise duty increases slightly. and other competition laws. To ensure greater profits
they will also have to find out ways to cut costs. So
1/2(PL) × (OL) PL statements (2) and (3) have been selected.
128. (c) Required Ratio = =
(OC) × (OL) 2(OC) 137. (a) According to the passage private oil companies can re-
enter the oil producing market if a transparent rule based
1 petrol pricing exists because they will be able to
LD + DL
PD + DL 2 3( DL) 3 innovate, cut their costs and earn more profits which is
= = = = {Q DL = OC}
2(OC ) 2(OC ) 4(OC ) 4 an attractive incentive for any business.
EBD_7711
94
138. (d) According to the passage due to the adverse impact of between man and State. Moreover the passage
climatic change on the ecosystem can be a possible confirms that this is a continuous tussle and is as
loss of certain species of animals and their habitats and old as the state history. The solution seems to be'
also a decrease in the services of the ecosystem. (d) still far off suggests that the conflict will remain
option is correct. unresolved.
139. (b) The passage clearly states that the climatic change may (b) and (d) are wrong as they present only the idea
have an adverse effect on the Himalayan ecosystem, contained in the last line of the passage.
by altering temperatures, changing precipitation (c) is wrong as the passage clearly mentions that old
patterns , leading to drought and consequently the values and ideas constantly yield place to new
death of several species of animals and plants including ones which is contrary to what is mentioned in
humans. (c). Moreover it is not the author's viewpoint.
140. (b) The most important message conveyed in the passage 148. (b) (a) is wrong as it is a general statement and does
is summed up in the last few lines. The idea is to shift present the crux of the passage.
the attention from merely the species-habitat focus to (c) is wrong as the opening line of the passage says
the entire biographical range meaning all the plants and that people talk about democracy (or understand
animals including humans so that climatic adjustments democracy) but they have a weakness favouring
can be taken care of more effectively. their caste or community or religion.
141. (b) It is clear from the passage that not only human activity (d) is wrong as it projects an extreme situation which
is causing environmental pollution, but also natural is out of the scope of the passage.
reasons can adversely affect climate systems. This is (b) is the most appropriate statement as it talks about
clearly stated in the first four lines of the passage. providing equal opportunities to all irrespective
142. (a) According to the passage government interference of the caste or community or religion. That's what
leads to distortions and inefficiency in the economy in is a True democracy.
the sense that there is room for corruption as well as a 149. (b) Clearly only 2 is correct. 1 is wrong as the passage says
lack of interest in investment on the part of the that 'Such access is not always available to all people
entrepreneurs. in developing countries like India and more so, in rural
143. (c) The first paragraph states that the basic philosophy of areas.' This means that they are sometimes available.2
globalization is to ensure absolute freedom for the is correct as mentioned in the last sentence of the
markets, to set their prices, produce their goods, and passage.
distribute them as per their own criterion. 150. (d) The crucial message conveyed in the passage is to
144. (c) The passage clearly states that in accordance with the promote financial inclusion such that every citizen of
conditions set by the WTO. etc. for globalization, public the country has access to bank accounts and can save
sectors should be privatized. So statement (1) is correct. money so as to have better finances . Establishing more
Employment and wages should be conditioned by the banks is a route to financial inclusion. Increasing the
free play of the market forces involved, otherwise it interest rate of bank deposits would promote savings.
might discourage investment as stated in statement (3). So (d) is correct.
Even social services like heath and education should 151. (b) The passage suggests the biggest hurdle in the world
welcome private players as is correctly expressed in agriculture is to maintain a demand - supply equilibrium.
statement (4). Based on the relevant information provided in the
145. (b) The entire passage focuses on the fact that the state passage, it can be ascertained that regulating the pricing
should play a reducing role in the process of component for ensuring affordability to the poor and a
globalization. This is elaborated in the last few lines of proper marketing mix would be beneficial.
the passage with particular reference to India. 152. (d) The biggest challenge that has always haunted world
146. (d) (a) is wrong as it states a fact mentioned in the agriculture is to achieve a balance the demand and
passage. It does not sum up the passage. supply for foods.
(b) is wrong as the passage does not restrict the 153. (c) To reduce hunger and starvation, ensuring equilibrium
problem to the developing countries only. between demand and supply of food is mandatory,
(c) is wrong as the passage only says that the food alongside ushering measures for Purchasing Power
production will have to double. It does not talk Parity, to help the poor consumers too.
about food scarcity at all. Rather it talks about 154. (b) Since there is disparity in the sustainable provision
how to meet the required target. and supply for food worldwide, this issue has gained
(d) correctly sums up the passage. Food security is significance.
increasingly a collective challenge and the 155. (c) The key responsibility areas of the civil service officers
passage suggests means to combat it. need to be clearly defined and standardized, based on
147. (a) (a) is correct as the passage talks about the conflict the political agendas. Lack of such clarity would lead
95
to misgovernance. Also, striking equilibrium between 170. (d) Assumption 1 can be made from the sentence,
external and internal accountabilities is mandatory for ‘Teacher accountability systems...
effective management.
Assumption 2 follows from ‘universal access to
156. (d) The passage does not provide relevant information on
either of the provided options.
education’ in the opening line.
157. (d) Civil services, being an extremely reputed taskforce, Assumption 3 follows from, ‘we must address
should implement reforms to create benchmark in service the entire gamut ...,’ which implies that India’s
and improve accountability of the office bearers. ‘demographic dividend’ or ‘huge young
158. (d) Seeking accountability through increased participation population across diversities’ could be tapped.
by individuals in the decision making process would
not help increase internal accountability. 171. (d) It is clearly mentioned in the first half of the
159. (a) Religious traditions, regardless of felicitating the second paragraph of the passage.
Almighty or the Supernatural, ushers belief, ethics and
a specific code of conduct in the society. Man being a 172. (c) The entire passage emphasises on giving quality
social animal derives the value set from the society, education to all children. Options (a), (b) and
important for maintaining relationships and goodwill. (d) are true, but none conveys the essential
Thus, human relationships can be assumed to have message of the passage.
been derived from the religious traditions.
160. (c) Fundamental rights, combined with duties, are 173. (b) It is clearly mentioned in the passage in the line
mandatory for dispensing justice in a society. “This theory is false, for the art of leadership
161. (d) As per the passage, all the three options pose can be acquired and can indeed be taught”.
challenges to the utilization of biomass as fuel. Issues 174. (a) Refer to last sentence of the passage. The two little
related to climatic variations, aggravation of carbon boys feared the barwing dogs.
emissions due to liquid bio-fuels and competition 175. (c) The stray dogs looked at the procession curiously
between biomass and food production, have adverse because everytime the old man threw seeds form the
effects. berries, the birds would fly to peck them.
176. (c) Cynthia was a shy girl and she believed that she was
162. (b) It is stated that unscrupulous cultivation of energy crops
plain and untalented.
will lead to an unhealthy competition with food crops, 177. (b) It is because Cynthia's classmates were not interested
thereby contributing to inflation and price hike for food in the previous performances.
crops. 178. (a) As Cynthia did not have confidence in herself and was
163. (b) Technological intervention, with proper monitoring, frightened to perform in front of her class mates. She
could assist in using biomass for power generation and was trembling. So, her knees were quaking.
achieving negative emissions. 179. (d) Cynthia's remarkable performance attracted
everybody's attention and everyone was stunned. After
164. (a) As per the information provided in the 2nd paragraph,
her performance ended, all the people in the room stood
some energy models (unproven technology) might up and applauded for Cynthia.
negate carbon emissions, thereby mitigating 180. (d) (a) is wrong as the passage is not at all related to
environmental pollution due to the greenhouse effect. Male chauvinism and infidelity
165. (a) The passage pertains to duty to vote. (b) is wrong as Love and betrayal is not the issue the
166. (d) The passage denotes that despair sometimes leads to passage is trying to sentitise about. The passage is
dictatorship in a country. more about a mothers' influence on her child. How she
governed his life?
167. (b) The assumption that one of the aims of sustainable
(c) is wrong as the passage is not at all related to
agriculture is to ensure minimal ecological imbalance Lack of legal safeguards for women. Guleri didn't
can be made from the passage. resorted for any such protection.
168. (c) Biopesticides are not hazardous to human health and (d) is correct as discussed in (b). The real reason
they are essential to maintain the biodiversity of any behind this situation was Manak's mother. He could
ecosystem. not realise her wife's love as he was under the influence
of her mother's mindset.
169. (c) The opening line of the passage says that ‘universal 181. (b) The key point of the passage is that there should be
access to education’ is no longer enough. So, 1 is a balance between work and leisure.
not of paramount importance. The second sentence 182. (b) The challenge the author throws to the public is to
achieve equal privileges, fulfilment of destiny and
says that ‘facilities’ or infrastructure is a prerequisite political tolerance.
but ‘insufficient’ to fulfil the purpose of education 183. (c) The nature of General Will, in the passage, is best
So, 2 is not of paramount importance. described as the collective good as distinct from
Private Wills of the individuals.
3 is mentioned in the second paragraph as being of
paramount importance.
EBD_7711
96
184. (d) It can be inferred from the first sentence of the 3rd Þ Monday + 7
passage especially from the part ‘ India should rid 10th Þ Monday + 7
herself ..........development of the body and the spirit’. 17th Þ Monday + 7
185. (b) Medieval Period Poets = A, B, C, D 24th Þ Monday
Modern Period Poets = E, F, G, H Fifth day from 21st is 25th day.
So, on 25th day is Tuesday.
Option (a) is eliminated as it is a modern poet. Sol. (Qs. 194-196)
Question on modern poet will come in next year. + Male
Option (c) is eliminated as the paper-setter does
– Female
not like to ask about F.
Option (b) is appropriate as the paper-setter likes
F and those who like F like (E) also. A (Engineer) D (Stenographer)
(+) (–)
186. (d)

Profession
B (Judge) C (Lawyer)
Women Dancers Musicians Actress Violinists
(+) (–)
Girija Ö
Vanaja Ö Ö
E (Doctor) F (Draughtman)
Jalaja Ö × (+)
(?)
Shailaja Ö Ö × 194. (a) Judge
Tanuja Ö Ö Ö 195. (c) Couples are AD & BC
196. (b) Stenographer
Pooja Ö
197. (d) English Hindi
187. (d) 6 4 9
L M N O P Q R S T 11 10 29
Difference between P & T is 5 i.e., T = 5 + 4 = 9
Difference between N & T is 3 i.e., N = 9 – 3 = 6 Students who can speak Hindi = 29 + 10 = 39
So, integer assigned to N = 6 Students who can speak only Hindi = 50 – 21 = 29
188. (d) Students who can speak only English = 50 – 29 – 10 = 11
189. (d) 198. (b) Guitar Violin
190. (b)
191. (a) a + b + c + d = 100
Also, a + b = c + d = 50 a b f
d d g
c= , \ + d = 50 Þ d = 33.3 c e
2 2
\ d>c d
Also, a > b & between a and d, a = d + 5
\ a>d
\ Alok (a) has the maximum money. Flute
192. (c) a + b + c + d + e + f + g = 120
A B 5 ´ 120
Ist g= =6
11th 10th 100
arrangement
g=6 ...(1)
10 students 9 students c + b + e = 30 ...(2)
A a = 40 ...(3)
B
So, a + b + c + e + d + f + g = 120 ...(4)
18th
From (1), (2), (3) & (4)
17 students
40 + 30 + d + f + 6 = 120
No. of students between A & B in the 1st arrangement d + f = 120 – 76
= 17 – 11 = 6 d + f = 44
\ Total no. of students except A & B is,
= 10 + 6 + 9 = 25
193. (b) Tuesday
97
199. (a) Flip
3 480 540
60 - +
11 11
1 1 1 1 1 1
Flip 660 + 540 - 480 720
= = = 65.4
1 2 2 2 2 1 1st tries 11 11
Flip
2nd tries 205. (c)
1 2 1 1 1 2
2 2 2 2 2 2 3rd tries
200. (c)
Top C
A

G
206. (b) 1st Column 3rd Column
Middle B

E
2nd arrow
F
Þ bend towards
Þ
rights
Bottom D
Þ 2nd arrow
D is at the bottom step of the ladder.
Þ
bend down
201. (c) Sequence according to height.
2nd Column
E >D >C >A>B

Mid Position
Therefore, ‘C’ occupy the mid position. Þ Second arrow bends
towards left
202. (b) 1 km
A C
Þ So second arrow
move up
1 km
207. (d) Sum of numbers in 1st Row, 29 + 13 + 18 Þ 60
1 km
F D Sum of numbers in 3rd Row, 30 + 27 + 3 Þ 60
So, 2nd Row, 33 + X + 19 = 60
1 km X + 52 = 60
1 km X = 60 – 52
E B X=8
Hence, A, D, E in a line. 208. (a) Remove one circle, a triangle is remove,
So,
203. (b) C B A D
Hence, A occupy seats adjacent to D.
204. (c) Two hands of clock are on one above between 8
to 9.
12 to 8 Þ 40 min after removing circle, triangle is also remove.
60 480 So,
40 min Þ ´ 40 = min past 8.
55 11
two hands of clock are on one above between 9 to 10.
12 to 9 Þ 45 min
60 540
45 min Þ ´ 45 = min past 9.
55 11
A circle and a triangle is reducing by one.
So two hands be lying one above the other =
209. (d) 18
EBD_7711
98
Starting point
1
Left S R
2 Right
T, 3 routes 1 km 1 km
Form S U V 3

1
V 2
T, 3 routes Destination
S U 3 point
Hence, (b) is correct.
217. (a)
1
2 T, 3 routes
S U 62
3 24 52
English
Mathematics

1
2
S T, 3 routes Total students = 130
U V 3 Total students failed = (62 + 52) – 24
= 114 – 24
= 90
Number of students passed finally = 130 – 90
1
U 2 = 40
S T, 3 routes 218. (c) Let a = Speaks one language
V 3 Let b = Speaks two language
Let c = Speaks three language
Total routes = 3 + 3 + 3 + 3 + 3 + 3 = 18 then
210. (b) From 1 to 2 - vertical interchange of half shaded a + 2b + 3c = 6 +15 +6
circle. a + 4 + 3 = 27
211. (d) Due to inflation the value of money only would a = 20
be halved \ Total number of persons in the group
212. (a) = 20 + 2 + 1 = 23.
213. (b) 219. (c) Let number of cars be x
Let number of scooters / motor bikes = y
Books Covers Volume Law/Medical \ Total number of parked vehicles = x + y
According to question
A Yellow New Law
4x + 2y = 2 (x + y) + 100
B Green New Law 4x + 2y = 2 x + 2y + 100
2x = 100
C Green Old Law
x = 50
D Yellow New Medical Hence, Number of cars parked is 50.
220. (c) The passage c learly talks about the role
E Green Old Medical
Mangroves play in some of the coastal food chains.
F Green Old Medical Other food chains - with or without Mangroves - might
Therefore, E and F are old medical extracts and have exist but they are not mentioned in the passage. So (c)
green covers. is correct. The passage does not say that no food chain
is possible without Mangroves so (a) is wrong. (b) is
214. (b) A < B < C
also wrong as the passage does not talk about
Now, compared to A, C runs faster than A.
Mangroves being an essential component of all marine
215. (c) Cauliflower > Lady finger > Cabbage
ecosystems. Further (d) is also wrong for the same
But Peas > Cabbage
reasoning as the passage does not talk about the
The conclusion that can be drawn from these composition of marine flora and fauna.
statements is that cabbage is the least tasty among
221. (d) (a) is wrong because the sentence says that
the four vegetables. liberty means the eager maintenance of that
216. (b) After both have travelled 3 km, their positions atmosphere which might require restrictions
would be on follows. as well.
99
(b) is wrong because if the people work only does not mean that no one wants to win. Hence, (b) is
according to the law they might become true. There can be people other than good athletes,
good citizens but might not work to their who eat well balanced diet.
best selves. 230. (c) Between 6 PM and 7 PM, the hour hand and the
(c) is wrong because if people do what they minute hand coincide at 6 hr ‘m’ minutes.
desire it might lead to materialist thinking
which is not the best self. 30h ´ 2 30 ´ 6 ´ 2 8
i.e. m = = = 32 mins.
(d) is correct because with the growth of human 11 11 11
personalitythe people would be their best Hence, the hour hand and minute hand coincide
selves.
8
222. (a) The ‘D’ is shifting one corner to the next in the at 6 hr 32 mins.
clockwise direction and each time it is getting 11
inverted. Thus, the D should appear at the top left After 3 minutes, i.e. at 6 : 36 PM the minute hand
corner pointing upward. Rest of the elements are of a clock will be ahead of the hourhand by 3 minutes.
shifting positions in the counter clockwise direction.
223. (a) In each column, let the number at the top be ‘x’
and the one at the bottom is ‘(x – 1)’. The relation
between columns are given by 231. (d)
(x) (x – 1) = 3(3 – 1) = 6 Shabnam Marathi No Driving
Therefore, 5(5 – 1) = 20
Anil Tamil No Driving
224. (c) 13
Tamil/
7 X 21 31 43 Rekha Driving
Marathi
+6 +8 +10 +12 Marathi/
David Driving
225. (d) By combining the given fragment, we get figure (d). Tamil
226. (c) Given, A is tallest among the four. Hence, A is male, From above table, it can be concluded that only
D is also male, as D is the brother of B. Then, B and C of those who can drive car can speaks Tamil.
are females. D is the husband of C. D is the shorter 232. (b)
than A and is taller than B. These, B is the shorter A Fighters
from above only (c) is incorrect. Enemy
227. (b) B Cowards
Man Wife
C Cowards
Friends
Enemy Cowards
D
Son Son Wife
E Fighters
All the male member of the family went out to
take part in a picnic. It is given that daughters were F Fighters
invited to a feast. From above table, it is clear that A, E and F are
Nothing is specified about men’s wife. Hence, the fighters.
statement that the man’s wife is likely to be left at 233. (c) From 2 and 3, I and B are on opposite faces. from
home is true. 1 and 4, Y and V are adjacent to O.
228. (a) Radha did not accept the argument that a person
becomes a better boxer after meditation. The reason Y
for this is that a boxer, according to her, should be I O B
aggressive. This indicates that Radha believes that
meditation makes a person less aggressive. V
229. (b) The given statements can be represented as– G
Hence, G is the colour of the face opposite to the
Eat well
Want to face coloured with O.
balanced
win 234. (d)
diet Wise Men Great

Good Athletes
It is stated that all good athletes want to win. It or
EBD_7711
100
239. (a) C’s surname is Riberio.
240. (b) The colour of the tie is Blue.
Wise Men 241. (a) A wore the sweater.
242. (a) The direction diagram of a person is as follows:

Great 19 km

B
D C
Conclusion I : False 4 km 15 km
Conclusion II : False 12 km
235. (d) O
15 km A
236. (c) ABC × DEED = ABCABC
We know that, when we write any Number in
form of ABCABC then that number must be divisible
by 13 , 11 and 7 or ABCABC is multiple of 1001 E
X
So, 1001 = DEED
1001 = 13 × 11 × 7
ABC × DEED = ABC ABC N
Þ ABC × 1001 = ABC ABC NW NE
On comparing
D = 1, E = 0
237. (b) According to question, W E
Priti < Swati < Kamala < Usha
From the above, it can concluded that priti is the
slowest runner. SE
SW
238. (b) According to the given information, the direction S
diagram of a person is as shown
below: The required distance is,
In DOAE
N
B OA = DB = 4 km
NW NE
12 km OE = AX = 3 km
AE2 = OA2 + OE2
A AE2 = 42 + 32
W E
AE2 = 25
AE = 5 km
5 km
243. (d) The number of small cubes with only one of the
SE
O
SW sides painted is 8.
S
244. (d) As S Y S T E M and N E A R E R
\ Required distance,
BO2 = BA2 + OA2
SYSMET AENRER
BO2 = 122 + 52
Similarly,
BO2 = 144 + 25
BO2 = 169 FRAC TION
BO = 13 km
So, he is 13 km far away from the starting CARFNOIT
point O. 245. (b) Let the number of boys be x, then numbers of girls
Sol. (239-240) : = 2x
On the basis of given information, the arrangement is as \ x + 2x = 60; or 3x = 60; or x = 20
following: So numbers of boys = 20 and numbers of girls = 40
Numbers of student behind Kamal in rank
Riberio Kumar Singh Jacket Sweater Tie Blue White Black = (60 – 17) = 43
A ´ ü ´ ´ ü ´ ´ ü ´ Numbers of girls ahead of Kamal in rank = 9
B ´ ´ ü ü ´ ´ ´ ´ ü \ Numbers of boys behind Kamal in rank
C ü ´ ´ ´ ´ ü ü ´ ´ = (43 – 31) = 12
101
246. (b) North day on every 60th
day.
C
\ Number of times all the 5 groups meet on the
3km
B
180
X’s same day within 180 days = =3
4km 3km 60
starting
point
1km
251. (c) No. of bee > Flower. So, checking by option there
A
West East are 3 and 4.
1km 1km Sol. (252-254) :
According to the given information, the
D 1km E
arrangement of five persons in a group is as
following :
X’s returning point
South
· X starts from point A, passes through point B Doctor Lawyer Artist Gender
and C and reaches point D. P
· X returns from point D, passes through point Q ü Male
E and reaches point A back. R ü Female
247. (d) Option (d) is the just contradiction of given
S
statement, if it is true, it would falsify the given
statement. T ü Male
248. (d) Question does not say anything about the gender
composition of the group invited for party. couple
T ¬¾¾¾ ® R , Q ¬¾¾¾
®
Brother
P
Further, we donot know which specific female (+) (- ) (+) (unmarried)
faculty know music. So either all in party were S is also unmarried.
males, or some males and some females who did 252. (a) T is the Doctor.
not know music were present in the party. So (d) 253. (c) R is the Artist.
is definitely correct.
254. (b) T is the spouse of R.
249. (b) The arrangement is in circular table 255. (b) Q is the Lawyer
· C is seated next to A 256. (c) Q is definitely a man.
Conclusion : AC – or CA 257. (c) Quantity of product to be completed = 19000
Firm production per day = 1000 – 5% of 1000
A C = 950
Firm production in 19 days = 19000 – 5% of 1900
C or A
5
= 19000 – ´ 19000
100
· A is seated two seats from D = 19000 – 950 = 18050
Conclusion : A – D or D – A So, firm needs one extra day other than 19 days
· B is not seated next to A to complete the order.
Conclusion : E must be next to A \ Firm will complete the order in 20 days.
Combining, A C D B E 258. (d) From statement 1,
E E A³B
From statement 2,
A A C£D
B D and from statement 3
OR
B>C
By combining the above three statements, we get
C C
D B A ³B>C£D
(There can be many arrangements) Here, A > B > C < D
So, only statement 1 and 2 are correct. or A = B > C = D
250. (d) Here, we need to take the L.C.M. of 2, 3, 4, 5 and 6 A>C
to find out the number of days after which all So, A is older than C.
these 5 groups meet on the same day. Sol. (259-260) :
So, L.C.M. (2, 3, 4, 5, 6) = 60 According to the given information, the arrangement
We can say that the 5 groups meet on the same of six boxes is as following:
EBD_7711
102
Hence the best answer is option (b).
Boxes G am es Colours 267. (b) Since, the month begins on sunday and if it has
29 days then
A Ten nis Oran ge
1st, 8th, 15th, 22nd and 29th are Sundays. While
B Volleyball Yellow/Blue 14th and 21st days are 2nd and Fourth Saturday.
C Cricket Green Hence,
D Football Yellow/Blue 29 – (2 + 5) = 22 working days.
E Golfball Violet 268. (c) Given information can be represented as:-
F Hockey Indigo Subject Biology Physics Chemistry Mathematics
DAY I II III IV
Cricket Tennis 269. (a) Let us apply basic logic. Conclusion I definitely.
Follows as that is the main assertion given.
B C B A (Orange) Conclusion II is incorrect, as it says "Even though
(Green) one's voice is not good, one can keep practicing".
259. (b) E boxes contains the golfball. It is an assumption. Hence best answer is (a).
270. (c) Except (c) other options are absurd.
260. (b) F is painted indigo is the correct statement.
271. (c) P + R – Q
261. (c) We cannot determined that the football is in the
box of which colour. Mother Father
R Q
262. (d) Since, there are 5 questions to be attempted and
each questions has two choices– True or False.
Therefore, the maximum number of candidates
so that none of them gives the answers to the five Son
questions in an identic al sequenc e is an P
arrangement of 2 different options at 5 places.
Hence, Q is the Father of P.
2 2 2 2 2 272. (c) The given information can be represented as :-
Above figure shows that each question can be
filled with 2 choices. Name Bank Tennis P G in PG in In
Employee Player Commerce Sociology Unemployed
So, required numbered of candidates = 2 × 2 × 2 × 2
× 2 = 32 Vimla ü
263. (d) According to given information: Kamla ü ü
Amala ü ü
Fruits Eaten Komala ü ü ü
Persons Grapes Pineapple Oranges Apple Nirmala ü ü ü
A ü ü Shymala ü
B ü ü
Hence, Nirmala is both a tennis player and a bank
C ü ü ü
employee.
D ü ü ü 273. (d) None of the above can be concluded with
certainty.
B and C both ate Oranges, So, it can be said that 274. (d)
the cause of sickness was eating of oranges. 275. (b) A B C D E F
264. (d) The death rate is declining more faster than in
birth rate. This is the prominent reason for 8 7 12 15 11 7
increament in the rate of population growth. Hence, D + E = B + C + F .
265. (b) The line “ but some of them had Friends in 276. (c) Biopesticides are not hazardous to human health
party Y” clearly indicates statement (b) is and they are essential to maintain the
correct. Best answer is (b) biodiversity of any ecosystem.
266. (b) Statement 1 is correct. Statement 2 is incorrect
because in the passage there is no mention of
people who opposed Z’s campaign strategy.
103
285.
277. (d)
Subject
Colours Pleasant Or Colours Hindi English Math History French Geography
Pleasant
Teachers
A P P P
1. False
2. Can’t say B P P P P P
3. Can’t say C P P
Hence (i) is false. D P P
278. (b) A is the husband of D. E P P
F–
Sol. (286 -287)
‘ « ’ Denotes one way, ‘ Û ’ Denotes two way.
n C « A, E « C, E « G, H « G
So

Couple B « F, A Û E, D « H, G Û B
+
Brothers +
D – A E G « C, F Û D, E Û D
286. (b)
r
hte

287. (d) F — D — E — C — A or F — D — H — G —
n
So
ug

Son C – A,
Da

or
F — D — E — A or F — D — E — G — C – A.
C– B+
279. (c) The given information can be represented as – So, 4 ways.
A C B D G F E H 288. (c) H — G — E — C and H — G — B — F — D —
Hence, G is the immediate right of D. E – C.
So, nowhere we have to travel through A.
2 5 6 Red Colour Chalk 289. (d) Lakshmi herself is the best player.
Best Worst
280. (b) 2 5 4 White Colour Chalk
L LB
5 8 9 Green Colour Flower

Red 6 LS LD
White 4 290. (b) According to question.
Pen < Pencil < Book > Cap
281. (c) The sum of rainfall for the first four days = 4 × So, Pen < Book is always true.
0.40 = 1.60 inch 291. (b) For easily count the number of triangles in the given
The Sum of rainfall for six days = 0.50 × 6 = 3.00 three-dimensional figure, devide the given figure into
inch three parts-top, bottom and middle.
Þ Sum of rainfall for last two days = 3.00 – 1.60 In each of the top and bottom part, there are 5 triangles
= 1.40 inch which are based on a pentagon. In middle part, there
are 5 quadrilaterals inclosed between two pentagons.
4 Further each quadrilateral is divided into two triangles.
Rainfall on 5th day = ´ 1.40 inch = 0.80 inch.
7 Hence in the middle part, there are 10 triangles.
Sol. (282-284): \ Total number of triangles in the given 3D-figure.
= (No. of triangles in top part) + (No. of triangle in
City Subject Lecturers bottom part) + (No. of triangle in middle part)
Kanpur Geography B = 5+ 5 + 10 = 20
292. (d) According to pattern,
Shillong History / Mathematics D
when * = 8 then,
Delhi Sociology C 8 + 18 + 28 + 83 + 81 = 218
Mumbai Economics A So, option (d) is correct answer.
293. (a) Each small cube lying at the middle of each edge of
Srinagar Commerce F the large cube has exactly two painted faces.
Chennai Statistics G Since there are 12 faces in the given large cube
Hyderabad History / Mathematics E therefore there are exactly 12 small cubes which have
exactly two painted faces.
282. (a) 283. (b) 284. (b) 294. (a) According to question
As, (8 + 4) – 6 = 6
EBD_7711
104
(10 + 6) – 5 = 11 309. (b) It is clearly mentioned in the last line of the passage.
(15 + 8) – 7 = 16 310. (a) It is mentioned in the passage how a male tiger that
So, (13 + 8) – 4 = 17 was relocated in Panna National Park trekked toward
its home 250 miles away; and that nearly a third of
India’s tigers live outside tiger reserves. These points
indicate that though initiatives have been taken for the
conservation of wildlife by relocating them, they have
not been often successful. The passage does not say
anything about any legistation in this regard.
311. (c) The passage clearly mentions that ‘most countries’ are
‘striving’ to address climate change threat. So, it is a
complex policy issue and development issue for all.
Sol. (295-297) So, 1 is not valid but 2 is.
295. (b) We can see line, two dots toward one end of the line Assumption 3 can be made from, ‘the problem is more
and three dots toward other end of the line. severe ........... finance development. So, the right option
296. (c) We can see, two dots, and four dots are on opposite faces.
297. (c) Line and cross are on opposite faces is (c).
298. (a) We can consider each picture formed with an 312. (d) The passage states the fact that household air
unique body (B) and an unique set of legs (L). pollution is hazardous to health of the people living
Then in poor areas to such an extent that more than 10 lakh
premature deaths occur every year due to it So, it
B1 B2 B3 would be appropriate to say that the access to
First Row : L1 L2 L3 cooking gas can reduce premature deaths in poor
households.
B2 B3 B1 313. (c) It is clearly mentioned in the second sentence of the
Second Row : L passage.
3 L1 L2
314. (b) It is mentioned in the last sentence of the passage that
the distinct possibility of ice-free summer has prompted
B2 B1 æ B2 ö countries with Arctic coastline to rush to get larger
Third Row : ç = ÷
L3 L 3 è L1 ø part of the Arctic which thereby leads to changes in
the geopolitics.
299. (b) According to figure. 315. (a) The passage conveys the facts that India provides
Square element moves clockwise direction in each food security to its poor which collides with its trade.
step and triangle element moves anticlockwise But being a signatory of WTO, it has to revise its PDS,
direction in each step. Option (b) is not conveyed.
So, option (b) is correct answer. 316. (b) As per the last sentence of the passage, today’s
Sol. (300– 305): educational system is very different i.e. it is a complex,
Students Cities Colleges States volatile and globally interconnected world; thus, in
A P/R Commerce Gujarat order to meet these, today’s learners need to acquire
B Q Sciences Kerala new-age skill-sets. Assumptions 1 and 3 cannot be made
from the passage.
C S Arts Rajasthan
317. (d) The passage conveys that individuals differ in many
D P/R Engineering Assam
respects and ‘we cannot reduce so many complexities’
Sol. (306-307): into one copybook diet plan. So, there is no perfect
D A F solution. Options (a), (b) and (c) mention only one
Corridor aspect each of the discussion and none qualifies as a
Entrance rational inference.
C E B
318. (a) Crop genetic diversification consists in raising a variety
306. (c)
307. (a) of crops depending on suitability to seasonal variations
308. (d) Last few lines of the passage state that meat production of rainfall and temperatures; and reduces susceptibility
through industrial farming is water - intensive; it requires to natural disasters like flooding and windstorm, etc.,
15000 litres of water for every kilogram of meat. We all in case of monoculture. Therefore, preserving crop
are aware of the fact that many people in the world strive diversity is an insurance against the effects of climate
to get water to drink and for their household works and change. Sentence one declares that monoculture carries
so, we can’t afford to lose such huge amount of water risk; so, (a) follows.
on producing meat in farms thus, it can be said that 319. (b) It is stated in the passage that presently, around 47
environmental cost of meat production is unsustainable percent of India’s population is dependent on
when it is produced through industrial farming. agriculture and it is also expected that around 2030,
105
nearly 42 percent of India’s population will still be better path to answer. Thus, it can be inferred that
predominantly dependent on agriculture. Therefore, science and faith can be mutually complementary if
considering these large percentages, it can be inferred their proper domains are understood. Moreover, rest
that Indian economy greatly depends on its agriculture. of the options apart from option (b) are irrelevant in the
320. (a) It can be inferred from the line ... rapid globalisation of context of the passage.
food... and trace of the passage. 322. (c) It can be inferred from the line “If the aspirations are
321. (b) The author states that there are some questions that more like the frsugal ones we had after the Second
science cannot really answer while faith provides a World War, a lot more is possible than if we view the
planet as a giant shopping mall”.
EBD_7711
106

Test–1 General Aptitude (GS Paper II)


1. Five litres of water is added to a certain quantity of pure milk The areas of two rectangles are indicated in the figure. What
costing ` 3 per litre. If by selling the mixture at the same price is the length of each side of the square?
as before, a profit of 20% is made, what is the amount of pure (a) 10
milk in the mixture ? (b) 11
(a) 22 litres (b) 25 litres (c) 15
(c) 27 litres (d) None of these (d) Cannot be determined as the given data are insufficient
1 4.
2. 12 men and 18 boys, working 7 hours a day, can do a
2
piece of work in 60 days. If a man works equals to 2 boys,
then how many boys will be required to help 21 men to do
twice the work in 50 days, working 9 hours a day ? B
in

Velocity
(a) 30 (b) 42 Tra
(c) 48 (d) 90 A
in
3. Consider the following figure and answer the item that Tra
follows:
0 Time t0
15
With reference to the above graph, which one of the following
statements is not correct?
48
(a) Train B has an initial acceleration greater than that of
Train A.
(b) Train B is faster than Train A at all times.
A square is divided into four rectangles as shown above. (c) Both trains have the same velocity at time to
The lengths of the sides of rectangles are natural numbers.
(d) Both trains travel the same distance in time to units.

Directions (Qs. 5–6): Study the following data carefully to answer the questions that follow.

Details of Statewise distribution of candidates appeared in an Examination

Statewise distribution of candidates appeared


Ratio of male and female candidates appeared
Total candidates appeared = 1,20,000

State Ratio
G A
A 5:7
9% 12%
F B 11 : 8
15% B
C 3:4
19%
D 9:7
E E 5:6
C
22% D 7% F 8:7
16% G 4:5
107
5. The total number of female candidates appeared from states 9. A train X departs from station A at 11.00 am for station B,
E & G together is what per cent of the total number of which is 180 km away. Another train Y departs from station B
candidates appeared in the examination from all the states? at 11.00 am for station A. Train X travels at an average speed
of 70 kms/hr and does not stop any where until it arrives at
(a) 13 (b) 11
station B. Train Y travels at an average speed of 50 kms/hr,
(c) 17 (d) 15 but has to stop for 15 minutes at station C, which is 60 kms
away from station B enroute to station A. Ignoring the lengths
6. What is the total number of male candidates appeared in
the train , what is the distance , to the nearest km, from
the examination from states B and F together?
station A to the point where the trains cross each other?
(a) 16,800 (b) 18,4000 (a) 112 (b) 118
(c) 13,200 (d) 22,800 (c) 120 (d) None of these
10. In a cricket match, Team A scored 232 runs without losing a
Directions (Qs. 7 – 8): Study the two figures given below and wicket. The score consisted to byes, wides and runs scored
answer the five items that follow. by two opening batsmen : Ram and Shyam. The runs scored
by the two batsman are 26 times wides. There are 8 more
36
byes than wides. If the ratio of the runs scored by Ram and
32 Shyam is 6 : 7, then find the runs scored by Ram.
28
(a) 88 (b) 96
24
20 Male (c) 102 (d) 112
16 Female
12
Directions (Qs. 11 – 20): Read the passages and answer the
8 question based on them.
4
0
PASSAGE - 1
CS

S
CS

Y
Y

NY

IC
G
TR
TI
SI

LO
TA

OM
IS

In national no less than in individual life there are no watertight


A
Y

O
EM

BO
PH

EM

N
CH

O
TH

CH

compartments. No sharp lines can be drawn to mark off" the


EC
Y
A

PS
M

political from the moral, the social from the economic regions of
life. Politicians often talk as though one has only to introduce
Figure 1: Number of Professors in selected disciplines in a
certain political and economic changes for paradise to descend on
University by sex
earth, forgetful of the fact that the efficiency of an institution
depends on the way it is worked, which itself is determined by the
20% character and wisdom of the men who work it.
11. Which one of the following statements most clearly suggests
40% 35 - 44 the central theme of the passage?
(a) Political and economic changes can solve all the
10% 25 - 34
problems facing the nation.
60 - 65 (b) There is no difference between the political, moral,
45 - 59 social and economic regions of life.
(c) It is not the institutions that are important but the
character and wisdom of the people who manage them.
30%
(d) National progress depends solely on the efficient
running of our institutions.
Figure 2: Age of Physics Professors 12. Which one of the following phrases best helps to bring out
the precise contextual meaning of 'watertight compartments"?
7. If the number of female Physics professors in the age group (a) Activities of life unaffected by public opinion.
25 – 34 equals 25% of all the Physics professors in that age (b) Spheres of life where no liberty of opinion is tolerated
group, then what is the number of male Physics professors in (c) Ways of life peculiar to each nation and each section
the age group 25 – 34? of society.
(a) 9 (b) 6 (d) Spheres of life which are independent and unconnected
with one another.
(c) 3 (d) 2
13. Which one of the following statements most correctly reflects
8. If the Psychology professors in the University constitute the attitude of the author towards politicians' opinions?
2% of all the professors in the University, then what is the (a) The author totally disbelieves what the politicians say.
number of professors in the University? (b) The author believes what the politicians say.
(a) 400 (b) 500 (c) The author is sceptical about the claims of the
(c) 600 (d) 700 politicians.
EBD_7711
108
(d) The author thinks that the opinions of the politicians 18. The passage emphasizes that modern technology
are contradictory. (a) is totally avoidable.
14. Which one of the following statements most correctly indicates (b) has caused serious hazards to life.
the implication of the phrase 'paradise to descend on earth'? (c) has greater effect on developed countries.
(a) A world of perfect economic, political and social well- (d) is the source of the miseries of mankind.
being. 19. The harmful effects of modern technology are
(b) A world ruled by religious persons. (a) widespread but short lived.
(c) A world of total liberty and equality. (b) widespread and long lasting.
(d) A world in which nobody needs to labour. (c) local and long lasting.
(d) severe but short lived.
PASSAGE – 2 20. With reference to the passage, the following assumptions
have been made :
I came home from one vacation to find that my brother Ron had
1. The widespread use of insecticides has caused
brought a dog while I was away. A big burly, choleric dog, he
ecological imbalance.
always acted as if he thought I wasn’t one of the family. There
2. Conservation of natural flora and fauna is impossible
was a slight advantage in being one of the family. For he didn’t
in this age of modern technology.
bite the family as often as he bit strangers. Mother used to send
Which of the assumptions is/are valid ?
a box of candy every Christmas to the people he bit. The list
(a) 1 only (b) 2 only
finally contained forty or more names. Nobody could understand
(c) Both 1 and 2 (d) Neither 1 nor 2
why we didn’t get rid of the dog!
21. Priya is taller than Tiya and shorter than Siya. Riya is shorter than
15. Which of the following descriptions fits the dog?
Siya and taller than Priya. Riya is taller than Diya, who is shorter
(a) The dog was tiny and delicate
than Tiya. Arrange them in order of their ascending heights.
(b) The dog was sturdy and short-tempered
(a) Priya – Siya – Riya – Tiya – Diya
(c) The dog was huge and cool
(b) Riya – Siya – Priya – Diya – Tiya
(d) The dog was small and sweet-tempered
(c) Diya – Tiya – Priya – Riya – Siya
16. The dog did not consider the writer as one of the family.
(d) Siya – Priya – Riya – Diya – Tiya
What do you think was the consequence of this?
22. Four children A, B, C and D are having some chocolates each.
(a) The dog barked at him all the time
A gives B as many as he already has, he gives C twice of what
(b) The dog drove him out of his own house
C already has and he gives D thrice of what D already has.
(c) The dog behaved with him in an unfriendly way
Now, D gives 1/8th of his own chocolates to B.
(d) The dog bit him more than he bit others in the family
Then A gives 10% chocolates he now owns to C and 20% to B.
17. The Christmas list contained more than forty names. What
Finally, all of them have 35 chocolates each.
does this suggest?
What is the original number of chocolates each had in the
(a) The writer’s mother had a lot of friends
beginning?
(b) The writer’s family celebrated Christmas well
(a) A -110, B-10, C-10, D-10 (b) A-90, B-20, C-20, D-10
(c) The writer’s dog had bitten at least forty people
(c) A-70, B-25, C-25, D-20 (d) A-125, B-5, C-5, D-5
(d) The writer’s mother sent them candy boxes
23. There are some balls of red, green and yellow colour lying on
a table. There are as many red balls as there are yellow balls.
PASSAGE – 3
There are twice as many yellow balls as there are green ones.
The assault on the purity of the environment is the price that we The number of red balls.
pay for many of the benefits of modern technology. For the (a) is equal to the sum of yellow and green balls
advantages of automotive transportation we pay a price in smog- (b) is double the number of green balls.
induced diseases; for the powerful effects of new insecticides, we (c) is equal to yellow balls minus green balls.
pay a price in dwindling wildlife and disturbances in the relation of (d) cannot be ascertained.
living things and their surroundings; for nuclear power, we risk
the biological hazards of radiation. By increasing agricultural Directions (Qs. 24-25) : Study the following information to answer
production with fertilizers, we increase water pollution. the given questions :
The highly developed nations of the world are not only the immediate Eight people are sitting in two parallel rows containing four people
beneficiaries of the good that technology can do, they are also the each, in such a way that there is an equal distance between adjacent
first victims of the environmental diseases that technology breeds. In persons. In row-l A, B, C and D are seated (but not necessarily in the
the past, the environmental effects which accompanied technological same order) and all of them are facing North. In row-2 P, Q, R and S
progress were restricted to a small place and relatively a short time. are seated (but not necessarily in the same order) and all of them are
The new hazards are neither local nor brief. Modern air pollution facing South. Therefore, in the given seating arrangement each
covers vast areas of continents. Radioactive fallout from nuclear member seated in a row faces another member of the other row.
explosions is worldwide. Radioactive pollutants now on the Earth's S sits second to left of Q. A faces the immediate neighbour
surface will be found there for generations, and in the case of Carbon- of S. Only one person sits between A and C. P does not face A. B
14, for thousands of years. is not an immediate neighbour of A.
109
24. Which of the following is true regarding D ? (A) at (ii) above, but has secured at least 55 percent marks in
(a) D sits at one of the extrem ends of the line Graduation in any discipline and at least 70 percent marks in
(b) A sits to immeditate left of D post Graduate Degree/Diploma in personnel Management/
(c) Q faces D HR, the case is to be referred to GM - HR.
(d) C is an immediate neighbour of D (B) at (iv) above, but has post qualification work experience of
25. Who amongst the following sits to the immediate right of at least two years as Deputy Manager- HR. the case is to be
the person who faces C? referred to President - HR.
(a) P (b) Q In each question below are given details of one candidate. You
(c) R (d) S have to take one of the following courses of actions based on
Directions (Qs. 26-27): Study the paragraph and answer the the information provided and the conditions and sub-conditions
questions that follow: given above and mark the number of that course of action as
your answer. You are not to assume anything other than the
A team of experts for conducting interviews consists of seven information provided in each question. All these cases are given
experts – Bhushan, Cyriac, Pramila, Ram, Suresh, Shekhar and Unni. to you as on 01.03.2012.
Of these Bhushan, Cyriac and Pramila are experts in social sciences Mark answer (a) if the candidate is not to be selected
while Suresh and Unni are experts in basic sciences. Ram and Mark answer (b) if the candidate is to be selected
Shekhar have exposure in both basic sciences and social sciences. Mark answer (c) if the case is to be referred to president-HR
Three panels have to be formed for the interview with a restriction Mark answer (d) if the case is to be referred to GM-HR
that a panel should have representation from experts with social
sciences and basic sciences background. Moreover, at least one DIRECTIONS (Qs. 29-30): Read the following passage and solve
member should be an expert of only one area. the questions based on it.
26. If Cyriac does not like to be a member of panel with Ram, Seven persons A, B, C, D, E, F and G contested in a game show
and Unni was in a panel with Shekhar, then the expert who that had a total prize money of ` 14 lakhs. Every contestant won
did not participate in the interview was : some prize money and the highest prize money was ` 3.5 lakhs. No
(a) Pramila (b) Cyriac two contestants won the same amount of prize money. For every
(c) Bhushan (d) Any of these person the difference with the next highest and the next lowest
27. If Pramila did not participate in the interview, then who contestant was the same.
was the person most likely to be with Unni ? (i) E won ` 2 lakhs.
(a) Suresh (b) Bhushan (ii) B won more money than A.
(c) Pramila (d) Any of these (iii) The difference in prize money between B and A was the
least.
Directions (Qs. 28) : Study the following information carefully
(iv) The difference in prize money between D and F was not the
and answer the given questions given below :
least.
Following are the conditions for selecting Manager-HR in an (v) There was at least one person whose prize money was
organisation : between that of E and G.
The candidate must – 29. If the total money won by A and D is equal to that of G and
(i) be at least 30 years and not more than 35 years as on the difference between E and D is at least 1 lakh, then which
01.03.2012. of the following must be True?
(ii) have secured at least 60 percent marks in Graduation in any (a) A and B together won ` 3 lakhs
discipline. (b) B and F together won ` 3.5 lakhs
(iii) have secured at least 65 percent marks in the Post Graduate (c) C and E together won ` 3 lakhs
degree/ Diploma in personnel Management/HR. (d) B and C together won ` 3.5 lakhs.
(iv) have post qualification work experience of at least five years 30. If D won more than E, and B and G together won ` 3.5 lakhs,
in the personal/HR Department of an organisation. which of the following must be true?
(v) have secured at least 50 percent marks in the selection (a) D won ` 3.5 lakhs (b) A won ` 1.5 lakhs
process. (c) B won ` 1.5 lakhs (d) C won ` 50,000
In the case of a candidate who satisfies all the above conditions except-
EBD_7711
110

Hints and Solutions


1. (b) Here, S. P. of mixture = C. P. of pure milk = ` 3 per litre
15, 5
100 + 20 1, 3 15
Now, S. P. of mixture = ´ C.P. of mixture
100
48, 24, 16, 12, 8
48
3×100
Þ C.P. of mixture = =` 2.5 per litre
120
1, 2, 3, 4, 6
Also, C. P of water = ` 0
Thus, we observe that 3 + 8 = 11 and 5 + 6 = 11, so the
By the rule of alligation : possible side of the square is 11 units.
C.P. of pure milk C.P. of water 4. (d)
`3 `0 Sol. (5-6):

State Total Male Female


2.5 (mixture) A 14400 6000 8400
B 22800 13200 9600
C 8400 3600 4800
D 19200 10800 8400
2.5 0.5 E 26400 12000 14400
2.5 F 18000 9600 8400
\ Ratio of pure milk and water in mixture = = 5 :1 G 10800 4800 6000
0.5
For five litres of water, quantity of pure milk 5. (c) Total no. of female candidates appeared from states E
and G together = 14, 400 + 6000 = 20, 400
= 5 × 5 = 25 litres
2. (b) 1 man º 2 boys Þ (12 men + 18 boys) º (12 × 2 + 18) 20, 400 ´ 100
Required percentage = = 17%
boys = 42 boys. 1,20, 000
Let required number of boys = x. 21 men + x boys º (21
6. (d) The total no. of male candiates appeared in the
× 2 + x) boys = (42 + x) boys. examination from states B and F together
Less days, More boys (Indirect Proportion)
More hrs per day, Less boys (Indirect Proportion) = 13, 200 + 9600 = 22,800
7. (a) Number of all physics professors in age group
Days 50 : 60 ü
30
15 ïï 25 – 34 = 30% of 40 = ´ 40 = 12
Hours per day 9 : ý :: 42 : ( 42 + x ) 100
2 ï
Work 1: 2 þï Number of female physics professors in the age group
25
æ 15 ö 25 – 34 = 25% of 12 =´ 12 = 3
\ [50 × 9 × 1 × (42 + x)] = ç 60 ´ ´ 2 ´ 42 ÷ 100
è 2 ø
\ Number of male physics professors in the age group
37800 25 – 34 = 12 – 3 = 9
Þ (42 + x) = Þ 42 + x = 84 Þx = 42.
450 8. (b) Let the number of professors in the university = x
3. (b) As area of a rectangle is the product of its length and According to question
its breadth. i.e., Area = l × b, therefore, for the rectangle 2% of x = 10
with Area = 15 sq. units, there are two possible 10 ´100
solution-1 and 15 or 3 and 5. For the rectangle with Þ x= = 500
Area = 48 sq. units. There are possible solutions - 2
1 and 48, 2 and 24, 3 and 16, 4 and 12 or 6 and 8. Thus 9. (a) 180 km
the figure with possible sides is given below. A B
C
¬ 11.00 am Y
X 11.00 a.m. ®
111
Time taken by Y for distance cover from B to C with From (i), (ii) and (iii) if they are arranged in order of
stoppages ascending heights.
Diya < Tiya < Priya < Riya < Siya
æ6 1ö 24 + 5 29
= ç + ÷ hrs = = hrs. Option (c) states the ascending order correctly and
è5 4ø 20 20 hence, should be the correct option.
Say they cross each other at x distance from A 22. (a) Work with options we find that option (a) is correct.
x 29 120 - x
\ = + Op tion (a) A B C D
70 20 50
Beginnin g 110 10 10 10
x x 29 12 110 – 60 10 + 10 10 + 20 10 + 30
\ + = + A gave
50 70 20 5 = 50 = 20 = 30 = 40
12 x 29 + 48 12 x 77 20 + 5 40 – 5
Þ = Þ = D gave
350 20 35 2 = 25 = 35
50 – 15 25 + 10 30 + 5
77 35 A gave
\ x= ´ = 112.29 » 112 km = 35 = 35 = 35
2 12
23. (b) According to question
10. (b) Let there be w wide runs.
Byes = w + 8 R = y and y = 2G
Runs scored by batsmen = 26 w \ R = 2G
Total run = 232 Hence, the number of red balls is double the number of
or w + w + 8 + 26w = 232 green balls.
224
Þ w= =8 Sol. (24-25): Row-2
28 R L
6 I E
\ Run scored by Ram = ´ 208 = 96 G F
13 H T
11. (c) It is clearly suggested in the passage that the efficiency T Q P S R
of the institution depends on the way it is worked and
the character and wisdom of the men who work it. Hence, L B C D A R
E I
option (c) is the correct answer. G
F H
12. (a) The contextual meaning of the phrase 'watertight T
Row-1 T
compartments' is 'the activities of life unaffected by
public opinion'. 24. (d) D sits third from the left or second from the right.
13. (d) The author says that the politicians often talk about A sits to the immediate right of D.
introducing certain political and economic changes. S faces D.
However, the author contradicts their view as the A, the immediate neighbour of D and faces R.
efficiency of an institution depends upon other factors 25. (b) P faces C and Q is to the immediate right of P.
as well. Hence, option (d) is the correct answer. 26. (d) Three panels can be found as follows:
14. (a) In order to make earth a paradise to live in, everything (i) Unni – Shekhar
from political, moral, social and economic well being
Cyriac – Suresh
should be perfectly synchronized with one another.
Hence, option (a) is the correct answer. Bhushan – Ram
Pramila does not participate
15. (b) The dog was sturdy and short tempered.
OR
16. (d) The dog bit him more than he bit others in the family.
(ii) Unni – Shekhar
17. (c) The writer's dog had bitten at least forty people.
Cyriac – Suresh
18. (b) The passage highlights how with modern technology,
Pramila – Ram
serious life hazards are being accompanied.
Bhushan does not participate.
19. (b) The harmful effects of modern technology are
widespread and long lasting. OR
20. (a) Only first assumption is true. Second is in correct as (iii) Unni – Shekhar
conservation of flora and fauna can be done with Pramila – Ram
efficient management. Bhushan – Suresh
21. (c) From statement (1) when we arrange them in order of Cyriac does not participate.
ascending heights. 27. (b) The possible panel is:
Siya > Priya > Tiya ....(i) Cyriac – Shekhar
Similarly from statement (2) Suresh – Ram
Siya > Riya > Priya ....(ii) Unni – Bhushan
Similarly from statement (3)
Riya > Tiya > Diya ....(iii)
EBD_7711
112
Sol. (28) : (c) 29. (b) A + D = G; and E = ` 2 lacs, D = ` 3 lacs, this leaves
B and F together to win 3.5 lakhs.
Candidate Conditions 30. (a) If B + G = ` 3.5 lacs, and E = ` 2 lacs which leaves
(i) (ii) or (A) (iii) (iv) or (B) (v) D to get the maximum.
Ashok ü ü – ü – ü ü
Alok û ü – ü ü – ü
Swapan ü – ü ü ü – ü
Ashok Pradhan satisfies conditions (i), (ii), (iii), (B) and (v).
Therefoe, his case would be referred to President – HR.
Test–2 General Aptitude (GS Paper II)
1. The work done by a man, a woman and a child is in the ratio
of 3 : 2 : 1. There are 20 men, 30 women and 36 children in 100C
a factory. Their weekly wages amount to ` 780, which is P Q
divided in the ratio of work done by the men, women and
children. What will be the wages of 15 men, 21 women and

Temperature
30 children for 2 weeks?
(a) ` 585 (b) ` 292.5
(c) ` 1170 (d) ` 900
2. Two guns were fired from the same place at an interval of
10 minutes and 30 seconds, but a person in the train
approaching the place hears the second shot 10 minutes Time
after the first. The speed of the train (in km/h), supposing Which one of the following statements is correct?
that speed travels at 330 metres per second, is:
(a) During heating, liquid P remain hotter than liquid Q
(a) 19.8 (b) 58.6
throughout
(c) 59.4 (d) 111.80
(b) At no point of time during heating did the two liquids
3. The variations in temperatures form 0 degree C to 100
have the same temperature
degree C with respect to time of two liquids P. Q are shown
in the graph given below: (c) P attainded the temperature of 100 degree C faster than Q
(d) Q attaind the temperature of 100 degree C faster than P
Directions (Qs. 4– 5): Study the following graph carefully to answer the questions that follow.

80
State A
70
State B
60
Population in millions

State C
50

40

30

20

10

0
2004 2006 2008 2010 2012 2014
Years

4. What is the ratio of the average population of three


states in 2004 to that in 2008? Year 2012 2013 2014 2015
(a) 1 : 2 (b) 2 : 3 Pop ulation in
20 21 22 23
(c) 12 : 23 (d) 13 : 24 lakh s
5. What was the ratio of average population of state ‘B’ Inco me in
for 2006, 2008 and 2010 to that of state ‘C’ for these 1010 1111 1225 1345
crores (` )
three years?
(a) 5 : 7 Which one of the following statements is correct inrespect
(b) 25 : 28 of the above data?
(c) 7 : 9 (a) Population increased by 5% or more every year.
(d) 15 : 19
(b) Income increased by 10% or more every year.
6. The following table gives population and total income of
(c) Per capita income was always above ` 5,000.
a city for four years:
(d) Per capita income was highest in 2014.
EBD_7711
114
Directions (Qs. 7–8): Study the following data carefully to answer the questions that follow.
Details of Statewise distribution of candidates appeared in an Examination
Statewise distribution of candidates appeared
Ratio of male and female candidates appeared
Total candidates appeared = 1,20,000

State Ratio
G A
A 5:7
9% 12%
F B 11 : 8
15% B
C 3:4
19%
D 9:7
E E 5:6
C
22% D 7% F 8:7
16% G 4:5

7. The number of female candidates appeared from State ‘C’ PASSAGE-1


is equal to the number of male candidates apperared in the
examination from which state? There are no shortcuts towards the creation of world-class
universities. The ability of private higher education to attain the
(a) A (b) D
standards of their counterparts in the US and other parts of the
(c) E (d) G developed world will depend on the willingness of the
8. The total number of female candidates appeared from states philanthropists towards giving them the long-term cushion of
E & G together is what per cent of the total number of sustenance. Examples like BITS Pilani and the Tata Institute of
candidates appeared in the examination from all the states? Fundamental Research show such philanthropy in excellent light.
(a) 13 (b) 11 However, the pressure of revenue generation, along with the
(c) 17 (d) 15 demands of profitability, has pushed many a private sector
9. In the Suniti building in Mumbai there are 12 floors plus educational institution from deviating from its original lofty goal.
the ground floor. 9 people get into the lift of the building 11. The author's viewpoint can be best summed up in which of
on the ground floor. The lift does not stop on the first the following statements?
floor. If 2, 3 and 4 people alight from the lift on its upward (I) Profit making has hindered many private sector
journey, then in how many ways can they do so? educational institution in its main objective of
(Assume they alight on different floors.) imparting quality education.
11C × 3P 11P (II) Philanthropy plays a major role in creating a world-
(a) 3 3 (b) 3 × 9C4 × 5C3
class educational institutions.
(c) 10P × 9C × 5C3 (d) 12C
3 4 3 (III) Creation of world-class universities in a long term
10. In the figure below, the rectangle at the corner measures 10 process.
cm × 20 cm. The corner A of the rectangle is also a point on (a) Only I
the circumference of the circle . What is the radius of the (b) Both II and III
circle in cm? (c) Both I and II
(d) All I, II and III
A
PASSAGE-2
October 16 is observed as the World Food Day to mark the creation
of the United Nation's Food and Agriculture Organisation (FAO)
(a) 10 cm (b) 40 cm in 1945. The world body envisions a "zero hunger world" by 2030.
(c) 50 cm (d) None of these Perhaps, the occasion is incomplete without remembering Nobel
Peace laureate Norman E Borlaug, whose "miracle seeds" of wheat
Directions (Qs. 11–20): Read the given passages carefully and
test yourself, whether you can answer the questions based on saved over a billion people from starvation. Borlaug also instituted
each, correctly. the World Food Prize in 1986, which is sometimes described as the
Nobel Prize in agriculture. It's important to understand the role of
science and technology in ushering the Green Revolution, which
115
ensured food security in India. Today, similar innovations in PASSAGE-4
biotechnology hold the promise to provide nutrition security.
12. The author's viewpoint can be best summed up in which of When Jonathan (the seagull) came, it was well after dark, and he
floated in moonlight on the surface of the ocean. His wings were
the following statements?
ragged bars of lead, but the weight of failure was even heavier on
(a) Science and Technology plays a vital role in producing his back. He wished, feebly, that the weight would be just enough
food crops. to drag him gently down to the bottom, and end it all. But soon
(b) Norman E Borlaug was a pioneer in agricultural he came back to normal. He pushed wearily away from the dark
science. water and few towards the land, grateful for what he had learned
(c) Green Revolution was initiated by Norman E Borlaug. about work-saving low-altitude flying.
(d) World Food Day marks the unity of all countries of 17. The word 'wearily' means
the world towards food security. (a) tireless (b) exhausted
(c) sadly (d) unconscious
PASSAGE – 3 18. The seagull suffered because
It is said that ideas are explosive and dangerous. To allow them (a) he had tried to do something that other seagulls had
unfettered freedom is, in fact, to invite disorder. But, to this not done.
position, there are at least two final answers. It is impossible to (b) probably he had been attacked by a stronger bird.
draw a line round dangerous ideas and any attempt at their (c) probably he had been attacked by some strong creature
definition involves monstrous folly. If views, moreover, which in the sea.
imply disorder are able to disturb the foundations of the state, (d) he had swooned and fallen into the water.
there is something supremely wrong with the governance of the 19. 'His wings were ragged bars of lead' means that
state. For disorder is not a habit of mankind. We cling so eagerly (a) his wings were damaged and supported by bars of
to our accustomed ways that, as even Burke insisted, popular lead.
violence is always the outcome of a deep popular sense of wrong. (b) his wings were damaged and therefore very heavy.
13. What is the central point that the passage emphasizes ? (c) he had rags and bars of lead on his wings.
(a) It is unnecessary to define dangerous ideas
(d) his wings were broken like pieces of lead.
(b) Dangerous ideas are born out of the enjoyment of
20. The lesson that he had learnt that day was about
freedom
(c) A well-governed state is unaffected by dangerous ideas (a) not fighting with stronger birds.
(d) Dangerous ideas originate from man’s preoccupation (b) flying carrying bars of lead on his wings.
with politics (c) diving too deep into the sea.
14. From a close study of the passage, which one of the following (d) flying at low altitudes.
statements emerges most clearly ? 21. M is the father of N who is the son of V. In order to know the
(a) The author is against the exercise of political freedom relation of M to P, which of the statement/statements is/are
(b) He is indifferent to dangerous and explosive ideas necessary?
(c) He welcomes violence as a method to change 1. P is the brother of V.
governments 2. The daughter of N is the granddaughter of V.
(d) He warns that violence is the outcome of popular (a) Only (1) (b) Only (2)
dissatisfaction with the government (c) Either (1) or (2) (d) (1) and (2) both
15. The author says, “We cling so eagerly to our accustomed 22. Dinesh walked 30 metres straight and took a right turn and
ways”. Which one of the following statements may be walked 40 m. He again took a right turn and walked 50 m
considered as the assumption of the author ? towards west direction. Find the direction from which
(a) We are afraid of social changes Dinesh has started his journey also, find the distance from
(b) Mankind is averse to any disorder the starting point.
(c) We have developed inertia that makes us incapable of
social action (a) West, 20 5 m (b) North, 20 5 m
(d) There is an all round lack of initiative in the society (c) East, 20 5 m (d) None of these
16. Which of the following statements may most correctly bring 23. A box contains five sets of balls while there are 3 balls in
out the significance of the opinion of Burke quoted in the each set. Each set of balls has one color which is different
passage ? from every other set, what is the least number of balls that
(a) Burke advocated violence against injustice must be removed from the box in order to claim with
(b) Burke’s opinion coincides with the author’s opinion on certainty that a pair of balls of the same colour has been
explosive and dangerous ideas removed?
(c) Burke hated any popular uprising (a) 6 (b) 7 (c) 8 (d) 9
(d) Burke had no belief in political liberty 24. A monkey climbs 30 feet at the beginning of each hour and
rests for a while when he slips back 20 feet before he again
EBD_7711
116

Hints and Solutions


1. (c) Men Women Children
122500
Work 3 2 1 In 2014 ® p.c.i = > 5000
Numbers 20 30 36 22
Ratio of wages = (3 × 20) : (2 × 30) : (1 × 36) = 5 : 5 134500
:3 In 2015 ® p.c.i = > 5000
23
5
Total wages of men = ´ 780 =`300 Option (c) is correct.
13
\ Wages of a man = ` 15 Sol. (7–8):
Similarly, wages of woman = ` 10 State Total Male Female
and wages of child = ` 5
A 14400 6000 8400
Total wages of 15 men, 21 women and
30 children B 22800 13200 9600
= 15 × 15 + 21 × 10 + 30 × 5 = 585 C 8400 3600 4800
Total wages for 2 weeks = ` 1170 D 19200 10800 8400
2. (c) Let the speed of the train be x m/sec. Then, E 26400 12000 14400
Distance travelled by the train in 10 min. = Distance F 18000 9600 8400
travelled by sound in 30 sec. G 10800 4800 6000
Þ x × 10 × 60 = 330 × 30 Þ x = 16.5. 7. (d) No. of female candidates appeared from C = 4800
\ Speed of the train Following in the table for male candidates appeared
æ 18 ö in the examination from different states :
= 16.5 m/sec = ç 16.5 ´ ÷ km / hr = 59.4 km / hr.
è 5ø A B C D E F G
3. (d) 6, 000 13, 200 3, 600 10,800 12, 000 9, 600 4,800
20 + 15 + 30 65 13 Clearly that state is ‘G’.
4. (d) Required ratio = = = = 13: 24
35 + 40 + 45 120 24 8. (c) Total no. of female candidates appeared from states
35 + 40 + 50 125 25 E and G together = 14, 400 + 6000 = 20, 400
5. (b) Required ratio = = = = 25 : 28
40 + 45 + 55 140 28 20, 400 ´ 100
6. (c) Rule out options:- Required percentage = = 17%
1,20, 000
Option (a): - We clearly see that from 2013 to 2014,
9. (b) We just need to select the floors and the people who
æ 1 ö get down at each floor.
increase in pop is less than 5% ç ´100 < 5 ÷
è 21 ø The floors selection can be done in 11C3 ways.
Option (b): - from year 2014 to 2015 The people selection is 9C4 × 5C3.
Also, the floors need to be arranged using 3!
1345 - 1225
Increase in income = ´ 100 Thus, 11C3 × 9C4 × 5C3 × 3! or 11P3 × 9C4 × 5C3
1225

120 480 10. (c) 20


= ´4 = < 10
49 49 10
\ Not true r r–10
Option (c): per capita income (p.c.i.)
Total Income in an year r–20
=
Total population in that year.

101000
In 2012 ® p.c.i = = 5050 > 5000
20
In the right angled D, r 2 = (r - 20)2 + (r - 10) 2
111100
In 2013 ® p.c.i = > 5000 = 2r2 – 60r + 500
21
Solving, we get, r = 50.
117
11. (c) It can easily be inferred from the second sentence Now, any further removal of balls from any set will ensure
onwards. that removed ball is of the same colour as one of the
12. (a) This is the crux of the passage and it can clearly be already removed balls, thus constituting a pair of the
removed balls of the same colour.
inferred from the last two sentences of the passage.
13. (b) The author talks of dangerous ideas which he says are Set 1 2 3 4 5
born out of the enjoyment of freedom. Balls 1+1 1 1 1 1
14. (d) The author warns that popular violence is always the Hence, minimum no. of removed balls = 6
24. (c) Let ascent of the monkey in 1 hour = ( 30 – 20 ) = 10 feet.
outcome of a deep popular dissatisfaction with the So, the monkey ascends 90 feet in 9 hours i.e., 5 p.m.
government. Clearly, in the next 1 hour i.e., till 6 p.m.
15. (c) The author is trying to highlight the developed laziness The monkey ascends remaining 30 feet to touch the flag.
that makes us incapable of social action.
Sol. (25-26):
16. (a) Burke is highlighting the violence against injustice. (Columnist)
17. (b) The word ‘wearily’ means exhausted. (Chartered M
18. (d) The answer is given in the second and third sentences Accountant)E L (Engineer)
of the passage.
19. (b) ‘His wings were ragged bars of lead’ means his wings
were damaged and therefore very heavy.
(Lawyer) J K (Professor)
20. (d) The answer is given in the last sentence of the passage.
21. (a) M is the father of N and N is the son of V.
Hence, V is the mother of N.
From (1), P is the brother of V G H (Scientist)
Therefore, M is the brother-in-law of P because V is the (Financial F
wife of M. Analyst) (Doctor)
From (2), the daughter of N, is the granddaughter of V.
From this we do not get any relation of M to P. 25. (b) 26. (c)
22. (c) A 30 m B Sol. (27-29):
We prepare a table from the given data as under :
Maths Geography History PoliticalSci. Biology
40 m 40 m A ü ´ ü ´ ´
B ü ü ´ ü ´
West C ü ü ü ´ ´
E 20 m D 30 m C D ´ ü ´ ü ü
50 m
E ´ ´ ü ü ü
AE = 2
AD +DE 2 27. (b) Clearly, from the table D is intelligent in Political Science,
Geography and Biology.
= ( 40 )2 + ( 20 )2 28. (b) B is intelligent in Mathematics, Political Science and
Geography.
= 1600 + 400 = 2000 = 20 5 m 29. (c) A is intelligent in Mathematics and History but not in
Thus, Dinesh starts from east direction and distance Geography.
between his starting and end position is 20 5 m. 30. (c) Why has the suspension of flights been made for a
limited period of four days? It must have been assumed
23. (a) Set 1 2 3 4 5 by the authorities of X-Airlines that the crisis may be
Balls 1 1 1 1 1 over after this limited period. It is also possible that the
Pilots’ Association may withdraw the strike call within
four days.
EBD_7711

You might also like